Analytical Solid Geometry - 3d
Analytical Solid Geometry - 3d
Analytical Geometry
(For Degree and Honours Students of Indian Universities & for Various
Competitive Examinations like P.C.S.Sl JLA.S. etc.)
By
A .R . Vasishtha D .C . Agarwal
Retired Head, M.Sc., Ph.D
Department of Mathematics Retired Principal & Head,
Meerut College, Meerut. Department of Mathematics
S.S.V. (P.G.) College, Hapur.
&
A.K . Vasishtha
M.Sc., Ph.D
C.C.S. University, Meerut.
Chapters Pages
I. System of Co-ordinates 1 — 14
2. Direction Cosines and Projections 15 — 45
3. The Plane 46 — 87
4. The Straight Line 88 — 173
5. Shortest Distance 174 — 200
6. Volmne of Tetrahedron 201— 216
7. Skew Lines 217 — 233
8. Change of Axes 234 — 241
9. The Sphere 242 — 309
10. The Cylinder 310 — 324
11. The Cone ; 325 — 393
12. Central Ccohcoids 394 — 464
Systems of Co-ordinates
When these three axes are taken in pairs, they give us three
planes YOZ, ZO X and XOY.- T hest three planes are called yz* zx
and xy-planes respectively. The set of these three planes is called
the set of co-ordinate planes (rectangular).
The axes are oblique axes, if they are not rectangular.
Note. In the rest of the book the axes will be assumed to be
rectangular unless otherwise stated.
§ 3. Co-ordinates of a point in space. Consider a point P in
space. Through P draw a plane PNAM parallel to TOZ plane
l.e. perpendicular to x-axis meeting it in the point A ; if OA=*a,
then a is called the x-co-ordinate of P. Similarly through P
draw planes PNBL parallel to the plane ZOX and PMCL parallel
to the plane XOY meeting y and z axes in the points B and C
respectively ; if 0 5 = b and OC=c then b is called y-co-ordinate
of P and c is called z-co-ordinate of P. The three numbers a, b, c
are called the co-ordinates of the point P and are written by
Systems o f Go-ordinates 3
.
OXYZ
O X Y 'Z '
,ZA ,XO
Octant
O XY'Z'
O XYZ
O X Y 'Z
6* *
O O
I
I
X + + + + — — — —
y + — I _ ' + + I _ + —
I i
Z + + — . — + + — —
Draw the new axis PX1, PYi and PZi parallel to the original
axes OX, OY and OZ respec
tively.
The position vectors of
the points P and Q with res
pect to O as origin are given
by
OP^Xi l +y, J+zJs,
O Q = xti+ yti+ zJt.
Also the position vector
of the point Q with respect to
P as origin is PQ. Now we have
P Q ^ O Q -O P = (x,i + y J + Zjk) - (X1I + J 1J + Z1It)
=(X1- X 1) i+ ( y ,- y ,) J-Hr1- Z 1) k
=(*i-*i, y » -y u z»-zih
Therefore the co-ordinates of the point Q with respect to the
new origin P are (X1-X 1, y t —yu Zt - Z i).
§ 7. The distance between two given points.
Let P and Q be two given
points in space.
Let the co-ordinates of
the points P and Q be (X1, _ylt
Z1) and (X1, y2. zt) with res
pect to a set OX, OY, OZ of
rectangular axes. The posi
tion vectors of the points P
and Q are given by
O ^=X 1I+ ^ j+ z jk .
and O g —Xii+J'tj+Ztk
Now we have P Q = O Q -O P .
“ (•’f J + J 's H - i^ - iX ii+ ^ j+ z .k )
=(X1-X 1) Ι-ΚΛ-Λ) J-Hs1-Z 1) k.
Λ T Q = I T Q I—λ/ίί**- ·—^1) *Hz1-Z 1)1).
Thus the distance PQ between two points P (X1, yu Z1) and
Q (JU y*. 2») is given by
Systems o f Co-ordinates 7
or o k = V ± ° Q + m* OJ?_
OT1+W2
nr -. ■ - ■ - ,. (OT,Xa+WaX,) * +(OT1J^2 + OT2Vi) j + (W 1X ^ m 2Z1) k
(OT1+ OT,)
(using (I), (2) and (3)]
Comparing the coefficients of i, j, k, we get
S q OT1X 2 +O T8X 1 OTi y 2 + W 2-V1 , OT1Z8 + OT8Zi
“ OT1-J-OT2 ’ y OT1+ OT2 * OT1+ OT8
Cor. I. The middle point of the segment PQ is obtained by
PUttingm1-OT2. Hence the co-ordinates of the middle point of
PQ are (} (X |+ x2), i Cv1+.V2), $ (Z1+ z 2)).
Cor. 2. If mt μ : I, then the co-ordinates of the point
P flfc ( X i + μ χ * >'i + μ}'I Z\ + μζ2\
* a te [ - j + r · μ + l ' μ + ί J
8 Analytical Geometry 3-2)
3 *H -X s±*, + hX|
Xi+*»+** -I-Xt
3c or X= 4
3+1
Similarly y = \ (^l+ ^ i+ y * + ^ )* (*ι+**+*·Ψί»)·
10. (A) Spherical polar co-ordinates.
Let X'OX, T O Y and Z OZ
be the set of rectangular axes.
Let P be a point in space.
Draw PN perpendicular from
P to the xy-plane. The posi
tion of P is determined if the
length OP, angles ZOP and
XON are known. Suppose
OP=r, Ζ.ΖΟΡ=θ and L X O N
measured positively in the
directions shown by arrows
in the figure. The quantities
r, Θ, <f>, defined as above, are
called the spherical polar co-ordinates of P and are written as
(r, $, φ).
Now we shall find relations between these co-ordinates and
cartesian co-ordinates. Let (x , y, z) be the cartesian co-ordinates
of P. Hence we have
Z=PAT=OP cos ( L O P N )= r cos (LZO P)= T cos 0. ...(I)
Also ON=OP sin LOPN=T sin 0 [ 7 LO N P = 90°]
Λ X=O N cos φ = τ cos <f>sin 0, ...(2)
and y = ON sin Φ= r sin φ sin 0. ---(3)
Thus relations (2), (3) and (I) give the relations between
x, y, z and r, 0, φ.
Now squaring the relations (2) and (3) and adding, we get
x'+j)*=aOA/*, or u*=x2+y* where u=ON
ot tf( x t + y*)=u=r sin 0. ...(4)
Dividing (4) by (I), we get tan 0=V'(**+X2)/z·
Dividing (3) by (2), we get tan φ= γ/χ.
Squaring (I) and (4) and adding, we get x i + y, + zi =r*.
Thus the relations between spherical polar co-ordinates and
cartesian co-ordinates are
x = r cos φ sin 0, y = r sin φ sin 0, z= r cos 0
xt + yi + 2t = ri, tan 0=-^(**+.?2)/*. tan Φ=υ Ιχ ·
10 Analytical Geometry 3-D
Exercises
1. Find the locus of a point P which moves in such a way that
its distance from the point A («, v, tv) is always equal to a.
Ans. Jc*+?*+**-lu x —2vy—2ivz-fu, + v1+ h’' —a*=0.
2. The axes are rectangular and At B are the points (3 ,4 ,5 ),
( —1, 3, —7). A variable point P moves such that (i) PA=PB
and (ii) PA*—PBt = Ik i. Find th e lo cu so fP in each of the
above cases.
Ans. (i) 8x+2.y+24z+9=0. (ii) 8 ^ + 2 ^ + 2 4 7 + 9 + 2 ^ = 0 .
3. Show that the points (1,2, 3), (2, 3, I) and (3, I, 2) form an
equilateral triangle.
Hint. Show that the length of each side of the triangle is V 6:
4. Prove that the three points A, B and C whose coordinates are
( 3 ,2 , - 4 ) , ( 5 , 4 , - 6 ) and ( 9 ,8 ,- 1 0 ) respectively are
collinear.
2
Direction Cosines and Projections
§ I. Angle between two non-coplanar (i.e. non-intersecting lines).
Let PQ and M N be two non-coplanar lines. The angle
between two non-coplanar lines PQ and M N is equal to the angle
between two straight lines OA and OB drawn from any point O
parallel to PQ and MN respectively. Thus the angle between
Λ*
The relation (2) shows that the direction cosines o f the tine OP
are the coefficients o f I, j, k in the rectangular resolution o f the unit
vector in the direction o f OP.
Thus if I, m, n are the d.c.’s of a line, then a unit vector
along that line is /i+ m j+ nk.
Λ I /i+ m j+ n k 1=1
or V ift + « * + n*) = I, or •/•+ « •+ a * —I.
§ 5. Direction ratios.
Definition. I f the direction cosines I, m, n of a given line be
proportional to any three numbers a, b, c respectively, then the
numbers a, b, c are catted direction ratios (briefly written as d.r .’s)
o f the given line.
Relation between direction cosines and direction ratios.
' Let a, b, c be the direction ratios of a line whose d.c.’s are
/, m, n. From the definition of d.r.’s, we have
l/a=m /b—nlc=k (say). Then I=Icai m=*kb, n—kc.
But /«+m*+»*=!·
Λ k* (a*+b*+c*)=l, or k*= l/(a*+b*+c*)
or k ^ ± l / y / ( a t +b, +c*).
Taking the positive value of k, we get
. a b c
" V io '+ b '+ c * )' '” = V (a*+ b*+ cV
Again taking the negative value of k, we get
—a -b —c
v V + ^ + c * ) ’ m ~ y(a * + b '+ c* y n=V ( a * + b * + o '
Remark. Direction cosines of a line are unique. But the
direction ratios of a line are by no means unique. If a, b, c are
direction ratios of a line, then ka9 kb, kc are also direction ratios
of that line where k is any non-zero real number. Moreover if
a9 b, c are direction ratios of a line, then a i+ ij + ck is a vector
parallel to that line.
Rule. Let a, b, c be the d r.'s of a given Iine9 then, to find
actual direction cosines o f this line, divide each o f a, b, c by
V X a'+ ^ + c2).
SOLVKD EXAMPLES 2 (A)
Ex. I. Find the d.c.’s o f a line whose direction ratios are
2, 3, - 6 .
Solution. We have Vt(Z)*4-(3/ + ( - 6 ) 1}= vM -t-9+36)-*7.
2 3 _^
Hence (by § 5) the d.c.’s of the given line are ψ , ψ , - y ·
18 Analytical Geometry 3-D
—. m 2 - I m . .
Thus — = 3- γ α ~2 £lVM18
I m n \/{1*+ιη*-\-η%) I
Τ β Τ β Τ β ν ΐ ΐ β+ 2 · + > ί = V (M /
Λ The d.c.*s of the other fine are l/\/(14). 2 / ^ ( 1 4 ) , 3/vO 4)·
Ex. 5. Find the direction cosines It m, n o f the two lines which
are connected by the relations /+ιπ+#ι=»0 and mn—2nl—21m=0.
(Meerut 1985)
SeL The given relations are
/+#n+«=»0 or / = —iff—λ ---(I)
and mn—2nl —2l m=0. ---(2}
Puttingthevalue of I from (I) in the relation (2)# we get
mu—2ι» (—jw—it)—2 (—m —n) m = 0 or 2^ + 5^ 111+ 21^-=0
or (2jr+ ji) (m +2n)=0.
in I
— =- a nd—2.
n 2
From (I), we have — «=>————= ——— I . /TI
v' n n « —P )
Now when m /n=—+ (3) gives //n = J —1 = —
Λ #π /1= π /-2 and //I = 11/—2
. I m n \/(J1+ lll¾+ ,1,) *
- I ° r = - 2 = y r{ l '+ i , +(-2)*>
The d.c.*s of one line are 1/^6» I/V r6, —2/\f6.
Again when m /2 = —2, (3) gives //a=»2 - 1 —1.
I m n y/(P+nP+iP) I
" I = - 2 = I = ^ (^ + (-2 )-+ 1 * }= ^
The d.c.*s of the other line are 1/ λ/6> -2 /-^ 6 , 1 /^ 6 .
§ ¢. Prejection of a point on a given line.
Let P be a given point and AB the ^
given straight line. Draw perpendicular
P M fto m P to ABt meeting AB in M.
Then the foot of the perpendicular M is
called the projection of the given point
P on the given line AB. e ~A
The point M is the point in which the
plane through P and perpendicular to
AB meets the line AB.
§ 7 . - —j—**— « r - —c----it y fa liar ta ansthrr Mae (in the
line on another given line AB. Let the points C , D' be the
projections of the points C, D on the line AB, then the segment
(4, 6, 3), (—1, 2, 4), (I, 0, 5) respectively, then find the projection
o f PQ on RS. Also find the projection o f RS on PQ.
Sol. To find the projection of PQ on RS, we should find the
d.c.’s of RS.
The direction ratios of RS are
1 - ( - 1 ) , 0 - 2 , 5 - 4 Ie., 2, - 2 , I.
Also /{ 5 = - ^ ( ( 1 - ( - 1 ) ^ + ( 0 -2 ) ^ + ( 5 - 4 ) ^ = - ^ ( 4 + 4 + 1 ) = 3 .
d.c.’s of RS are 2/3, -2 /3 , f/3.
Hence the projection of PQ on RS i$ (See § 11)
= / (Xi-X,) + W (>*-3’i) + n (z»-2|)
= } (4 -3 )-1 (6 -4 )+ } ( 3 - 5 ) = f - |- i = - f Ans.
Again to find the projection of RS on PQ, we should find the
d.c.’s of PQ.
The direction ratios of PQ are
4 - 3 , 6 - 4 , 3 - 5 i.e., I, 2, - 2 ,
Also PQ = v/{(4 - 3)*+(6 T4)2+ (3 - 5)2}= V ( I + 4+ 4 )= 3 .
Λ the d.c.’s of PQ are 1/3, 2/3, -2 /3 .
Λ projection of RS on PQ is (See§ 11)
\ = I 0 - ( - 1 ) } H ( 0 - 2 ) - 1 ( 5 - 4 ) = } - $ - } = - }· Ans.
Ex. 4. I f P, Q, R, S are four points with co-ordinates
(2, 3 , - 1 ) , (3, 5 , - 3 ) , (I, 2, 3), (3, 5, 7) respectively, prove by
projections that PQ is at right angles to RS.
Sol. In order that PQ is at right angles to RS, the projec
tion of PQ on RS should be zero.
The direction ratios of RS are 3—1, 5 —2, 7—3, i.e., 2, 3, 4.
Also RS= V ( O - I ) ' +(5 -2 ,2+ (7 —3)2}
—^ (4 + 9 + 1 6 ) = ^(29).
.·. d.c.’s of RS are 2/^(29), 3,V(29), 4/y(29).
Hence the projection of PQ on R S is (See § 11)
Lel AB and CD be two given lines whose d.c.’e are Iu i»i* *1!
and It, m2,n 2 respectively. Through the origin O draw lines OP
and OQ parallel to AB and CD
respectively so that direction
cosines of OP and OQ are
Iu mu n,; and /*, m2t H2 respec
tively. Take a point P(xu yu zt )
on OP such that OP=Ti. Since
the d.c.’s of OP are Iu mit nu
therefore the co-ordinates of P
may be written as
Viru HJiTit H1T1).
Λ X1- I 1r „ Xi-Tn1Tit Zi = HlTi. -(1 )
Similarly ir we take a point Q (x2, y t, z%) on OQ such that
OQ=T2tIhea
Xi^U ru Λ = IH2Tt , Zt=H2T2. —(2)
Let Θ be the angle between the lines AB and CD, then 9 is the
angle between OP and OQ.
Now the projection of OQ on OP
—Ii (Jfa- O)-Hiw1 ()'i - 0 ) +W1 (Z| 0)
IiX2+ M 1X2 + I i i Z2 — ( 3)
But the projection of OQ on OP=OQ cos θ= τ2 cos Θ. ...(4)
From (3) and (4), wc have r2 cos $—I1X2+/w ^»+H1Z2
r2 cos Θ= I1I2T2+TniIn2T2-^-HlTt2T2 using (2)
or cos Θ- I 1I1+In1In., -I niD2. -(5 )
Remark If Θ is the acute angle between the two lines, then
cos Θis + ive and so we have
cos Θ==| IiI2-^miHi21 Hin2
Corresponding formula when direction ratios of the iines are
given. Let O1, bit C1 and a2, b2, c2 be the direction ratios of the
two given lines . Then their actual direction cosines are given by
Oi bj _ _ __ Cl _____
and
V i tOi 2 HV + c, V V W + ^ + C i 2') · V iO i'+ bS+ cS)
_____ Q2 __ b2 ______ c*______
v W + V r c r V V io 2*+ b j+ c 2·) * *
Using these values of d.c.’s in (5), the augle 0 between these
two Iines is given by
a,a,-H b,b* r C1C1
cos Θ«
+ ΒΛΗ^ι *) V +Ba2-HC1*) ...(6)
26 Analytical Geometry 3-/)
i \ k
/. n1
It m% W2
C-(W Iifia -W a W i) i+ (V 2 -w2/|) j + (ZifW2 - Z 2W1) k.
sin* 0 = (a x b )* - (axb )* (ax b )
= (fWiWa—fWaWi^+ifii/a-Wa/i^+C/ifWa- Um i ) 1.
Condition for perpendicularity. Thetwo lines are perpendi
cular if and only if a«b=0
i.e., iff (/ii+w ij + wik)«(/ii + msj+ *sk)= 0
i.e. 9 iff /i/2^fw1W2H-Wjff2=O.
Condition for parallelism. The two lines are parallel if and
only if the vectors a and b arc collinear
i.e.y iff a = Ab, where A is some scalar i.e . 9 real number
i e . 9 iff /!! + WjjH-W1Ii = A (/2i + w2j + w2k)
Le ., iff Zi=AZ2, /Wi= Aw2t Wi = Aw2
W|
i e . 9 H r i- = S . 1W2*
U W2
(ii) To find the angle between two lines whose dr.'s are a1$ bl9
Ci and a%9 Zf2, c2.
We have
A = a vector along the line whose d.r.’s are al 9 Dl 9 C1
—^ii + Z/ij + Cik,
B = a vector along the line whose d.r.’s are a2t b2%C2
= u2i + Zf2j + c2k.
Now proceed as in case (i). Here | A 1=^/(^1*+¾1+ ^ 1) and
I B !---v W + W + r?2)·
§ 3. To find the perpendicular distance of a point P (x\ y'f z ’)
from a line through A (a, b, c) and whose direction cosines
are I, m, n
Let AB be a line through A (a9 b9 c) and whose d.c.’s are
I9 m9 n. Let IjN be the perpendicular from P to AB.
Now AN-- projection of the
line segment joining A (at b9 c)
Direction Cosines and Projections 29
We have, PN9= A P t - A N t
or /W*={(x'-o)*+(/-Z>)*-f(2'-c)*}
- { ( x ' - o ) l- H y '- b ) m + iz’- c ) »}*
or FAZ*={(*'—«)*+(/—6)·+ (z’—c)1) {/*+ ot*+ n*} -
—((Xi- a) / + ( / —6) m+(2’—c) n}*
= Σ { ( / —b) n—(zr—c) ot}* [by Lagrange's identity]
Λ JW =Vt? {(/-6) O -(Z t-C ) ot}*].
Aliter. Let /,P A N = » .
We have, P N t = A P t sin* 9.
Now 6 is the angle between the lines AP and AB. Here
the d.c.’s of AP are (x’-a )IA P , (y ’-b )/A P , (Zr-C )IA P
and the d.c.’s of AB are /, ot, n
I (y r-b )IA P (Zr-C )IA P t
Λ sin* tf=|·
: OT /I
- ( - l ) ( - ^ ) +( - l ) ( - ^ H l ) ( i b )
R=Cos"1^(2/3).
Ex. 4. Prove that the straight lines whose direction cosines are
given by the relations al+bm+cn=Q and fmn+gnl + hlm=0 are
perpendicular If fla+ glb + h/c=0
and parallel if V (af ) ± V ( b g ) ± \/( ch) = Q-
(Meerut 1984, 85 P, 87, 89)
Sol. From the first relation, we have n= —(al+bm)/c.
Putting this value of n in the second relation, we have
al+bm\
C
or afml + bfm2+ agl2+ bglm - chlm = O
or ag —a + — (af+ bg-ch) + bf= 0.
ml m ...(D
Now if Iu mu W1 and I29 w2. n2 are the direction cosines of the
two lines, then the roots of (I) are Iijm1 and /2/w2.
. Λ Γα. ' h h bf Λ/a WiiW2
product of the roots= —. — = — or -?- = — rr
nil wa ag fja gb
/|/2 W1W2
' m - by ^mmetry.
( f!°) (gib)
We know that the lines are perpendicular if
/1/2+ WjW2+ WiW2=O
i e if f/a+glb + hlc=09 which proves the first part.
If the lines are parallel then the direction cosines are the
same. This shows that the roots of (I) are equal, for which the
condition is tB7=AAC9
i.e. (af +bg - ch)2=4ag.bf.
32 Analytical Geometry 3-D
-I.
Hence in this case the actual direction cosines I9 w, n are
/W1W2-W 2W1, Wj/2—w2/lf /JW2-Z 2Wi. Proved.
Ex. 10. Show that the direction cosines o f a line perpendicular
to a pair of mutually perpendicular lines with direction cosines as
Iu mu W1 and /2, w 2, W2 respectively are
WiW2 -W2W1, Wj/2—w2/i, Z1W2-Z2Wi. (Bhagalpur 1966)
Sol. See example 9. This is other way of stating the same
problem.
Ex. 11. Prove that three concurrent lines with direction cosines
Iu Ήι, Z2, w2, w2; and /3, w3, w8 are coplanar if
A W i Wi
/* W 2 W2 = 0 .
/. W 3 W3 ( M e e r a t 1983 S )
Ex. 12. Prove that the three lines drawn from a point with
direction cosines proportional to I, —I, I; 2, —3, 0, and I, 0, 3 are
coplanar.
Sol. Let a9 b9 c be the direction ratios of the normal to the
plane in which the two concurrent lines with direction ratios
I, —1 , I and 2, —3, 0 lie.
Clearly these lines will be perpendicular to this normal. Hence
applying the condition for perpendicularity of two lines, we have
1.0 + ( - 1 ) .6 + 1 .C=Of ’ ...(I)
36 Analytical Geometry 3-D
and 2 . e + ( - 3) .h + 0 .c = 0. ...( 2)
„ , . .. a b c a b c
Solvngtbese. ¢ + 3 - 5^ - - 3+2 » ' 3 - f ^ l
Again the third concurrent line with d.r.’s I, Ot 3 will lie in
this plane if the normal with d.r.’s 3, 2, —1 is also perpendicular
to this third line.
We have (3) (1)+(2) (0 ) + ( - 1 ) ( 3) = 0, showing that the lines
with d.r.’s 3, 2, - I and 1 ,0 ,3 are perpendicular. Hence the
three given lines are coplanar.
Ex. 13. The d.c.’s of two intersecting lines are Iu f?i and
Uf Ff1 Show that all lines through the intersection o f these two
whose d c ’s are proportional to I1+ kl3, mi + km%%nx+ kn% are cop
lanar with them.
Sol. Let /, FFf, π be the d.c.’s of the normal to the plane in
which two intersecting lines whose d c.’s are I1^mu nx and
U$ Fff1, π* lie. Clearly these lines will be perpendicr.hr to this nor
mal. Therefore /iZ+Fn^+FiiFf —O, ···(*)
and V+Fff1FH+ Ff2Fi=O. ·*·(2)
Now any line through the point of intersection of these lines
with d.c.’s proportional to h + ktu mt + kmt, nx+kn2 will lie in
this plane if the normal to this plane is also perpendicular to this
line.
We have l(li+ klt)+ m (m i+ km 2)+n(nl +kni)
s = lli+ m m i+ nnl -\-k{IU + mmt +nni)
= 0 +A:.0, using (I) and ( 2)
= 0.
This shows that the line whose d.c.’s are I, m, n is perpendi
cular to a line whose d.r.’s are l\+ klit m i+ km 2, nx+kn2.
Hence all lines through the intersection of the given Iinesand
with d.c.’s proportional to lt+ klt, m ,+ km a, nx+ kn2 are coplanar
with them.
Ex. 14. Ifa variable line in two adjacent positions has direction
cosines I, m, n and 1+81, m+8m, n+8n, show that the small angle
89 between the two positions is given by (80)*=»(8/)*+(8m)*+(8n)*.
• (Meerut 1984 P, 87 P, 88)
Sol. Since l,m ,n and (1+81), (m + 8m), (η+δ/ι) are the
actual direction cosines, we have /*+m *+n*=l ·- (I)
and (l+8l)*+(m+8m)*+(n+8nj*=* I
or (/*+ m*+ π»)+ 2 /8 /+ 2m8m+ 2πδπ+ (81)*+ (8m)* + (Sn)*= I
Direction Cosines and Projections 37
:. 9—π IS.
Ex. 16. Show that the area of a triangle whose vertices are
the origin and the points (xlt y t, zt) and (xt, >’*, z*) is
3S Analytical Geometry 3-D
Le., ( - r - ’ r)·
Λ the direction ratios of the external bisector PM are
2, i i . 5.
Le., are —11. 20, 23.
Ex. 20. I f the edges o f a rectangular parallelopiped be a, b, c
show that the angles between the four diagonals are given by
-1 / ± β * + 6*±ο »1
'+ b ' + c*)/
Ita*+
(Kanpnr 1982, Meerut 84 S)
Sol. Let O, one of the vertices of the rectangular parallel
opiped, be takeu as origin and the three coterminous edges OA,
OB and OC as the co-ordinate axes.
Direction Cosines and Projections 41
§ I. Plane.
Definition. A plane is a surface such that all the points of a
straight line joining any two points on the surface lie on It. Or in
other words if we take any two points on the Surface9 the straight
line joining these two points wholly lies on the surface.
§ 2. (A) The equation of a plane. (Normal form).
Tofind the equation of a plane in terms of p l.e.9 the length of
the perpendicular from the origin to the plane9 and direction cosines
I9 m9 n of this perpendicular.
Let OX9 OYt OZ be a set
of rectangular axes with O as
origin. Let p be the length
of the perpendicular ON
from the origin O to the given
plane ABC. We shall take
p always positive. The d.c.’s
of the perpendicular ON are
I9 m9n. The direction of
AX
Xi y\ Zl I
Xt y* Zi I
Xi y* Zl I
Cor. Conditionforfour points (xlt y u Z1), (x„ y t , zt), (x*. y», z*)
and (x<t ylt Z4) to be coplanar.
The equation of the plane passing through first three points
is given by equation (5). If the fourth point namely (x4. yt, z4)
also lies on this plane, then the co-ordinates of this point will
satisfy the equation (5), so that we have
Xt y* Zt I Xi y\ Z1 I
Xi y\ Zl I Xi y* Zi I
=0, l.e..
Xt y% Zl I X% y* Z9 I
Xi yi Zl I Xt y« Zt I
- ( 6)
52 Analytical Geometry 3-D
or 2x+ 3 y —5z—38=»0
or 2 r+ 3 p —5z=38.
Ex. 3. O fstheoriginand A is the point (a9 b9 c). Pindtbe
i.c's of the join OA and deduce the equation o f the plane through A
at right angles to OA.
Sol. The co-ordinates of the points O and A are (O9 O9 0) and
(o9b9 c) respectively. Thus the direction ratios of the join OA are
a—O9 b —O9 c —0 i.e.9a, b9 c.
Hence the d.c.’s of the join OA are
alV (a'+ b'+ c*)9 b/V<d*+b*+c«), dV (a*+ P+ c*).
Now we are to find the equation of the plane passing through
the point A (a, b9 c) and perpendicular to the line OA. Here the
line OA is normal to the plane and its d.r.’s are at b9 c. Therefore
the equation of the required plane is
a(x—a)+ b(y—b)+ c(z—c}= 0. or ax+by f cz=a%+b%+<*.
Ex. 4. Find the equation of the plane perpendicular to the line
segment from (—3, 3, 2) to (9, 5, 4) at the middle point o f the
segment.
Sol. The end points of the given line segment are ( —39 3, 2)
and (9t 5t 4). The d.r.’s of the line segment are 9—(—3), 5—3,
4 -2 i.c., 12, 2, 2. The co-ordinates of the middle point of the
line segment are ($(9-3), $(5 + 3), $(4 + 2)) i.e.9 (3, 4, 3).
Thus the plane is to pass through the point (3, 4, 3) and d.r.’s
of normal to the plane are 12, 2, 2. Therefore the required equa
tion of the plane is
12 (jc—3 ) + 2 ( ^ - 4)+2fz—3)= 0
or 6(x —3 )+ 0 > -4 )+ (z —3)=0, or 6x+.y+z=25.
Ex. 5. Find the intercepts made on the co-ordinate axes by the
plane x —3y+2z=9.
Sol. The equation of the given plane is
*-3>>+2z=9. ...ti)
Dividing both sides;by 9· the equation (I) may be written as
x_ Z - z »1 .
o_ + -3 + ( 9 /2 ) ' ..(2)
Comparing the equation (2) with the equation
x/a+y/b+z/c=-I 9
he intercepts on the co-ordinate axes are given by
a= th e intercept on the x-axis=9
b = the intercept on the j-axis®»—3
nd c—the intercept on the z-axis·*9/2.
54 Analytical Geometry 3-D
Ex. 6 . A plane meets the eo-ordinate axes in A, B. C such that
the centroid o f the triangle ABC is the point (p, q, r)i show that the
equation o f the plane is x/p+ ylq+ z/r= 3 .
(Ka spar 1983: Mecrat 85S}
Sol. Let the equation of the plane be
x/a+ylb-\-z/c= l. —(I)
The plane (I) meets the x-axis in the point A, so putting y = 0
and z = 0 in (I) we get x= a. Thustbe co-ordinates of the point
A are (a, 0, 0). Similarly the plane ( I) meets y and z axes in the
points B and C whose co-ordinates are given by B (0, b, 0) and
C (0, 0, c).
Thus the co-ordinates of the centroid of the triangle ABC are
given by (J(e+ 0 + 0), J(0 + i + 0), |( 0 + 0 +c)) i.e., (Jo. Jb, Jc).
But it is given that the co-ordinates of the centroid of the
triangle ABC are (p, q, r), so that we have
P = Jo, q= {b, r = Jc or o=3p, b=3q, c=3r.
Substituting these values of o, b, c in the equation ( I ), the
equation of the required plane is given by
xH3p)+yH3q)+zl(3r)=‘l or x/p+ylq+z/r = $·
Ex. 7. Find the equation to the plane through the three points
(0. - I. - I ) , (4, 5, I) and (3, 9. 4).
Sol. The equation of any plane passing through the point
(0, —I. —I) is given by
o(x—0) + H p - ( - 1)}+C{ z - (—I )}=0
or ox+*(^ + l)+ c(z-|-l)= 0 . ...(I)
If the plane (I) passes through the point (4, 5, I), we have
4a+ 6b+ 2c= 0. ...( 2)
If the plane (I) passes through the point (3, 9, 4), we have
3o+10b+5c=0. ...(3)
Now solving the equations (2) and (3), we have
o ___ b c _ 4
3 0 -2 0 “ 6^ -20 “ 4 0 - I 8 =A I8ay*
Λ o=10A, b = —14A, c=22A.
Putting these values of a ,b ,c in (I), the equation of the
required plane is given by
A[10x—l4(_y+l)+22(z-!-l)]=0
or 10x-14O> + l)+ 2 2 < z+ l)= 0
or 5x—7 y + llz + 4 = 0 .
Ex. 8. Show that the four points (0, —1, —1), (4, 5, I), (3,
9 ,4) and (—4, 4, 4) are coplanar. (Meerut 1982)
The Plane 55
The d r.’s of the normal to the plane (I) are au bu C1 and the
d.r.’s of the normal to the plane ( 2) are ut, b2t c%.
~ If 0 is the angle between the planes (I) and (2), then 0 is the
angle between the lines whose d r.’s are ult bit Ci and a%, b2fc2·
. _ QlQ t+M a+ CiCt________
V (ei* + V + Ci*> V (ei* + V + Cie) -(3)
For the acute angle between the two planes, cos 0 is positive
and for the obtuse angle it is negative. The numerical value of cos 0
in both these cases is the same because cos (π—0; = —cos 0.
Condition of perpendicularity of two planes.
Two planes are perpendicular if their normals are perpendi
cular. Therefore the planes (I) and (2) are perpendicular if the
lines whose d.r.’s are U1, A1, C1 and u8, bit c8 are perpendicular the
condition for Which is
Uiat + btb2H- C1C2« 0. · · -I*)
Condition of parallelism of two planes.
Two planes are parallel if their normals are parallel. There
fore the planes (I) and ( 2 ) are parallel if the lines whose d.r.’s are
u,, hi, Ci and a%9 ha, c2 are parallel the condition for which is
ClIci ..<(5)
l.e., the coefficients of x, y, z in the equations of the two-planes
should be proportional.
Remember. The equation of any plane parallel to the plane
ax+ by+ cz+ d^Q
is ax+ b y+ cz+ X ^ 0.
$ 11. The two sides of a plane.
Two points P (xlt y u Z1) and Q (x2, yt , zt) lie on the same or
opposite sides of the plane ax + by+ cz+ d—U according as
axi+ byi+ czi+ d and axt -{-byt +czt + d
are o f the same or opposite signs.
T h eeq u atio o o fth ep lan eis ax+by+cz+d=Q .,.(I)
Suppose the line PQ meets the given plane (I) at the point R
such that PR : RQ = Zzi1 : mt. Then the co-ordinates of the point
R afe Zm1Xt + mtXx m,yt + mty, Qt1Zfhm tZ1 V
\ mx+mt ' AZi1 + mt ’ /Ziii-AZit /
Since the point R lies on the plane (I), therefore
Q ZmlXt +ZZttXi \ - b / z/ι, Vt +ZZit y,\ c / zziizt + ZZitZi \ .
\ /zii+zzit Z*1" \ zzii r zzit I zzti + /zit . / +
OV ml(axi +byt + czt +d) + mt(axx+ byx+czt +d)----0
The Plane 57
/Mt α χ ,+byy+czi+d
m% Qxi ^ b y i + cz*-t-d ...( 2)
Now the ratio Otxjmi is positive or negative according as PQ
is divided at R internally or externally Le., the points P, Q lie on
the opposite or same side of the plane (I).
Hence from (2) if axx+ by\+czx+d and axt+ byt+ czt+ d are
of the same sign, then r»i/ma is negative Le., the points P and Q
Iieon th e sam esid eo f the plane (I) If αχχ+byx+CZl -^d and
axt+ byt+ czt+ d are of the opposite signs, milmt is positive Le.,
the.points P and Q lie on the opposite sides of the plane (I).
§ 12. To find' the length of the perpendicular from the point
(Xi, yi, Zr) to a given plane.
Let the equation of the given plane be
a x+ b y+ cz= d = 0. ...(I)
To find the length of the perpendicular from the point
(*i, yi, zi) to the plane (I).
Shifting the origin to the point (JC1, y x, Z1), the equation (I)
becomes a (x+xjJ + fi (y+ yi) + c (z + r,)+ d = 0
or ax+by+cz+axx+byx+czx+d=!). . . ( 2)
Dividing both sides of ( 2) by y/(a*+b*+c*), we get
__ a . b c
V(o*+b*+c2) X+ y / i ^ +b' +c *) y + 2
, Ox1^ b y l +Cz1+ d
V l* + * * + * 1) " * ...(3)
The equation (3) of the plane is in the normal form with a
proper adjustment of sign throughout the equation.
The length of the perpendicular from the new origin to
the point (I, —2. —5), therefore, the equation of the required
plane is given by z = —5 or z+ 5 = 0 .
Remark. If the axes of x and z be taken in a horizontal plane
(the plane of the paper, say) then .y-axis is perpendicular to this
plane and hence the equation of the required horizontal plane will
be y= —2 .
Ex. 3. Find the equation of the plane through the point
(—I, 2, 4) and parallel to the plane 2x+3.y—5 z+ 6 = 0 .
Sol. The equation of the given plane is
2jc+3.y—5z+ 6= 0. --(I)
Since the equations of the parallel planes differ only in the
constant term, therefore the equation of any plane parallel to the
plane (I) is given by
2x+3y—5z+ J r= O . --(2)
If the plane (2) passes through the point (—1,2, 4), we have
2 ( - 1 ) + 3 (2 )-5 (4)+fc=0 or * = 16.
Substituting this value of k in the equation (2), the equation
of the required plane is 2x + 3.y—5z+16=0.
Ex. 4. Find the equations of the planes paraliel to the plane
3x—fry—2z 4 = 0 at a distance 3 from the origin.
Sol The equation of any plane parallel to the plane
3x—6y—2z—4 = 0 is 3 x -6 y —2z+fc=0. ---(I)
Let p be the length of the perpendicular from the origin to
the plane (I). Then
[See § 12]
^ ^ { ( 3 / + ( - 6 / + ( - 2/}
or p— ±*/7.
But according to the given condition p is 3. Hence
*/7 = ± 3 => * = ± 21 .
Patting the values of k in (I), the equations of the required
planes are given by 3x - 6jc- 2z ± .11 =0.
Ex. 5. Find the equation of the plane parallel to the plane
2 x -3 y —52+ 1= 0 and distant 5 units from the point ( —1, 3, I).
Sel. The equation of any plane parallel to the plane
2x—3 y^5z + 1 =-0 is 2 x—2 y — 5z+ * = 0 . ...(I)
L et^ b e the length of the perpendicular from the point
(—1» 3, I) to the plane (I), then [see § 12]
__ , 2 ( - 1 ) - 3 ( 3 ) - 5 ( 1 ) + * , -1 6 + * 7
ic VU 2I H t - 3 / + ( - 5 / ) β ± ν(3»)
60 Analytical Geometry 3-D
a ( - 3 - 1 ) + 6 ( l+ 2 ) + c f - 2 - 2 ) = 0
or —4e+ 3 b—4c= 0, or 4 a -3 6 + 4 c = 0 . .:.(2)
If the plane (I) is perpendicular to the plane x + 2 y —S r=5,
we have a. I + b .2 + c .( - 3)=0, or a + 2 b -3 c = 0 . ...(3)
Solving the equations (2) and (3) for a, b, c, we have
a b c
( - 3 ) ( - 3 ) - 2 .(4) “ 1 x 4 —4 .( - 3 ) = 4 x 2 - 1 .(- 3 )
or y -= jg —-^-=A, (say)· .'· α=λ, 6 = 16A, C=IlA.
Potting the values of a ,b ,c in (I), the equation of the
required plane is
λ [ ( * - 1) + 16 0-+2)+ 11 (2-2)1 = 0,
or x+I6>’+ l l r + 9 = 0 .
Ex. 9. Find the. angle between the planes 2x - y + z = 6
and x + y + 2 z= 7 .
SoK The d,r.’s of the normal to the plane 2 x—y+z**6
are 2, —1, I.
The d r.’s of the normal fo the plane x \ y - \- 2 z —l are I, 1,2.
Now the angle between two planes is equal to the angle
between their normals. Therefore if Θ be the angle between the
given planes, we have
cos β = - 2· + ( - 1 ) .1 + 1.2__________
λ/{(2~)*+'(- i J H (D i)VU υ ’+ιυ'-Η -ο*}
= 3 /6 = J . Λ O=J tt.
Εχ. 10. Find whether the two points (2, 0, I) and (3, —3, 4)
lie on the same side or opposite sides o/ the plane x - 2^+2=6.
Sol. Talcing all terms to the left hand side, the equation of
the plane may be written as x —2,y+z —6 = 0 ..(I)
Substituting the co-ordinates of the point (2, 0, 1 \ the value
of the left band side of the equation (I) of the plane
=-2—0 + 1 - 6 = - 3 .
Again substituting the co-ordinates of the point (3, —3, 4),
the value of the left hand side of the equation (I) of the plane
= 3 + 6 + 4 -6 = 7 .
Since the values —3 and 7 are of opposite signs, the given
two points lie on the opposite sides of the given plane. [See § 11]
.
Ex. 11 Findthe equation of the locus o f a point P whose
distance from theplane bx —2y+ 3z+ 4 = 0 is equal to its distance
from the point ( —1 ,1 ,2 ).
62 Analytical Geometry 3-D
Ex. 3. Find the equation of the plane through the line o f inter-
section o f the planes x + 2 y + 3 z ~ 4 = 0 and 2 .v + j~ z + 5 = 0 and
perpendicular to the plane 5 x + 3 y + 6 z+ 8 = 0 .
(Lockflow 1982, Meerat 84S)
Sol. Theequation of any plane through the line of inter
section of the planes x + 2 y + 3z—4 = 0 and 2x+ y —z + 5 = 0 is
< x+ 2y + 3 z--4)+2 (2 x + v —z+ 5 )--0
or x (1+22) + v (2 + 2 )+ z ( 3 - - 2 ) - ( 4 - 5 2 )- 0 . ..( I )
If the plane (T) is perpendicular to the plane 5jc-f- 3_v+ 6z-f 8=0,
we have (1 + 2 2 ).5 + (2 + 2 ).3 + (3 -2 ).6 = 0
or 5 + 1 0 2 + 6 + 3 2 + 1 8 -6 2 = 0 , or 72 + 29 = 0, or 2 = 29/7.
Substituting the value of 2 in (I), the equation of the required
plane is
■* (I - 58/7)++(2 -29/7)+z (3+ 29/7) - (4+ 145/7)=0
or —51x - 15y + 5 0 z - 173=0,
or 5lx+15y--50z+173=0.
Ex. 4. Find the equation of the plane through the line o f inter
section o f the planes a x+ by+ cz+ d= 0 and ax+ β γ + y z + ^ —O and
perpendicular to the xy-plane.
Sol. The equation of any plane through the line of inter
section of the planes a x+ by+ cz+ d= 0 and ax + fty + y z + 8 = 0 is
(ax+ by+ cz+ d)+ \ (ax+ /Jy+ yz+ 8)= 0 · ...(.I)
or x (a + a 2 )fy (6 +/? 2)+ z (c+y2)+(if+82)=0. - (2)
Now the equation of the .rj'-plane is given by z = 0
or 0 .x + 0 .y + l.r- 0 ; -.(3)
If the planes (2) and (3) are perpendicular to each ■other,
we have
0.(a -f- a2) + 0.(6+ β\) + I / c -f y2) = 0, or 2^ -cl·/.
Putting this value of 2 in the equation ( ! ), the equation of the
required plane is
(ax+ by+ cz+ d )-(c!y) («x+ ftv+ yz+ S )= 0
or γ (ax+ by+ cz+ d)—c («x+ i3>>+yz+8)=0
or (ay—ra) x + (6 y —<*0) y+ (cy cy) z+ (dy—cb)—0
or far—cot) x+ (by—cfi) y+ (d y cS)=0.
Ex. 5. Find the equation o f the plane through the line o f inter
section o f the planes ax+by+cz + d —0 and <χχ+βν+γζ+8—0 and
parallel'to x-axis.
Sol. Theequation of any plane through the line of inter
section of the given planes is
66 Analytical Geometry 3-D
IPct
or cos· * « : =cot B cot C
(W + c V + a V )
Similarly if β and 7 are the angles which the plane (I) makes
with the coordinate planes 7 = 0 and z = 0 respectively, then we
have cos* 3 =cot C cot A and cos* y=cot A cot B.
Ex. 9. Find the equation o f the plane which bisects the Join o f
F (*ι. Λ· *i) and Q (*ι. Λ» Zt) perpendicularly.
Sol. The required plane passes through the middle point of
the segment PQ and is perpendicular to PQ. The co-ordinates of
the middle point of PQ are
(I (*1+*.), i (Λ+Λ)> i (Zi+Z*)·
Also the direction ratios of PQ are (JC1 -X 1, yx—yx, Zx- Z i). Thus
the direction ratios of the normal to the plane are Xx- X i, yi~y*,
zx- Zi and hence the equation of the required plane is given by
( * i - ^ ) {*—i (*ι+*ί)} + ( Λ - Λ ) { y - i 0 ’i+>'*)}
+ (Zi—Zi) {Z—J (Zl + Z*)}=0
or x (x i- x ,) + y (yx -y t)+ z (zx- z t)
-= Η(*Λ- x»*)+iy * - λ *Η-Ui*-·z**)}·
Ex 10. From any point P are drown PM and PN perpendiculars
to zx and xy-planes. O is the origin and «, β, y and S are the angles
which OP makes with the co-ordinate planes and with the plane
OMN. Prove that i f the coordinates of the point P are (a, b, c),
then.
(0 the equation o f the plane OMN is xla—y/b-~zlc=0
abc
(«) S=sin~l V<e*+b*+c*) ν(δν*+<Αι»+<Λ>*)
and (sis) cosec* S=Cosect a+cosec* β+cosec2)'.
Sol. The co-ordinates of the point P are given to be (a, b,c).
The equation of the zx-plane is 7 = 0. Now since PM is drawn
perpendicular from P to the zx- plane, therefore M is the foot of
jthe perpendicular from P {a h c) to the zx-plane and hence the
co-ordinates of the point M are (a, 0, c). Similarly the co-ordin
ates of the foot N (lying in the xy. plane) are (a, b, 0).
Now we shail find the equation of the plane OMN.
The equation of any plane through the origin O (0, 0. 0) is
Ax+ By+ C z= 0. ...(I)
Ifthe plane (I) passes through the points M (a, 0, c) and
N (a, b, 0), we have A.a+B.0+C.c=0 a.nd A.a+ B.b+C.0—0.
Solving these for A, B, C, we get —— = JL = SL .
—be ca ab
The Plane n
aiX+biy'+ctf+dimmO, ..(I)
and V + ^ + v H = 0 · --'-(2)
The equations (I) and (2) should be so written that the cons
tant terms dx and d2 are both positive. However, we can also
write the equations (I) and (2) in such a way that dx and dt are
bofh negative.
If (X1, y l9 Z1) be the coordinates of any point on the plane
bisecting the angle between the given planes, thpn the perpendi
cular distances of this point from both the planes should be equal
numerically. Since we consider perpendicular distance as positive
* when measured in the direction from the origin to the plane, for
points on the plane bisecting the angle in which the origin lies,
the perpendicular distances will have the same sign, and for points
on the other bisector, opposite signs. Therefore if the point
(X1, y X9 Z1) lies on the bisector of the angle in which the origin Iies9
we have
a^ i + bxyx+ C1Z1+ dx α2χ Λ+ h2yx+ c2zt +dz
V W + V + Ci*) ~ V W + W + c f)
Since thisrelation is satisfied by every Point(Vl i ^l i Z1)On
this bisector, the equation of this bisector plane is
Q\X+ bxy + c\z -M 1 a2x + b2y + c 2z+ d2
V W + b i 2 + C i 2) ~ V W + V + C *)1 ' -(3 )
Similarly if (xx, y l9 Z1) lies on the bisector of the other angle
between the two planes, we have
a xx x+ b ± y i + c xz x+ d x O2Xx+ b2y x4 - C2Z1+ d2
VW +bf+cf). " V W + b f +cf)
The equation of this bisector plane will be
axx + b iy + c xz + d x a2x + b 2y + c 2z+ d 2
V W + b f + c f ) ~ ~ V W + b * + c 2*) ...(4)
Hence the equations (3) and (4) are the required equations of
the planes bisecting the angles between the given planes. The
plane (3) bisects the angle in which the origin lies and the plane
(4) bisects the angle in which the origin does not lie. But we
should not forget to write the equations (I) and (2) in such a way
that dx and d2 are of the same sign.
To distinguish between the two bisecting planes as regards the
bisector of the acute or obtuse angle between the given planes.
If we are required to find which of the two bisecting planes
given by (3) and (4) represents the plane bisecting the acute or
obtuse angle between the given planes (I) and (2), we find thfc
The Planes 75
x -2 y -2 z + 9 4 x -3 H -1 2 z + 1 3
V O + 4 + 4 ) ^ V U 6 + 9 + I4 4 )
or 13 ( —jc—2v—2z+9) = 3 (4jc—3 ^ + 12z+13)
or 25jc+ I ly+ 62z —78=0. .(3)
We have pioved above that origin lies in the acute angle bet
ween the planes and so the equation (3) is the equation of the
bisector plane which bisects the acute angle between the given
planes.
The equation of the other bisector plane (/.*., the plane bisec
ting the obtuse angle) is
- jc 2)> 2z+ 9 4jt S y + 12 z + 13
V U +4+4) “ V U 6+*+144)
or JC+ 35y Oz 156—0. .(4 )
.. the equations (3) and (4) give the planes bisecting the
angles between the given planes and the equation (3) is the bisec
tor of the acute angled
Ex. 3 Find the bisector of the acute angle between the planes
2jc—>>+2z+3=0 and 3x - 2 y + 6 z + 8 - 0.
Sol. Proced as above in Ex. I or Ex 2. The required bisector
plane is 23jc—13^+ 32z+ 45= 0.
Ex. 4. Find the equation of the plane that bisects the angle
between the planes 3 jc— 6j >+2z+ 5 = 0 and Ax — I2.y+3z=3 which
contains the origin. Is this the plane that bisects the obtuse angle ?
Sol. Proceed as above in Ex. I or Ex. 2. The required
bisecting plane is 67jc - 162^+47z+44=0, and this is the bisector
of the acute angle.
§ 18. Combined equation of a Pair of planes.
To findthe condition that the general homogeneous equation
of second degree in jc, y and z namely
ajp+byt+cz*+ 2fyz+2gzx+2hxy=*Q.
may reprsent a pair o f planes and to find the angle between them
Also to find the condition o f perpendicularity o f these planes.
The general homogeneous equation of second degree is
ax*+by*+cz*+2fyz+2gzx+2hxy=0 ...(I)
Let the equations of the two planes represented by (I) be
Z1JC+JW1J+W1Z = 0 and ZaJc+/iijjH* JifZ = 0.
78 Analytical Geometry 3-D
These equations will not contain the constant terms for other
wise their product will not be homogeneous. Thus we have
a x *+by*+ C z t A 2fyz+ Igzx+ 2hxy
= (IlXAm tyAn1Z) (ItX+miy+riiz).
Comparing the coefficients of like terms on either side, we
have
IlIt=O, m1mt =b, /I1Zj2=C, /M1Zi2-I-ZniZi1= I / ,!
H1ItA n J 1=Tg, Ifnt A lfn 1=Th J‘ ...(2)
The required condition is obtained by eliminating I1, /M1, M1
and It, mt, nt from the relations (2). This is conveniently done by
considering the following product of two zero-valued determi
nants :
/i k 0 z, k 0
W1 MI2 0 X ZZI4 ZMi 0 =0.
*1 »2 0 ”% ni 0 [Remember]
Multiplying the two determinants by row-by-row multiplica
tion rule, we have
V ik IiftttAliftt1 Iftt A ltFt1
M1I1A m tIi 2ZM1ZM1 Zn1Ms -I-ZMt M1 =0.
H1It A t l t II M1ZMt -I-Mt ZM1 Tn1Ot
On putting the values of I1It, Z1Znt +ZtMi1 etc. from (2), we have
Ta Th 2g a h g
Th Tb 2/ = 0 or h b f
2g 2 / . 2c g f C
or a b c A V g h -a p -h g * eh*=0. .. (3)
This is the required condition that the equation (I) represents
a pair of planes passing through the origin.
To find the angle between the planes. L e tg b e th e angle bet
ween the two planes represented by the equation (I).
Then Θis the angle between the planes I1X A mXyAn1Z=O and
Zt*+ niJy+Mtz= 0 and so is given by
tan 0 = (IM1)It -/MtB1)*]*/*
IikA tttftitAnint ’
where Z1Z1+ /M1ZM1 + M1M2 = a A b + c
and Σ (m1nt —m ft1)*=E [(/M 1MiH-MisM1) * - 4znIzntzi,zit]
The Plane 79
=Σ (AP-Abc)
=A (/* —ftc)+4 (g*—ca)+4 (A*—eft),
so that [^(Ot 1W2- 01,/1, ) ^ = 2 ^ ( /* + £ * + * * -* £ -« * -« * )·
• a _ 2 V iP + g * + h * -l» c -c a -a b )
* * -------------- a + b + c -.(4 )
or , _ ta„ - . [ W + £ ± ! ^ | £ ^ r £ « l
Condition of perpendicularity. The two planes given by (I)
will be perpendicular if 0=£π i.e. tan 0= tan J tt= oo. Hence the
relation (4) gives a + b+c=0.
Thus two planes given by (I) will be perpendicular if
the coefficient of Xa+ the coefficient Of y2+ the coefficient of za=0.
Solved Examples 3 (E)
Ex: I. Prove that the equation x2+4y2 —z2+ 4xy= 0 represents
a pair o f planes and find the angle between them.
Sol. Comparing the given equation with the homogeneous
equation of second degree in X9y , z [See equation (I). § 18], we
have a * I, b = 4, ¢ = - 1 , / = 0 , g = 0 , A==2:
Λ abc+ Ifgh—a /2—bg*—cA8
—1 .4 .( - 1 ) + 2 .0 .0 .2 - 0 - 0 - ( - 1 ) (2)2= - 4 + 4 - 0 .
Hence the given equation represents a pair of planes.
If Θ is the angle between the planes, then
tan f *+g*+A» - be - ca-ab) [See resu,t (4)> § 18]
fl+A+c
2 ^ /(0 + 0 + 4 + 4 + 1 -4 )
putting for a, ft, c etc.
1+ 4 - 1
= W 5. .·. tf=tan-1 (iV5)·
Alternative method. The given equation may be written as
(jc*+4jcj'+4iy *)-zi =0, or (x+2y)t —zt = 0
or (x+ 2y+ z) (x+ 2y—z)=0.
Λ * + 2 ^ + z = 0 , x + 2 y —z=0.
These being linear equations in x, y, z represent two planes.
If 0 is the angle between these planes, then
COS e ______ 1.1+2.24-1.(-1) 4 2
V ( l+ 4 + I ) V ( l + 4 + l ) ~ 6 “ 3
Λ tan 0 = W 5 .
Ex. 2, Prove that the equation
2x*—6ya—12z*+1Syz+ 2zx+ xy= 0
represent^ a pair o f planes and find the angle between them.
80 Analytical Geometry 3-D
>» z Z I
area A on zx and xy-planes, then
& y= A .m (3) and Λ .= Δ.π
Z1 I y\ I
where Δ»=ί xI Zi I . Δ .- i X2 y» I
*3 Zz I *8 y» I
Squaring (I), (3) and (4) and adding, we get
Δ*2+ Δ /2+ Δ*2= A2 (/2+m2+/i*)=A2.l
or Aa= Δ*2+ Δ /2+ Δ·2· ...(5)
This gives area A of the triangle 'ABC.
Solved Examples 3 (F)
Ex. I. Find the area o f the triangle whose vertices arc A(I. 2, 3),
B (2, - I , I) a n d C ( I, 2, - 4 ) . (A ga 1979)
Sol. Let A*> Δν, Λ* be the areas of the projections of the
The Plane 85
y* Zi . i I= i -I I I 21
'2 ’
y a Z9 1 1I 2 -4 I
Zl I I 3 I
h
_I I Zt I =i 2 I I T_
I ** 2
I Xi Z9 I I -4 I
(numerically),
\ * y i I I 2 I
and A = ~ i X2 y2 I =i 2 -I I =0.
Xa I I 2 I
the required area A =V TA a2 *-· A /2+A»*]
441 , 49
-Ji ■0^=-iv/(490) square units.
Ex. 2. Aplanemakes intercepts OA=a, OB—band OC=c
respectively on the co-ordinate axes. Show that the area o f the tri
angle ABC is (Z>zt,2-|- c2a2+ ο2Λ2).
Sol. The points At B and C lie on the axes of x, y and z
respectively, so that their co-ordinates are A{a, 0, 0) B(0, b, 0),
C(0, 0, c).
Let Adenote the area of the triangle ABC. The projection
of triangle ABC on the plane is the triangle OBC and if A*
denotes its area then
A m=\.OB.OC=\bc. . ...(I)
The projection of the triangle ABC on the zx-plane is the
triangle OCA and its area A* <s given by
A r =i-OC.OA=ica.
Also the projection of the triangle ABC on the xy-plane is the
triangle OAB and its area A# is given by
lS t—\.OA.OB==\ab. ...(3)
the area A of the triangle ABC is given by
A*+ A*8+ A**+ A*8= i (b'c'+c'a'+a'b*)
or A = i \/(h 8c8+ c 8a8+a*0*).
Ex. 3 Find the area o f the triangle included between the plane
3x —4_y-f z - 12 and the co-ordinate planes.
86 Analytical Geometry 3-2)
From equations (2) and (3) we observe that the form of equa
tions of the straight line remains unaltered if we use direction
ratios instead of direction cosines.
Corollary. From equations (I) and (3), the general co-ordi
nates of a point on a line are given by
( X i + l r - i . X x + m r , Zi + n r ) or ( x t + a r 9 y i + b r 9 Z 1Jr Cr).
(c) The parametric form.
In the equations (I) and (3), r represents a real number which
changes a the position of the point P o n i h t line changes, so
that r is a parameter. For convenience let this parameter r be
denoted by Ί*. Hence parametric equations of the straight line
are given by
X--X1+ It, y= y i+ m t, z ^ + n t . (4)
or \= X !+ a t, y = y i+ b t, Z=Z1+ ^ ...(5)
where I9m, n and a, b, c are direction cosines and direction ratios
of the line respectively.
Note. If we use.the actual direction cosines then the co
ordinates of a point (on the line) distant r from the given point
(xl9 J 1, Z1) and (xx+ l r 9Ji+ m r, zx+n r). From this point of view
the equations/l) or (2) are also called distance form of the equa
tions of the straight line.
§ 3. Line through two points.
To f i n d th e e q u a tio n s of a s t r a i g h t lin e p a s s in g th r o u g h tw o
p o i n t s w h o s e c o - o r d in a te s a r e (.V1, y l9 Z1) a n d ( jc2> } r2t -2)·
Let A (vx, J 1, Z1) and B (x29yt9 z2) be the given points through
which the line passes. The direction ratios of the line passing
through the points-A and B are X9^ x l9 z2—*1-
Hence the required equations of the straight line are given
by [from equations (3), § 2 (B)]
x *1 _ y -y i Z-Z1
Xr-X1. y * - y * >·(!)
. ' . SOLVED EXAMPLES (A)
Ex I. Find the point in which the line —— —Z~y£
meets the plane x —2y+z=?20.
Sol. The equations of the line are
_ £ l~ 2= r {sav)
3~ 4 12 r y;* ...(I)
The equation of the plane is x —2^+ z= 20. ---(2)
The Straight Line 91
Sol. Note here that we are not required to find the perpen
dicular distance of the point (I, -r-2, 3) from the given plane, but
we are required to evaluate the distance of the point (I, —2, 3)
from the given plane measured parallel to a line whose direction
cosines are proportional to 2, 3, —6. For this we proceed as
follows :
The equations of the line through the point (I, —2, 3) and
parallel to the fine whose direction cosines are proportional to
2, 3, —6 are given by
x - 1 y + 2 _z —3
r (say).
T “ 3 “ —6
The co-ordinates of any point on it are (2 r+ l, 3 r - 2, 6/-+3).
If this point lies on the given plane x —y -\-z= 5, we have
2 r+ l -(3 r- 2 ) + ( - 6/-+3) = 5, or -7 r= —I, or r= l/7 .
\ The point of intersection is ~γ j
.·. The required distance=The distance between the points
(I, - 2, 3) and (9/7, —11/7, 15/7)
I1 Iiil H1 It m. nt
Sol. Let the equations of the required line through the
point (x„ ylt r,) be
x -Χ ι v >i z - z \
I / ' τη ~ ~ n ’ -( 1 )
94 Analytical Geometry 3-D
=V[9/10)a+(3/2)2+(6/5)2J=3V-/2.
Ex. 13. Find the incentre o f the tetrahedron formed by the
planes Jt=Oi v=0, z= 0 and xA -v+ z= a.
Sol. The three planes a*=0, v=0 and z= 0 meet in the
point (0,.0, 0). Hence the incentre of the tetrahedron lies on the
perpendicular from (0, 0, 0) to the plane .v+^rj-z—r?.
The d.r.’s of the normal to the plane a* + > 4 z - a are I, I, I.
Hence the equations of the perpendicular from (0, 0, 0) to the
plane x + y + z = a aie
The Straight Line 97
The d.r.’s of the normal to the plane (I) are 2, —I, I. Hence
the equations of the line through (I, 3, 4) and perpendicular to
the plane (I) are
jc-1 y - 3 2 —4
= r (say).
2 “ -1 ~T~
Any point on (2) is (2 r+ l, —r+ 3 , r+ 4). If this point is the
foot of the perpendicular i.e. the point N, then it will lie on the
plane (I) and we have
2 ( 2 r + l ) - ( —r + 3 ) + (r+ 4 )+ 3 = 0 ,
or 6 r+ 6 = 0 , or r— — I.
Putting this value of r, the co-ordinates of N are (—1,4, 3).
Now as explained above, N is the middle point of PQ. Hence
we have - I . .1+*! Λ_3+Λ 3 4+2i
2 ’ 2
or X i - —3, = 5 , Z1=2·
The image of P (I, 3, 4) is the point Q ( —3, 5, 2).
§ 4. To transform the general form of the equations of a straight
line to symmetrical form.
Let the general form of the equations of the straight line be
given by the equations
« iJ + ^ + C iZ + ^ O l
, eiX+A*y+CjZ+</t =0) .---(I)
Now we are required to write down the symmetrical form of
the straight line given by equations (I). For this we must know
(i) the direction cosines or direction ratios of the line and (ii) the
co-ordinates of a point on the line. To find these two we proceed
as follows :
Step I. To find direction cosines or the direction ratios o f the
line given by equations (I). Let /, m, n be the direction cosines or
direction ratios of the line. Since the line is common to both the
planes and therefore it is perpendicular to the normals of both the
planes. The direction ratios of the normals to the planes given
by equations (I) are au blt ct and at,'b2, c, respectively. Hence
we have
lax+m bi+nc^O and lat+mbt+nct = 0.
Solving these equations for /, m, n, we have
I _____m________ n
b\Cj b^ci C1U1—Cjflj a%bf—ttfii
100 Analytical Geometry 3-D
(I), let us find the point where it meets the plane 2 = 0. Putting
2=0 in the equations given by (I), we have
3x+ 2y—4=0, 4x-f_v+3=0.
Solving these, we get
x v I - -
6-f 4 ~ —16—9 ~ 3 —8’ ° Γ 2 ,y '~S·
Λ The line meets the plane z= 0 in the point (--2, 5. 0).
Therefore the equations of the given line in symmetrical form
are
jc+2 y - 5 z - 0
--3 “ 2 “ - 5 *
Ex. 2. Find the equations to the line through the point (I, 2, 3)
parallel to the line x —y + 2 z —5=0; 3jc+ ^ + z —6=0.
[Jodhpur 1967]
Sol. The equations of the given line in general form are
λ - y + 2 z --5 = 0 , 3 x+ y+ z -^==0. ...(I)
Let /, m, n be the d.c.’s of this line. Then we have
I -m+2n=Q, 3/+ m + /i= 0.
Solving these, we have
I m n I m n
I + ΟΓ ~ 3 ==5~= ~4‘
Since the required line is parallel to the line (I), the d.c.’s of
the required line are proportional to /, m, n i.e. —3, 5, 4.
Hence the equations of required line are given by
( x - 1 ) / -3 = 0 --2 )/5 = (2 -3 )/4 .
Ex. 3. Find the equations. o f the line through the point
(Xi, y lt Zi) and parallel to the line
OiX+hj^+C^+rf!=0, <7ax--f 6tv+c2z+</*—0.
Sol. Let /, m, n be the d.c.’s of the given line, then procee
ding as in § 4, we get
I _____ m n
Vl ClOt —Cj0| ...(I)
The required line passes through (X1, ylt Z1) and its d.c.’s are
proportional to /, m, n given by (I). Hence the equations of the
required line are given by
X-X1 _ y -y t, _ Z -Z 1
A1Ca- A aC1 C1Aj -Cjfl1 O1Aa- U2U1
Ex. 4. Find in symmetrical form the equations o f the line
x= ay+ b, z=cy+ d.
102 Analytical Geometry 3-D
b
Solving (2) and (4), we get
- 3 3 —4 ~ 9 '
Putting these proportionate values of a, b, c in (I), the requi
red equation of the plane is given by
—33 (x—2)+ 4 O '+ 1)+ 9 (z—0)= 0, or 33x+4y—9z -70= 0.
Ex. 5. Find the equation o f the plane through (2, I, 4) perpen
dicular to the line o f intersection o f the planes
3x+4y+Tz+4=Q and x —^ + 2 z + 3 = 0
Sol. Let /, m, n be the d.c.’s of the line of intersection of the
two planes 3 x + 4 y + 7 z+ 4 = 0 , x —^+2z+3*»0.
Then we have 3/+ 4 ot+ 7 b = 0, /- m + 2 n = 0 .
m I m n
Solving,
8+ 7 “ 7 - 6 ” - 3 - 4 ’ ΟΓ 15~ I ~ —T
Thus d.r.’s of the normal to the required plane are 15, I, - 7 .
Alsothe required plane is to pass through the point (2 ,1 ,4 ).
Hence its equation is
1 5 ( * - 2 ) + l O'—1 )-7 (z—4)=0
or 1 5 ^ + ^ -7 2 - 3 = 0 .
Ex. 6. Prove that the lines 3jc+2y+z-5=0«=»x+<y—2z—3
and 2 x - y —z —0 = 7 x + IOjp - 8z—15 are perpendicular.
Sol. Let I19 nii* be the d.c.’s of the first line. Then
3 ^+ 2 //^+ /^= 0 , Ιχ+ηΐι- 2/^=0. Solving, we get
I1 OT1 W1 _ I1 OT1 B1
- 4 - l “ l + 6 — 3 —2 - 5 " 7 “ I '
Again let /„ mt, nt be the d.c.'s of the secon I line, then
2/*—ot»—b*= 0, 7/t + IOot1- 8nt =0.
C l- h OT2 B2 . Z1 Bl1 B1
Solving, 8+ 10- - _ 7 + l6 - 2o + 7 ΟΓ 2 “ 1 “ 3 ’
Hence the d.c.’s of the two given lines are proportional to
—5, 7, I and 2, I, 3. We have
-5 .2 + 7 ,1 + 1.3=0
.'. the given lines are perpendicular.
§6. Tofind the equation o f the plane through a given Une
whose equations are given in (i) general form and (ii) symmetrical
form.
(i) Let the equations of the line in general form be given by
P s a 1x+ b1y+ c1z+ d1=0, Q =a2x+ bty+ ctz+ dt =0: ...(I)
Then P+XQ=Q Le.,
a ^+ b rf+ C iZ + d ^X (atx+ b iy+ c1z+ di)=Q
is an equation of first degree in x, y and z and hence represents a
The Straight Line 107
* β ’—β y '- y =0
I m n
or ( x - a) {η(β’- β ) -m (y '--y ))= 0 .
Ex /. Show that the plane through the point (a, β, γ) and the
liner . -\-q=rz+s is given by
I.χ py+q rz+ s
« ρβ +q ry + s = 0.
I I I (Meerut 1983 S '
Sol. The equations of the given line are
x= py+ q= rz+ s,
x - 0 _y+ (qlp) z+(s,tr)
I VP ~ ' Vr · .,.(i;
The equation of any plane through the line (I) is
e(x 0)+b(y+qlp)+c(z+s!r)=0 (2)
where l.a + ( l/p ) .i+ ( l/r ) .c = 0 . .1)
112 Analytical Geometry 3-D
The plane (2) will also pass through the point (a. 3, γ ) if
aa-f b(fl+q!p)+c(y+slr)=0. ...(4)
The equation of the required plane is obtained by eliminating
the constants a, b, c between the equations (2), (4) and (3) and
hence it is given, by
y-\q\p z+ sir
β + q!p y+ slr
Vp 1/r
Multiplying second and third columns by p and r respectively,
we get the required equation as
p y+ q rz+ s
a ρβ+ q ry+ s 0.
I I I
Ex. 8. Show that the equation of any plane through the line
X-K
I z yH T - zjI T is (* " a) T + (y P) m + (z - y) T = °
where λ+ μ + ν = 0 .
Sol. The equations of the line are
( x - * ) ll= ( y —p)/m=--(z-y)!n. .··(!)
The equation of any plane through the line (I) is
a[x -α )+ δ (ν —fi)+c(z - y ) = 0 —(2)
where al+bm+cn=Q. —(3)
Now choosing the values O=X11I9 6—μ/m, c=v//i,
the equation (2) of the plane is given by
( x - & ) (XIl)+ ( y - β ) (μ!*η)+(ζ—γ ) ( v /ji) — 0 '
where -m + — -n= 0, .o f λ + μ + ν = 0 .
I m n
This proves the required result.
Ex. 9. Fintf the ajurfion o f the plane through the line
ax+ by+ cz—1 ^ x + b 'y + ^ z
and (xx+fiy-ryz - i'.v-f/^v+y'r.
Sol. The equations of the first Mne are
ax+ by+ cz=0-~c x+ b 'y c'z.
This line clearly passes through the orii *. : /. #;.\ n the
d.c.’s of this line then we have
tf/4· Am+c7z = 0 and a7+ft'm +c'/i=0.
The Straight Line 113
Solving these,
I _ m — n
bc'—b'c ca'—da ab’—a'b
Hence the symmetrical form of the first line is
__x___ y z
bc'—b’c ca'—da aV —o b ···(!)
The equations of the second fine are
ax+ + yz= 0 = a'* + β 'γ+ y z .
This line also passes through the origin. Hence its symmctri-
cal form is * — - , y . = .,, Z—,-0 ■ n\
βγ —β y yOL—ya ap —a p
The equation of any plane through the line (I) is
A x+ B y+ C z= 0 ..-(3)
where A(bc'—b'c)+B(ca’—c'a)+C(efr'—a’b)=0. ···(*)
Also if the line (2) lies in the plane (3) then the following two
conditions must hold :
I. The line (2) passes through the point (0, 0, 0) and so the
plane (3) should also pass through (0, 0, 0) and clearly it is true.
II. The normal to the plane (3) should be perpendicular to
the line (2). the condition for which is
Α(βγ'—β'γ) + Β(γα'—y'ot)+ α’β) = 0. ...(5)
The equation of the required plane is obtained by eliminatiog
A, B, C between the equations (3), (4) and (S). Hence the equation
of the required plane is
x y z
bc'—b'c ca'—c’a ab’—a'b =0.
β γ'—β'γ γχ'ι—γ ’χ οφ'—α'β
E*. 10. Find the equation o f the plane through the point
(2, —1, I) and the line 4x—3y + 5 = 0 = y —2z—5.
Sol. The equations of the line are
4*—3 y + 5 = 0 , y —2z—5=»0.
The equation of any plane through the given line [using
Ρ + λ β = 0 ] is 4 x —3y+5 + X (y ~ 2 z —5 )-0 . ---(I)
If the plane (I) passes through the point (2, —I, I), we have
4 ( 2 ) - 3 ( - D +5+A { - 1 - 2 .1 - 5 ) = 0
or 16—8A= 0 or λ = 2.
P uttingthevaIueofA in (I), the equation of the required
plane is
4x—3 y + 5 + 2 (y —2z—$ ) - 0 or 4x—y —4z—5.
114 Analytical Geometry 3-D
Ex. II. Prove that the equation to the two planes inclined at an
angle a to xy-plane and containing the line y —0, z cos β = χ sin β is
(x*+y*) Iant β + ζ* -2 ζ χ tan fi=y* tan* a. (M.U. 1990P)
Sol. The equation of any plane through the line Ot
z cos j8=jc sin β is given by
(x sin β —ζ cos β) + Xy=O9 ...(I)
The other plane is the xy-plane, whose equation is z = 0 i.e.
0.*+<X y+l.z= 0. ...( 2)
The d.r’s of the normal to the plane ( I ) are sin β9 X9 —cos j8,
and the d.r/s of the normal to the plane ( 2> are 0, 0, I. Also the
angle between them is a and hence we have
cos a— s*n 0·Ο+λ 0 —cos βΛ
"V isio* β + > + c o s " β)~ν (θ + 0 + 1 )
or cos a v^sin 1 0+cos 2 β+Χ*)= —cos β
or cos a \/( \ + λ 1)***—cos β
or (λ1 + 1) cos* a=cos* β
or A1 cos* a=cos* /3—cos* a
or A= ± ^(cos* β —cos* a)/cos a.
It gives two values of A and hence there are two planes con
taining the given line and inclined at an angle a to the xy-plane.
Substituting these values o fA in (l) and multiplying both the
equations thus obtained the combined equation of the two required
planes is given by
I * sin f - , cos 0 + v W J > Ti « ! i i L , I
I cos a J
t o a COS* β -COS* a ) I n
J r r cos a I
or / sin
(* - B—z
O cos β)*--------
OsI cos* β5----------
—COS* a v,*= 0Λ
I m n
The equations of the given Iioe in symme: rical IVI Ul WV
x —a y —B z —y . ,
— = i_ r = * _ Z = r (say).
m (i)
The co-ordinates of any point N on the lire (I) are
(e+ /r. β +m r. y+nr). (2)
Let this point ATbe the foot of the perpendicular h i: , the
point P(xi, yi, r,) to the line (I), so that the line PN is perpendi
cular to the line (I).
The direction ratios of the line PN are given by
n + lr—x„ p + m r -y „ y + n r—Zi. ...(3)
Since PN is perpendicular to the line (I), using the condition
O^i1+ - J - C 1Cj =O, we have
I (* + lr—x ,)+ in (fi+m r—y t)+ n (y+ n r—z,)= 0
or r (/*+m *+n*)=/ (jr,—e)+m (yt - f iH n (z,—y)
or r ~ [ / (x ,—a)+m ( γ , —β ϊ+ η (Zi—y )] ’(P + n P + iP ).
(4)
I Analytical Geometry 3-L·
hence these are d.r.’s of the line through (I, —1, 3) and perpendi
cular to the plane (2), and therefore the equations of this perpen
dicular line are
x —l = jM-J z - 3
Ti (say).
Ϊ 2 I
A nypoioton it is (^ + 1 , 2 ^ - 1 ,^ + 3 ) . Let this be the
point Q and so it will Ιϊς on the plane (2).
·'. ^1+1+2 ( 2 ^ - 1 ) + ^ + 3 = 6 , or T1= I .
Putting this value of , the foot of the perpendicular Q is
(5/3,1/3,11/3).
The equations of the projection i.e. the equations of the
line Pgjoining the points P (3, —2, 7) and Q (f, $, V1) at*·
x —3_ + 2 _ z —7 x —3_y~\~2_2—7
3 ^ t“ - 2 - i " 7 ~ V ° r ~ 'W ...(7)
Remark. If the equations (2) and (6) are transformed} to
symmetrical form, we shall get the equations (7) of the projection
PQ- We note that if we want to find the equations of the projec
tion in general form then we use definition one, and if we want
symmetrical form then we use definition two.
Ex. 7. i f L is the line tfte direction
cosines of the projection of L on the plane 2 x + y —3z=4 and the
equation of the plane through L parallel to the line
2x+5^ + 3z=4, x —y -3z=6.
Sol. The equations of the line L are
jc—I —O_z+ 2
r, (say).
2 ^ --TI = I .(D
Any point on the line (I) is ( 2 r + 1, —r, /'-2 ).
If it lies on the given plane 2jc+ j >- 3z=4, ...(2)
we have 2 (2 r+ 1 ) + ( - r)—3 (r- 2 ) = 4 , or 0 .r+ 4 = 0 .
This relation is impossible and we do not get any value of r
as the coefficient *of r is zero. This shows that line (I) is parallel
to the plane (2). Hence method of definition 2 (see Ex. 8) cannot
be used here. Now we shall use method of definition I.
The equation of any plane through the line (I) is
A ( x - 1)+ B y+ C (z+ 2)=0, ...(3)
where 2A—2 H C = 0 . . (4)
The plane (3) will be perpendicular to the plane (2) if
2Λ+2?—3C=0. ..-(5)
The Straight Uae 123
/. W1 Wi ■ 0. ...(A)
It Mt *2
The equation of the plane containing the lines (I) and (2) is
obtained by eliminating A, Bt C between' (3), (4) and (S) and is
given by
I AC-X1 V-Vi Z -Z j
I
/. Ml Wi = o.
I* Mt Wf
Zi-Zt
B
H2 II
Xl-Xt
*
£
h I
IW 1 n x
It IW 2 W2
02 ft,. C2 dt
03 ft. C3 dz
04 ft« C4 dt
Remark. The expansion of a fourth order determinant is
usually difficult and therefore it is convenient to solve numerical
examples by first reducing the equations of the lines to symmetri
cal forms and then proceeding as in case (A).
SOLVED EXAMPLES (F)
Ex. I. Show that the lines £ (jc+ 3 )= J (y + 5 )« —J (z—7) and
^ ( χ + 1 ) = |( ^ + 1)«=* —(z + l) are coplanar. Find the equation o f
the plane containing them. (Rohilkhand 1982; Madras 76)
128 Analytical Geometry 3-D
ut
3r,+ 2= 4ra+3, or
II Il
(4 )
I Ί
and 4/i+3== 5f2+4, or • (5)
Solving (3) and (5), we get T1= - I , r*= —I.
These values of T1 and r%also satisfy the equation (4), and
heqce the given lines (I) and (2) are coplanar. Putting the value·
of Tj (or ofr*), the point of intersection P (or Q) is ( —1, —I, —I).
Now the equation of the plane in which the lines (I) and (2)
lie is given by
x -l y —2 z —3
2 3 4 =0 [See § 8 (A)]
3 4 ‘5
or x-2y+ z= *0.
Ex. 3. Show that the lines
(x + 1 ) / ( - 3 )= (y —3 )/2 = (r+ 2 )/l
and x /l= 0 > -7 )/(—3)=(z+7)/2
intersect. Find the co-ordinates of the point o f intersection and the
equation to the plane containing them.
[Agra 1979, Gorakhpur 82, Kanpur 81, Rohilkhaad 77]
Sol. The equations of the given lines are
^ + 1 ) /( - 3 ) = 0 - - 3 ) / 2 = ^ + 2 ) /1 = * (say) ...(I)
and ( x - 0 ) /l= 0 - - 7 ) /( - 3 ) = ( z + 7 ) /2 = r , (say) ...(2)
The co-ordinates of any point P on.the line (I) are ( —3rj—I,
2 * + 3 , T1- 2) and those of any point Q on the line (2) are (rt(
—3r»+7, 2r*—7).
If the lines (I) and (2) intersect (l.e. are coplanar), then for
some values of T1 and r* the points P and Q coincide. Thus, we
have - S r 1- I = T*, or 3 * + * = —I .. (3)
2tj + 3 = - 3 t,+ 7 or 2 r,+ 3 r,= 4 .. (4)
and * —2=2r*—7, or T1- 2r*= —5 ...(5)
Solving (3) and (4), we get T1= - I , r*= 2.
These values of T1 and r%also satisfy the equation (S), and
hence the given lines (I) and (2) intersect. Putting the value of *
(or of T*), the point of intersection P (or Q) is (2, 1, - 3 ) .
Now the equation of the plane in which the lines (I) and (2)
lie is given by
130 Analytical Geometry 3-D
*+l * -3 z+2
-3 2 I =0
I -3 2
or (X +l) ( 4 + 3 ) - 0 - 3 ) ( - 6 - 1 ) + ( z + 2 ) ( 9 - 2 ) = 0
or * + ^ + 2 = 0.
Ex. 4. In each o f the following cases show that the two given
lines are coplanar :
(0 $ (*—5 ) = i O'—7 ) = —i (2+3); $ (x—8 ) = 0 '- 4)=$ (z -5 ).
Alsoflnd their point o f intersection and the equation of the plane
in which they lie.
(ii) x - J 0 - 2 ) = $ (2+3); $ (x—2)=$ 0 - 6 ) = $ (2-3).
Also find their point of intersection and the equation of the
plane in which they lie. [Indore 1979, Madras 75, 78]
(H i) $ (x -l) = $ 0 - 0 = $ O -i) ; i 0 - 5 ) = J 0 -7 ) = 0 -9 ) -
Also find the equation o f the plane containing them.
[Ranchi 1976]
(/V) ( X - I ) = J 0 - 1 ) = $ 0 - 1 ) ; J 0 - 4 ) = J 0 - 6 ) = J 0 - 7 ) .
Also find their point o f intersection. [Gorakhpur 1978]
Sol. Proceed exactly as in Ex. I (2 or 3) above. The
answers are
(i) (1 ,3 .2 ), 17x—47^-242+ 172= 0.
(ii) (2, 6, 3), x - 2 y + z + 7 = 0 .
(iii) x —2^+ 2= 0. (iv) (2, 5, 7).
Ex. 5. Prove that the lines
(x -a )!a '= O - 6)/6'= O - c)/c’
and (x —a')la= (y—b’)/b= (z—c')lc
intersect and find the co-ordinates o f the point o f intersection and the
equation of the plane in which they lie. [Agra 1982]
Sol. Any point on the first line is P (a'r,+ a, 6 7 i+ 6 , c’ri+c)
and any point on the second line is g (ar»+a', 6r*+6', cr*+c').
The given lines will intersect if for some values of rt and r, the
points P and Q coincide. Thus, we have
a 7 i+ a = a r 1+ a ', i
67i+ 6 = 6 r* + 6 ', I These equations are clearly satis·
and c'ri+c=crt + c’ ) field by r i= r s= l .
Hence the given lines intersect. Putting the value of T1 (or
of r»), the point of intersection P (or Q) is (a + a ', 6 + 6 ', c+c').
The Straight Line 131
Now the equation of the plane in which the given lines lie is
given by
χ —α y —b z —c x y z
2a a OC-f*δ
2/5 β β+Υ
b —a b —a b + c—a—d
a a. α+δ
β β β+Υ
= 0 , the first two columns being identical.
Hence the condition of coplanarity of the two lines is satisfied.
Hence the given lines are coplanar.
Now the equation of the plane in which the given lines lie is
given by
x —a+ d v—a z —a—d
«—δ a α+δ 0.
2a a a +£
2/J β β+Υ
Subtracting two times the second column from the first
column, we get
x + z —2y y -a z —a —d
0 a a -f δ
0 β β+Υ
or x + z —2y=»0 as the required equation.
Ex. 7. Prove that the lines
x__y__ z x y £
α β y' act δβ cy' J m n
N d// lie in one plane i f
(//a) (b-c)+ (m ll3) (c-a)+ (n/y) ( a - b ) = 0.
IGarhwaI 1978; Kanpur 81, 83; Meerut 89]
Sol. We clearly see that the three given lines pass through
the origin O and therefore they will be coplanar if they are
perpendicular to a line through the origin 0.
Lcf it η, ζ be the d. c.’s of this line through the origin 0 .
Hence if this line is perpendicular to the given lines, we have
i* +ηβ + Cy=O, -d )
i βα + φ β + icy = 0, -(2 )
and ξ Ι + ^ + ζ η —Ο. -(3 )
SoIving (I) and (2), we have
i _ v ._____ L _
cj3y—bj3γ any —cay ba-β - ααβ
όΓ ___ (__ _____ n__ ___ __ ζ___
(b—c) βγ (c—a) a y (a —b) <χβ
Putting these proportionate values of i, η, ζ in the equation
(3), we have.
//Jy (b—c ) + m « y ( c — α)+ηαβ (a—f r ) = 0 .
Dividing throughout by αβγ, we get
(//«) (6-c)+(m //3) (c -a )+ (n /y ) (a-b)*~0
as the required condition.
The Straight Line 133
I 1/« II y
[See § 8 (A), condition (A)]
0 β/Λ—b/a S/y—d/c
or 0 1/e—1/« II c - H y »o,
I I/* II y
subtracting the third row from tbe second row
or
(αβ—ba) ( y —c) (cS—dy) (a - a ) n
or ------------------------------ e r a ---------------------------------- = v ,
aoLCy cyacc
or (αβ—bx) (y—c)—(cS—dy) (*—o)=0.
This is the required condition.
Ex. 10. Prove that the lines
3x—5=»4j> —9==»3z and x — I =*2y—4=3z
meet la a point and the equation of the plane in which they lie is
3jc— 8>»+3z+ 1 3 = 0 . (Kashmir 1975)
SoL The equations of the given lines are
x —5/3 y —9/4 z
3x—5·= 4y—9=3z or
i/3 1/4 “ 1/3
x —5/3 y - 9/4 z
or
4 “ 3 ”4 ...(I)
X—I y —2 z
and x —l = 2 j —4=3z or
I 1/2 1/3
x —I y —2 z
or
ϋ ~~3~ = 2 -(2 )
The Straight Line 135
χ ~ ι y - 2 z — = r (say),
'2 3 •- ( 1 )
and 4 .^ -3 ^ + 1 = 0 , 5 * -3 z + 2 = 0 . ...(2)
The co-ordinates of any point on the line (I) are
( 2 r + l,3 r + 2 ,4 /+ 3 ) . .(3)
The lines (I) and (2) will intersect i.e., will be coplanar if
this point also lies on the line (2). This point satisfies both the
equations of the line (2), if we have
4 (2r-(-1) —3 (3r4-2)+1 --0, or r- — I
and 5 ( 2 r + l ) - 3 (4r+3)+2--=0, or r —- I .
Since both the equations give the same value of r, the two
given lines intersect. Putting this value of r in (3), the point of
intersection is( —I, —1, —1).
Ex. 13. Show that the lines J ( x + 4 ) = | (jy+6)=» —$ (z—I)
and 3x—2j>+z+S=0=2jc+3.y+4z—4 are coplanar. A lsofind
their point of intersection and the equation of the plane in which they
lie.
Sol. The equations of the given lines are
x + 4 y+ 6 z—I .
I — - - = - 1 = r(say ) . ( 1)
and 3x—2>»+z+5=0, 2x+3.y+4z—4 » 0 . ...(2)
The equation of any plane through the line (2) is
(3χ—2.y+z+5)+A ( 2 jc+ 3 ^ + 4 z — 4)=0
or _ (3+2A) x + ( —2 + 3A)j>+(»+4A) ζ + (5 -4 λ )= 0 . ...(3)
Now if the plane (3) is parallel to the line (I), then we have
(3+2A) ( 3 ) + ( - 2+3A) (5 ;+ (l + hA) ( - 2 ) = 0 , or A=-£-.
Putting this value of A in the equation (3), the equation of
the plane through the line (2) and parallel to the line (I) is
given by
>0
( 3+ ΐ 3) * + ( - 2+ ϊ ? ) ' + ( 1+ Τ3) ί + ( 5~ ϋ ) ·
or 45jc—17^+25z+53=0. -..(4)
Clearly the line (I) passes through the point ( —4, —6, I) and
it satisfies the equation (4) as
45 ( - 4 ) - 1 7 ( - 6 ) + 25 (1) + 53= 0.
Hencethelines (I) and (2) intersect and they liein the
plane (4).
Ty find the point of intersection.
The co-ordinates of any point on the line (I) are
(3 r-4 , 5 r - 6, - 2 r + l ) . .(5 )
The Straight Line 137
The lines (I) and (2) intersect, if this point also lies on the
line (2) if it satisfies both the equations of the line (2). '
Hence we have
3 (3r—4 ) - 2 (5r—6 )+ (--2 r + 1)+ 5 = 0 , or r = 2
and 2 (jr —’4)+ 3 ( S r - 6) + 4 ( —2 r + 1)—4 = 0 , or r= 2 ;
Since both the equations give the same value of r, the two
given lines intersect. Putting the value of r in (5), the point of
intersection is (2, 4, —3).
Ex. 14. Show that the lines
T x - A y + Hz+ 16 = 0 = 4 x + 3 j—2z+3
and x - 3 y + 4 z + 6 —0 ^ x —y + z + 1
are coplanar. (Gorakhpur 1975)
Sol. If the given lines are to be coplanar» then we must have
7 - 4 7 16
4 -2 3 = 0. [See § 8 (C)]
I -3 4 6
I - I I I
Applying Rl - T R i . R2-A R iy R9- R iy the determinant on the
left hand side
0 3 0 • 9
0 7 -6 -I
0 -2 3 5
i
I —1 I I I
3 0 . 9
7 -6 -2 2 * by C9- I C 1
-2 3 11
= - 3 {—66—( —66)} = —3 (-66+ 66)= -0.
Hence the given lines are coplanar.
Ex. 15. Show that the lines x + y + z —3=0=»2x+3y+4z—5
138 Analytical Geometry 3-D
Γι + 4—13r2+(16/9), -*-(5)
- 2 r ,- l- 7 r ,+ ( l/9 ) ,. ...(6)
and Ti = - 9r*. ...(7)
Solving (5) and (7), we get r,=> —10/11, r*= 10/99.
The values of r, and r2 also satisfy the equation (6) and
hence the given lines are coplanar. Putting the value of r2 (or of
r2) the point of intersection P (or Q) is ( —9/11, —8/11, —7/11).
Now the equation of the plane in which the lines (3) and (4)
[t.e. (I) and (2)] lie is given by
x —4 y+ l z
I I =0 [See §8 (A)]
13 7 -9
or ( x - 4 ) ( - 2 ) ( - 9 ) -7.1}- (y+1) { 1 .(-9 )-1 (13)}
+ z { ! - 7 - ( - 2 ) (13)}=0
or (JC-4). I l+ 0 > + l) .2 2 + z .(33)=0 or x + 2 ^ + 3 z = 2 .
Ex. 16. A, A'; B, B'; C, C are points on the axes. Show that
the lines of intersection of the planes A'BC, ABC'; B'CA. BCA
C'AB, CA'B' are coplanar.
Sol. Let the co-ordinates of the points A and A' be (a, 0, 0)
and (a', 0, 0); B and B' be (0, b, 0) and (0, b', 0); C and C be
(0, 0. c) and (0, 0, c‘).
The equations (in the intercepts form) of the planes A'BC
and AB'C are respectively given by
* +* + is I and * + £ + * . . 1 .
a' b c a b c
Therefore, the equation of any plane through the line of inter
section of these two planes is given by
( ^ +τ - ' Κ +Κ - - Ή
or
...(2)
The symmetry in the equation (2) shows that the lines of
140 AnalyticaI Geometry 3-D
intersection of the other two pairs of planes also lie in the plane
given by (2).
Hence the lines of intersection of the given three pairs of
planes are coplanar.
Ex. 17. Find the equation of the plane through the line
x/l<=ylm = zIn
and perpendicular to the plane containing the lines
x\m —yln*=tzH and x/n—y ll—zlm.
I Lucknow 1982, Ranchi 79, Punjab 77, Agra 75, M.U. 89(S)]
Sol. The equation of any plane through the line
x ll—ylm =z/n is
A x+ B y+ C z= 0, ···(!)
where Al+ Bm + Cn=O. ·*·(2)
Also the equation of the plane through the lines
x/ffi=ji/n=>z// arid jr/n=^//=-z/m
(note that both of these lines are passing through the origin) is
X y Z
C
(/—m) (In+ mn+ lm)
A _ B _ C
m —n n—l l —m
Patting these proportionate values of At B, C in (I), the equa
tion of the required plane is given by
(m—n)+ (n—l) y + (l—m) ζ » 0 .
Ex. 18. Find the foot and hence the length o f the perpendicular
from the point (5, 7, 3) to the line
( x - 15)/3 - {y—29)/8=I z - 5 ) /( - 5).
Find the equations of the perpendicular. Also find the equation o f the
plane In which the perpendicular and the given straight line lie.
Sol. Let the given point (5, 7, 3) be P.
The equations of the given line are
( x - 1 5 )/3 = 0 > -2 9 )/8 = (z -5 )/(-5 )= r (say). ...(I)
Let N be the foot of the perpendicular from the point P to
the line (I). The co-ordinates of N may be taken as
(3r+15, 8r+29, —5r-f-5). ·*·(2)
A the direction ratios of the perpendicular PN are
3 r+ t5 -5 , 8 r+ 2 9 -7 , - 5 r + 5 - 3 ,
l.e. are 3r+IO, 8r+22, - 5 r + 2 . ...(3)
Since the line (I) and the line PN ute perpendicular to each
other, therefore 3 (3r+ 10)+ 8 (8r+22)—5 ( —5r+2)«=0
or 9 8 r+ 196=0 or r —- —2.
Putting this Value of r in (2) and (3), the foot of the perpen
dicular N is (9, 13, IS) and the direction ratios of the perpendicu
lar PN are 4, 6, 12 or 2, 3, 6.
A the equations of the perpendicular PN are
(* -5 ) /2 = ( y —7)/3= (z —3)/6. ...(4)
Length of the perpendicular PN
=»the distance between P(5, 7,.3) and N{9, 13, 15)
W { (9 -5 )« -K 1 3 -7 )* + (1 5 -3 )* } = 1 4 .
Lastly the equation of the plane containing the given line (I)
and the perpendicular (4) is given by
x —15 .v—29 z—5
3 ‘
OO
_ 5 =0 [See § 8 (A)]
2 3 6
142 Analytical Geometry 3-D
JCi 7i Zl
/i IW1 "l
Adding the second row to the first row, we get
y Z
i x
Jl Zl >0 ...( 4 )
i *
! /i mx *1
or (ntPi—Oi1Z1) Jf-K /A —« A )y + (« » A —ZiTk) z= 0 . ...(4')
The Straight Line 147
Similarly the plane through the origin and through the second
line is
X y Z
y* Zl
-.(5)
I2 m 2 M2
or (/¾V2-TntZj χ + ( / 2ζ 2- μ2χ , ) y + (OttXt - I t f t) z = ° C5')
The planes (4) [or (4')] and 5 [or (5') together give the required
line.
Ex 9. Find the equations to the straight line drawn through
the origin which will intersect both the lines
I y+3 z - 5 x —4 y + 3 z —14
and
I “ 4 “ 3 2 " 3 4
Sol. The equation of any plane through the first line is
A (x - 1)+21 (y + 3 )+ C (z—5)=0, ...CD
where 1..4+4.a+3.C =0. ...(2)
Tf the plane (I) passes through (0, 0, 0), we have
—A + 3B —5C=0. ...(3)
Solving (2) and (3). we get .4 /2 9 = .8 /(-2 )= 0 /(-7 ).
.Substituting these proportionate values of A, B, C in (I), the
equation of the plane through the origin and the first line is
29x—2y—7z«0. -C 4)
Similarlythe equation of the plane through the· origin (Md
the second line is
9x —2y—3z=0. ·· C5)
The planes (4) and (5) together give the required line.
Ex. 10. Find the equations o f the line through· the point
(3. I, 2) parallel to the plane 4;c+y+5z=0 so as to cut the line
x + 3 = y + 1= 2 (z—2). Find alsq the point of intersection.
Sol. Let the point P be (3, 1, 2).
The equation of the given plane is 4 x + y + 5 z= 0 . .. (I)
The equations of the given line may be written as
(x + 3 )/2 = ( y + l) /2 = ( z - 2 )/l= r (say). ...(2)
Let the line through P (3, I, 2) parallel to the plane (I) cut
the line (2) in the point Q.
The point Q may be taken as (2 r-3 , 2r—I, r+2).
.·. the d.r.’s of PQ are 2 r - 6 , 2r~2, r. •►■(3)
148 Analytical Geometry 3-D
-(5)
Note. In the equations (I) of the line AB, I, m, n have been
taken as the actual direction cosines of the line. In case direction
ratios a, b, c of AB are given, we should either first find the direc
tions cosines of AB or we should divide the R.H.S. of (5) by
(a2+ 62+ c*).
To find the co-ordinates of the foot of the perpendicular N.
Since N, the foot of the perpendicular, is a point on the
line AB given by (I), its co-ordinates may be written as
(Ir+<x, mr+ β, nr+γ). -(6 )
The d.r.’s of PN are Ir+ a - X1, mr+ β —yu nr+ γ —Z1.
Also PN is perpendicular to AB.
150 Analytical Geometry 3-2)
-J S -o r^ N M P ? ·
Putting the value of r in (3), the d.r.’s of PN are 7/25, 1/25,
—5/25 i.e. are 7, I, - 5 .
*. the equations to PN i.e. of a line passing through
P (I, 2, 3) and having d.r.’s 7, I, —5 are
x - l y —2 z - 3
. 7 - 1 --5 ·
Ex. 2. How for is the point (4 ,1 ,1 ) from the line of inter
section o f x + y + z —4 = 0 = * —2y—z —4 ?
Sol. Let the point (4, I, I) be taken as P.
The equations of the given line .42) (say) are
(x—4 )+ y + z = 0 , (x—4)—2y—z=0.
Solving for x —4, y, z, we get
x —4 y Z- x —4 y z
- 1 + 2 - 1+ 1“ —2 —I ·°Γ I — 2 “ —3* -(I)
The Stntigkt Line ISl
or A 1= I Uft+ Λ Ρ + ( - X i - ΖιΡ+(Ρι—* ιΗ
= I (Xia+ V + ft* + A ft+ ftX i-X ift) -(¾
Let be the perpendicular distance of P (Xb yu Zj) from the
plane x p + z = I. Then
„ X i-f t + z .- l
p* ν α + Η » ) ' ·<3>
Aeuxding to the given problem, P1-=Tpt.
Squaring, Pi1= 4A*
or I (ft1+P11+Zi, + P ift+ z1x1- x 1p1) = i ( X i- P i+ f t - 1J*
or Xi*+Jie+Z i*+7A +ftX i -X iPi=2 (ft*+Jft*+*i*+l —2ftPi
+ 2 ftft~ 2 * i—2 ftft+ 2 fi—iz,)
or Xia4-Pis4-Zi1-SpiZ i+ ? z A -3 x iP i-4f t + 4P i - 4A + 2=i0·
the locus bf P (x„ p», Z1) is given by
jZ+p^+z1—5pz+3zx - 3xp - 4x+4p—4z+ 2 = 0 .
K r 5. Pied the Ioaa o f a point whose distmcr from x-axis is
twice its distance farm the yz-plaee. (Agra M l)
SaL Let the given point be P (xlt Jb z,) whose locus is
required to be found.
The equations of x-axis are
(x —0)/1 = (p —0)/0= (z—0)/0, —(I)
<*«»«· the x-axis passes through (0 ,0 ,0 ) and has dx.’s 1 ,0 ,0 .
Ifp 1 be the perpendicular distance of P(XfePfeZ1) from (I),
then
I I 0
A 1=
Pi , Z1 Z1 X1
or A1= [-Zi)*+(Pi)* or Pi1=P1*+Z1*. ...(¾
Let P t be the perpendicular distance of P iiX1, Jf1, f t ) from the
pz-plane. Then p,=x-co-ordinale of P=X1. ...(3)
According to the given problem, we have
Pi=Tpt or p f= 4 p t*
ο*- P i^ ft 1= 4X1*, [using (2) and (3)]
Λ the locus of the point Pix11pto Z1) is 4x* -ρ*--ζ*=0.
Ex. L Fied the length o f the perpendicular drawnfrom origin
to the Uee x+ 2p+ 3z+ 4= 0«= 2x+ 3p + 4z+ 5. Also find the equa
tions o f this perpendicular and the co-ordinates o f thefoot o f the
perpendicular. (Meant D M S, «4 R)
SuL The equations of the given line AB (say) are
x + 2 p + 3 r+ 4 = 0 = 2 x + 3 p + 4 z+ 5 .
The symmetrical form of the above given line is
The Straight Line 153
Ex. 7. Find the equations o f the two planes through the origin
which are parallel to the line (x - 1)/2 = Cv+- 3)/( —I) = ( z + 1)/(—2)
and distant 5/3 from it. (Meerut 1976, 89, 89 S)
Sol. Let the equation of any plane through the origin
be
ajt+Ay+cz=0. ...(I)
The equations of the given line arc
J t - 1 p+3 z + 1
2 ” -I * -2 ’ ...(2)
(f the plane (I) is parallel to the line (2), then the normal to
il) whose d.r.’s are a, b, c will be perpendicular to the line (2) and
hence we have
2.a—l.b —2.c=0 or 2a—b—2c=0. ...(3)
According to the given problem, the plane (I) is at a distance
5/3 from the line (2).
Hence the distance of (I, - 3 , - I), a point on the line (2),
from the plane ( I) = 5/3
154 Analytical Geometry 3-D
g .l+ 6 .( - 3 ) + c .( - l) 5
V(a*+b*+c*\ ” 3*
On squaring. 9 (a —3 6 -c )2=25 (a2+ 6 2+ c2)
or 9 (a2+962+ c2 -6a6 —2ac+66c)=25 (a2+62+ c 2)
or 16a2—56b2+ 16c2+ 54ab + 18ac —56be= 0
or Sa2—2862+ 8c2+ Ila b + 9ac —286c=0. ...(4)
Putting the value of 6= 2 (a - c) from (3) in (4), we get
8a2—28 x 4 (a -c )2+Sc2+27a.2.(a-c)+9ac-2bc.2 ( a -c )= 0
or —50a2+ 125ac —SOc2=O or 2a2 -5tfc+2c2= 0
or (a —2c) (2a—c)= 0 or a=2c, Jc.
For a—2c, from (3). 6 = 2c
and for a = £c, from (3), 6=a —c.
Putting a=2c, 6=2c in (I), the equation of one plane is
2 cjc+ 2 c v + c z = 0 or 2x+ 2j’+ z = 0 . -.-(5)
Again putting a= Jc, 6 = - c in (I), the equation of the second
plane is \c x —cy+ cz= 0 or x —2y+2z=0. ...(6)
The equations (5) and (6) are the equations· of the required
planes.
§11. Intersection of three planes. (M.U. 1990)
Let the equations of three planes be given by
U1= O 1X + b Ly + C1Z + J 1 = O 9 ...(I)
U 1= O 2X + b 2y + c tz + d %= 0 , ...(2)
and ih = a t x +b ty + c tz + d t = 0 9 ...(3)
where M1, U 1 and M3 denote respectively the left hand members in
the equations (I), (2) and (3) No two of these three planes are
parallel.
We know that two non-parallel planes intersect in a straight
line and hence we get three lines of intersection by taking two
planes at a time out of the three planes given by (I), (2) and (3).
There arise following three cases :
Case I. The three lines of intersection explained above may
coincide i e. the three given planes have a common line of inter
section.
Case IL The three lines of intersection explained above may
be parallel to each other and no two of them coincide. In this
case the three given planes form a triangular prism.
Case III. The three lines of intersection explained above may
intersect in a common point. In this case the three planes inter
sect in a point.
The Straight Line 155
Uj b, U2 dt
Let the determinant obtained by omitting the first column in
(i) be denoted by Λ ι i-c. we put
h Cl Cfl
A 1= bt C2 Cft
bt C2 O3
Similarly, the determinants obtained by omitting the second,
third and fourth columns will be denoted respectively by Δ*. Δ«
and Δ*· Thus we put
Ui Cl di I Ol bi di
Δ 4= U2 bt ct •
Uj b3 C3
The symmetrical form of the line of intersection of the planes
(I) and (2) is [See § 4]
. I bjdt M i \ ,, ( CtiQf-dtfli\
\α φ 2- α φ ι } y \ Uib2 -UjblJ____ z —0
A1Ca—Aa^1 ““a A -A 2A1* ...(4)
where αφ%—αφ&£ιΟ.
Now we shall discuss the three cases given above in detail as
follows :
Case I. The three planes intersect in a common line.
The equation of any plane through the line of intersection of
the planes (I) and (2) is given by
W1+ An2=O
or (OiX+biy+CiZ+di) + λ (atx +b 2y d t)= O
or (ai+λα*) x +(6 i +A62) ^+(C1+Ac2) 2 + (^ ^ 4 )= 0 ....(5 )
if the three given planes intersect in a common line, then for
156 Analytical Geometry 3-D
some value of λ the plane (5) should represent the plane (3). Thus
comparing the coefficients in the equations (5) and (3), we have
0i+Atfa^ 6i+A6a C1 - l A c 2 dx+Xd2_ . *
az C3 d%
Λ tfi+Atf2 /Xtf3= 0,
Ci+A fy-f^*=0,
C i+ A c s — /xca«=»0,
and —^itZj=O.
Now we are to eliminate two arbitrary constants λ and μ and
this can be done from any three out of the four equations given
above. Hence eliminating A and μ from any three equations taken
at a time out of these four equations, we have the conditions as
01 bi Cl
aI b* Cj = 0 Le. A 4=O
03 h Cj
01 h 4
*<■,
A j= O
O
h
I
0a Cli
03 bz <1,i
01 Cl di
0a Ca dt = 0 i.e A j- O
az <t di
bi Cl dx
d* =0 i.e.
Il
V
>
P
C2
b* C3 d,
Hence the three planes will have a common line of intersec
tion of Δ λ—Ο, Δ ι= 0 , Δ 2= ° 3η(1 Δ i —O.
Alternative method. If the three given planes (I), (2) and (3)
intersect in a common line then the line (4) [the lineof intersection
of the planes (I) and (2)] will lie in the plane (3), so that we have
[by § 5 (iii)]
03 ( V i - V i ) + * a (cI^a“ C gtfi)+(tfA -02*«)=O
The Straight Litre 157
«1 bi Cl
i.e., A* bt Ct =O i.e., A«=0
a. bt c,
and
Le., Ot (Aid2- A,r/,)+ A3 ( d ^ - d jA ^ + d , (O1A1-A 2A1) - 0
fli bt
K .di
i.e., Oi bt . i/2 = 0 i.e., Aa=O.
d2
O3 N dt
[Note, While writing the symmetrical form of the equations
(4) of the line of intersection of the planes (I) and (2), we have
taken the point on the line for which z —0. If we take the point
for which x = 0 the condition Ai=O is obtained instead of Aa=O.
Similarly by taking the point for which _y=0, the condition Aa“ 0
will be obtained.]
Hence the planes (I), (2) and (3) will intersect in a common
line if Aa=O, Δ 3 = 0 , A t - O and Ai=O.
Case II. The three planes form a triangular prism.
The three planes will form a triangular prism if the line of
intersection of any two pianes is parallel to the third plane and
does not lie in it.
The line of intersection of the planes (I) and (2) is given by
(4). The line (4) will be parallel to the plane (3) if
O3 (AiC2- A 2C1) f A., (V1A2 - C 2A1) + ^, (Atftl - A 2Al ) = O
I O1 A1 Cl 1
! “3 c3 1
The line ( I ) will not lie in the p la n e (3 ) if
*(¾ ¾ )
i.e., a3 (AxrZ2-A jd1)+A3 (tlxat—dMy) t-d3 (AlAt- A 2Al) +0
Ar bT di I
i.e., CU bt d3 j^O i.e., AtT^O.
Aj b3. d.
158 Analytical Geometry 3-D
b% c2 d t a, c2 d t
h C3 d z flj Ca d a
Z -I
*1 I 1 " ~ d i Cfl b i Cl
a 2 h d t *2 b t C2
*a b* < k. Qz \ ' c3
X —y Z — 1
or
I 00
\<
II
l< i
A a ~ Δ «
L Δ »
or X = - - z = -
Ni
Il
, ,
Δ / . ( 6)
Hence the three planes will intersect in the point whose co
ordinates are given by (6) if Δ 4=5^0.
Working Rule. Let the three planes be given by the equa
tions (I), (2) and (3). Now proceed as follows :
(1) First evaluate A 4- If A 4T^O, then the three planes
intersect in a point whose co-ordinates are given by the relations
(6) above.
(2) If A 4=O, then evaluate Aa, A t and Ai-
(i) If A 3^0 (or A it^O or Aa^O), then the three plants
form a triangular prism.
(ii) If Ae = O, Aa=O and A i= O , then the three planes inter
sect in a common line.
The Straight Line 159
-( 2 )
N>
II
I
O
N
I
X
2x+8_y—2z—1=0 -(3 )
The rectangular array of coefficients is
I 5 2 - 4 2 j
4 -2 -5 —2 .
I 2 8 -2 -I !
We have,
5 2 -4 I 2 -4
Δ «= 4 -2 -5 -I -2 -5
2 8 -2 0 8 -2
on adding the 3rd column to the 1st column
I 2 -4 , on adding the first row to the
second row
= 0 0 -9
0 8 -2 I
=72^0.
Hence the given planes intersect in a point.
Adding (I) and (2), we get
9x—9z=0 or x=z.
Putting x = z in (3), we get
8y—1=0 or y = 1/8.
160 Analytical Geometry 3-D
2 4 2
We have, Δ 4= 5 I -I
I -I
T1
0 6 4 by Rt - SRat
R \—2Ra
I 6 4
I -I -I
= 24 -2 4 = 0 .
Since A«=0. therefore the three planes either intersect in a
line or form a triangular prism.
2 4 -7 2 6 5
Now A s= 5 I -9 = 5 6 21 ,
I -I -6 II 0 0
j by C*4 Cx, Ca+6Ci
= 1.(126-30)=96=^0.
Hence the three planes form a triangular prism.
Ex. 2. Show that the planes
I x - S y - I z = Q t S x - H y ^ lS z = O t S x -S ly -S S z = O
pass through one line and find its equations. (Meerut 1977]
Sol. The rectangular array of coefficients is
2 - 3 -7 0
3 -1 4 -1 3 0 •
8 -3 1 -3 3 0
2 -3 -7
A t= 3 -1 4 -1 3
8 -31 -3 3
2 -I -I , by C ,+C 1,
C,+3Ci
3 -11 -4
8 T 23 -9
162 Analytical Geometry 2-D
0 0 -I . by Ci+2Ct,
Ct —Ct
= -5 -7 -4
-1 0 -1 4 —9
= -1 ( 7 0 - 7 0 ) = 0 .
Since A«=0, therefore, the three planes either intersect in a
line or form a triangular prism.
2 —3 0
Now At* 3 —14 0 = 0.
8 -3 1 0
Similarly A i =O and At=O.
Hence the three planes intersect in a common line.
Clearly the three planes pass through (0, 0, 0) and hence the
common line of intersection will pass through (0, 0, 0).'' The
equations of the common line are given by any of the two given
planes. Therefore the equations of the common line are given by
Ix - S y - T z = O ,
and S x - I4y— 13z=0.
Λ the symmetrical form of the line is given by
x y z x y z
39—9 8 ~ —2 l+ 2 6 = —28+9 ° r - 5 9 “ 5 _ - 1 9 ’
Ex. 3. Prove that the planes
x—2y+ z—3=0, x + y —2r—3 —0, x—z—1=0
form a triangular prism.
Sol. Proceeding as in Ex. I above, we get A t-O an d Ar^O-
Hence the’given planes form a triangular prism.
Ex. 4. Prove that the planes
x=cy-\-bz, y= az+ cx, z= bx+ ay
pass through one line i f a*+b* +ca+2aftc= I, and show that the line
o f intersection then has the equations
x y z
V (I-A f) " V U -C 2) ’
(Bundelkband 1978; Punjab 77; Rajasthan 75, 77]
Sol. The equations of three given planes are
x - c y - b z = 0, c x -y + a z = 0 , bx+ ay—z —0.
The Straight Line 163
b a -I o
I —C -b
We have, A«= C -I a
b a I
»1 (I —at)+c ( —c -ab) —b (ac+b)
Ot -H t - Ct -
I —C 0
Also A s= C -I 0 •0.
b a 0
Similarly As=O and Ax=O.
Hence the given planes intersect in a line if A *=0 i.e., if
a%+ IP+c%+Zabc= I.
Clearly the given planes pass through (0, 0, 0) and hence the
common line of intersection will pass through (0, 0,0). Let /, m,
n be the d.r.’s of this line. It being perpendicular to the normal
of each plane, we have
I - cm—bn= 0 ...(I)
cl—m +an=0 ...(Z)
bl+am—n = 0. .-..(3)
Solving ( I) and (2), = -JL jr
or - J _____ ” ____ ! _
ac+b bc+a l~c* ...(4)
Solving (2) and (3), J L _ _ !L . (J)
m* n*
1—6*= 1—e*’ ...(8)
Now from (7) and (8), we get
/* m2 n*
1—a* =1 -Z>i = l - c 2'
. I m n
V O -a 2)_ VO-**] _ V (i-c 2) -(9)
Λ the d.r.’s of the common line of intersection of the given
planes are given by (9). Since the line passes through the origin,
hence, its equations are given by
x y z
VO - a %) ~ V 0 ~ * 2) = V 0 -c2)’
Ex. 5. Prove that the planes
x+ ay+ {b+ c) z+ d = 0,
, x+ by+ (c+a) z+ d = 0,
x+ cy+ {a+ b) z-M =0,
pass through one line. [Allahabad 1978, Kanpur 82]
Sol. The rectangular array (or matrix) is
I Cl b+c d
I b c+a d
I C a+b d
I a b+c
We have, Δ 4— I b c+a
I c a+b
I a+ b+ c b+c adding 3rd
column to 2nd
I a+ b+ c c+a
I a+ b+ c a+b
I I b+c
- ( a+ b+ c) I I c+a =0
I I a+b
I a ar I a I
Also Δ *= I b a I =M I b I
I c d I c I
<■,\s "■
The Straight Line 165
-0 -n (0 -
.E
(» /-0 )
E
I
= 0 —n (wv+m/i)—λ (/i/-0)
= —n (IX+mp+nv).
If the planes are to meet in a line then Δβ must also be zero.
Then we have n (lX + m p+ nv)= 0
or lX+mp + ny = 0. ...( 4 )
[We have assumed /?^£0.]
If the condition (4) is satisfied, then we can see that Δ -"
166 Analytical Geometry 3-D
and also Δ ι* 0 . Hence (4) is the condition for the three given
planes to have a common line of intersection.
Let a, b, c be the d.r.’s of the line of intersection, It being
perpendiculer to the normal of each plane, we have
0 a+ nb—mc= 0 -(5 )
—n e+ 0 .6 + /c= 0 - ( 6)
ma—lb+0.c=0. -(7 )
Solving (5) and (6),
a___ 6 __ £_ JL — £_
nl ~ m n ~ n * I “m“ u’
showing that the d.r.’s of the line of intersection are I, m, n.
To find the equations of the line of intersection. The line of
intersection meets the plane z = 0 in the point given by
n y —X =0, - η χ -μ β Ο , [Putting z= 0 in (I) and (2)]
or X= —μ/η, y=X/n.
Λ the line of intersection passes through the point
. ( —μ/η, X/n, 0) and has d.r.’s I, m, n and therefore its equations are
given by
χ+ μ/η y - X /n ^ z - 0
I ~ m ~ n •••(8)
The distance of the fine (8) from the origin, Using § 10, the
required distance p is given by
» * = _ _ i _ Ji m » r
* (P + n f+ n %
) 110 —A/« 0 —0 I
n I I* I I m
+ 0 -0 0-X/n
I
(/·+«.■+«»)
I
[using (4)]
Λ p={(Aa+ ^ 2+v‘)/(/2+ m 2+ n 2)}1'a.
Ex. 7. Prove that the planes x = y sin φ+ζ sin φ, y = z sin Θ
+ x sin φ, and z —x sin φ-by sinti witl intersect in the line
x ___ y_ z
cos 9~cos φ"
[Allahabad 1982; Garhwal 78 S]
Sol. The equations of the planes may be written as
x —y sin φ—z sin ¢= 0, ...(I)
x sin φ—y+ z sin ¢=0, ...(2)
x sin φ+ y sin Θ—z=0. ...(3)
The Straight Line 167
Let us find the line-of intersection of the planes (I) and (2).
Let I, m, n be the direction cosines of the line of intersection of
the planes (I) and (2). It being perpendicular to the normals of
both the planes, we have
l—m sin φ—η sin I sin φ -m + w sin ¢=0.
Solving, we get
_______ I_________________ m________ n
—sin φ sin 9—sin φ ~ —sin φ sin φ - sin Θ = —I +sin* ψ
_______ I_______________ m____________w
sin ^ sin ¢ + sin φ '“ sin φ sin ^ + S in i- I-Sin* φ * ...(4)
If 9 4-^+ ^= $π, then we have
sin 0=sin ($π—(φ+φ)}=ξθ& (φ+φ)
=cos φ cos φ—sin φ sin φ
or sin 0-f-sin ^ s:n 0=cos ^ cos ¢. ...( 5)
Similarly sin ^-(-sin φ sin 0=cos φ cos Θ. -"(6)
Using the relations (5) and (6), (4) becomes,
I m______n
cos φ cos 9 ~cos φ cos φ “ cos* φ
ΟΓ
I m n
--------- — --------- C a ---------
COS 0 COS^ COS φ ·*·(7)
Clearlytheplanes (I) and (2) passthrough (0 ,0 ,0 ) and so
their line of intersection will pass through (0, 0, 0) so that its
equations are given by
x/cos 0= yjcos φ= ζ/cos φ. ...(8)
N ow it remains to prove that the line (8) must lie on the
plane (3).
The point (0, 0, 0) through which the line (8} passes also lies
on the plane (3). Also the normal to the plane (3) whoss d.r.’s
are sin φ, sin Θ, —1 must be perpendicular to (8), the condition
for which is
cos 9 sin ^-J-cos φ sin 0 + co s^ ( - 1 ) = 0
or sin (0 + ^ )—cos ^ = 0 , or sin (|π - φ)—cos φ = 0
or cos φ —cos φ—0 or 0 = 0 which is true.
Hence the line (8) also lies on the plane (3). Thus the equa
tions (8) are the equations of the required line.
Ex. 8. Show that the planes ax+ hy+ gz—0, h x + b y + fz -0 ,
gx+ fy+ cz= 0 have a common line of intersection if
168 Analytical Geometry 3-D
a h g
Δ= h b f
g f C
and the direction ratios of the line satisfy the equations
_m * _ n*
« Δ '_3 Δ ~ 0 Δ ·
da db Sc
Sol. The equations of the given planes are
ax+ hy+ gz= 0 ...(I), hx+ by+ fz= 0 ...(2)
g x+ fy+ cz= 0. ...(3)
The rectangular array of coefficients is
a h g 0
h b f 0
g f C 0
We have
a h g =A (as given in the problem)
Δ*= h b f
I- g f C
or A i= &=abc+2fgh—af*—bg*-chK (4)
Also
a h 0 = 0 . Similarly Aa=O and Δ ι= ° ·
Δ ·“ h b 0
g f 0
If the given planes intersect in a line then A i must be zero
(as Δ». Δ 2 and Ai are already zero). Hence the given planes will
have a common line of intersection if A i=O or A=O
or abcA-Ifgh a p —bg2—Chi -O . -- (5)
Let /, m, n be the d.r.’s of the common line of intersection of
the given planes. It being perpendicular to the normals of the
planes, we have
al+ hm +gn—0, hl+ bm +fn=0, gl-{-fm+cn—0.
Solving any two (say the first two) of these relations, we get
I m n
The Straight Line 169
I2 nf
Squaring, {hf_ b g f- ^ h_ a f) t- {ab_ htf - ...(6)
DifiFerentiating (4) partially w.r.t. a, A, c respectively, we have
0A/9a=Ac—/ 2, 8 /\fb = a c - g2, d/\/8c=ab—A2. ...(7)
Now (A /-Ag)2=A2/ 2+ A V lbfgh
=A2/ 2+A2g2- A ( a f2+bg2+ch2-,-abc) using (5)
= A2/ 2+ A V - f lA/2— A V — AcA2+ ab2c
= h2f 2- a b f 2-b c h 2+ab2c
= - / 2 (aA-A2)+Ac OA-A2)
=O A -A 2) ( A c - /2).
Similarly (gA a / ) 2=(aA A2) (ea—g2).
Sub stitu tin g th ese valu es in (6), we get
/2 ___ wr________
(aA-A2) (Ac / 2) (aA—A2) (ca —g 2) OA-A2)2
. /2 m2 M2
or
A c - / 2 - 'c a - g!2 aA A2
I2 m2 n2
or
ο 'Δ ~ 9Δ ^ 0 Δ [using (7)]
aa 0A' 0C
Ex. 9. Abr vvAar values of k do the planes
x - v + z + 1=0, kx+ 3y+ 2z ~ 3=0, % x+ky+z--2 —0
(i) intersect in a point ; (ii) intersect in a line ; (iii)form a
triangular prism ?
Sol. The rectangular army of coefficients is
I -I I I
k 3 2 3
3 k I --2
N o w w e calcu late th e fo llo w in g d e te r m in a n ts:
Δ *= I -I I O -I O
A 3 2 = A+ 3 3 5
3 k I 3+A k A +1
adding 2nd column to 1st and 3rd
=(A +3) O -I O
I 3 5 = 0 + 3 ) 0 + 1 —5)
I A k +■ I
= 0 + 3 ) 0 - 4 ; ..
170 Analytical Geometry 3-D
Δ»=* I -I I 0 -I 0 , adding I
to 1st i
k 3 - 3 = Jfc+3 3 0
3 k -2 3+fc k k —2
rr i( k+ 3 ) (Jk--2 ),
L^ t = I I ·. I 0 I 0
k 2 -3 Jfc- 2 2 -5
3 I -2 2 I -3
adding ( —1) times 2nd column to 1st and 3rd
-■ - { ( * - 2) (-3 )+ 1 0 } =3Jfc-16,
and Δ ι " -I I I 0 I I
3 2 -3 = 0 · 2 -3
k I -2 k -2 I -2
adding 3rd column to 1st(i)
—- 5 (k—2).
(i) The given planes will intersect in a ,.pint if ArT^O an^
so we must have k -φ — 3 and A:^4. Thus the given planes will
intersect in a point for all real values of k other than —3 and 4.
(ii) If f c = ~ 3, we have Aa=O, Ae=O but Aa^O. Hence the
given planes will form a triangular prism if k ——3.
(iij) If k= 4, we have Aa=O but Aat^O. Hence the given
planes will form a triangular prism if k*=*4.
We observe that for no value of k the given planes will have
a common line of intersection.
Ex. 10. The plane xla-ry/h+ z/c= ]. meets the axes in A9 B
and C. Prove that the planes through the axes and the internal bise
ctors o f the angles of the triangle ABC pass through the line
x___________ y___________ z
a \/( b 2-\-c2) “ by j(c2+ a2)~~C\/(a2-\-b2)
Sol· Plane x/a+ylb+z/c=*! meets the axes in A (a, 0, 0),
B (0f b, 0), C (0, 0, c).
The equation of any plane through x-axis is (Le. y = 0, z=0)
is .y-t-λζ—0. ...(I)
The d.r.’s of AB are —a, b ' 0
The Straight Line 171
—a
D.C.’s of AB are -,0
Via*+/»*)’ V(a2+*>2)
—a
D.C.’S Of AC y ^ a»+c*yu 0,
v. y/(Q*c+ c*)
D.C.’s of interior bisector of L-BAC are
1 . 1 \ b c
- t (;tV (a2+ * 2) V (a * + c 2) / ’ 2 V ia2+**) ’ 2 V ia2H-C*)
Now if plane (I) passes through the internal bisector of
L.BAC, then normal of (I) will be perpendicular to the internal
bisector of l_ BAC.
is I lx —v—3z 35=0.
5. Show that the equation of the plane containing the line
ί > - Η ’*=0
and parallel to the line x/a-~zjc=\, is
-a£ _ Zb _ e. £ + 1_ 0.
6. Find the equations of the perpendicular from the point
(I, 6, 3) to the line
Xl y — I z —2
1 "r T " = ~ T ~ ’
Find also the coordinates of the foot of the perpendicular.
Ans. Coordinates of the foot of the perpendicular are
(1,3,5).
Equations of the perpendicular are
x —I y — 6 z—3
0 “ -3 = T "·
7. Show that the following pairs of lines are coplanar :
(i) λ '~ 4 = —J ( j;+ l)= z
and 4x —y + 5z—7 = 0 = 2 x — 5 y — z — 3.
Also find the equation of the plane containing them.
(Kanpur 1980)
(i'i) = O '-Z K -.fz+ l)
and x + 2 y + 3 z = 0 = 2 x + 4 y + 3 z + 3 .
Also find the point of intersection.
Ans. (i) x + 2 y -f-3z=2 (ii) ( 9 ,- 6 ,1 ) .
Show that the lines
x + 2 y — 5 z + 9 = 0 = 3 x —y + 2 z — 5
and 2 x + 3 y — z — 3 = 0 = 4 x — 5 y + z + 3 are coplanar.
9. Prove that the lines
x - 3 y + 2 z + 4 = 0 —2 x + y - t- 4 z + I
The Straight Line 173
and 3 x + 2 .y + 5 z — l = 0 = 2 y + z
intersect at the p o in t (3, I, — 2).
10. F ind the eq u ation s o f the line w hich can be draw n from th e
p oin t (2 , — I, 3) to in tersect the lines
Η * - 1 ) = έ ϋ > - 2 ) = έ (7-3)
and J (x— i ) = (z+3).
Ans. J 2x+4y—9z+7 = 0 = 11x - W y + 2 z - 38.
11, Find the distance of (—2, i, 5) from the linethrough (2,.3, 5)
whose direction cosines are proportional to 2, —3, 6.
f * (6 1 ).
12. Prove that the equations of the perpendicular from th e p o in t
(I, 6, 3) to the line
v— I z—2 x — I V— 6 z—3
X= 2 = — are —0 = ^-— 3
and the coordinates of the foot of the perpendicular are
(1 ,3 ,5 ). (Meerut 1977)
13. Find the locus of a point which moves so that its distance
from the line x —y —z is twice its distance from the plane
x + y + z = I.
A n s. Λ·ί + > · ί + 2 2 + 5 Λ ^ + 5 ^ 2 + 5 ζ Λ Γ — 4 x — 4 v — 4 7 + 2 = 0 .
5
Shortest Distance
Definitions.
§ I. Skew lines.
Two lines are called skew lines or non-intersecting lines if they
do not lie in the same plane. Skew lines never intersect and are
not parallel.
Shortest distance. The straight line which is perpendicular to
each of the two skew lines is called the line of shortest distance.
The length of Jhe line of shortest distance intercepted between the
skew lines is called the length of the shortest distance. The
shortest distance is briefly written as S.. D.
§ 2. Length the equations of the line of shortest distance.
To find the shortest distance between two given lines and to
obtain the equations of the shortest distance.
(Allahabad 1978 ; Behrampur 81; Gauhati 78; Indore 78;
Kanpur 77. 78, 83; Punjab 76; Rajasthan 73, 77; Rohilkhand 81)
Several methods, depending upon the forms of the equations
of the skew lines, are followed to find the shortest distance. They
are as follows :
Method I. Projection method. The equations of the skew lines
being given in symmetrical forms.
Let the equations of the given lines
be
I
X -X l
SSi
*
IN
N
11
I
I
Ii mi »1 (I)
and
X - X t •y—y * Z- Zt
U m2
The line (I) is passing through the
point A (xl9 y u Z1) and has d.c.’s pro
portional to Iu mu nv The line (2) is passing through the point
y 2 * z%) and has d.c.’s proportional to I29 m%9 n2.
Shortest Distance 175
Z2 JM2 M2
This is the required length of the S.D. between the given
lines (I) and (2).
The equations of the shortest distance. Clearly the line PQ of
the shortest distance is coplanar with both the given lines (I) and
(2). Hence the line PQ of shortest distance is the line of inter
section of the two planes, namely, (i) the plane containing the
given line (I) and the IineZjC o f shortest distanc; and (ii) the
plane containing the given Iin i (2) and the line PQ of the shortest
distance.
Now the equation of the plane containing the given line (I)
and the line PQ of the shortest distance whose d.c.’s are Z, mj, m is
i1
-X i
I
y -y i
N
lx mi «! = 0. -(4 )
i . m M
Also the equation of the plane containing the given line (3)
and the line PQ of the shortest distance whose d.c.’s are Z, mi, m is
176 Analytical Geometry 3-Z)
X-X2 y -y 2 Z -Z 2
U m2 n* =0. ...(5)
I m n
The equations of the line PQ of the shortest distance are
given by the planes (4) and (5) taken together.
Note. If the shortest distance PQ=0, then the two given
lines (I) and (2) will intersect Le., they will be coplanar. From
(3), we observe that
X2 - X 1 y 2 -y \ Za — Z 1
h W 1 Wl
It W 2 W 9
which is also the condition for the two lines (I) and (2) to be
coplanar. [See chapter 4 § 8 (A)].
Hence we can give an another statement for the two lines to
be coplanar.
itTwo lines are coplanar if the shortest distance between them
vanishes”
M ethodii. General co-ordinates. The equations of the two
lines being given in symmetrical form :
Let the equations of the two lines be given by (I) and (2)
[See method I].
Thegeneral co-ordinates of the points on the two lines(l)
and (2) are given by
M j1+ y l9 ¥ i+ Z i) , say the point P.
and (/2ra+ x 2, Wzr*+y 2>w2ra+ z2), saY the Point Q-
Let P and Q be the points where the line of shortest distance
meets the given lines (I) and (2) respectively, so that the line PQ
is perpendicular to both the given lines (I) and (2).
Now find the d r.’s of the line PQ and apply the conditions
that the line PQ is perpendicular to both the given lines (I) and
(2). Thus two equations in rx and r2 are obtained. Solve these
equations to get rx and r2. Having found i\ and r2, the co-ordinates
of the points P and Q and also the d.r.’s of the line PQ are
known. Now we can at once find the length PQ of the shortest
distance and also the equations of PQ.
Shortest Distance
177
SOLVED EXAMPLES
Ex. I. Find the shortest distance between the lines
( * - 1 )/2 = (,-2 )/3 = (2 -3 )/4 ;
( * - 2 ) /3 = ( ,-4 )/4 = (2 -5 )/5 .
Show also that the equations of the shortest distance are
l l x + 2 , —7 r + 6 = 0 = 7 je + ,—52+7.
(Agra 1974, 78; Berahmpor 76S, 8IS; Madras 76;
Meerut 78, 86, 89; Vikram 78)
Sol. The equations of the given lines are
( x - l ) / 2 = ( , - 2)/3= (2-3)/4 = r, (say) ...(D
( * - 2 ) /3 = ( ,- 4 ) /4 = ( 2 - 5 ) /5 = r , (say). ...(2)
M ethodi. (Projection method). Let /, m,n be the d.c.’s of
the line of S. D. Since it is perpendicular to both the given lines
(I) and (2), therefore we have
2/+ 3m + 4n=0; 3/+4m +5n=0.
Solviog, we get
I m n Λ/(/*+/η*+η*) I
- 1 = 2 “ - l “ { ( - 1)»+ (2)»+(-l)*}= V6
A The d.c.’s of S. D. are - 1/V 6, 2 /,/6 , - 1 / ^ 6 .
Now A (I, 2, 3) is a point on line (I) and B (2, 4, 5) is a
point on the line (2). Hence
the length of S.D .=the projection of join of A and B on the line
whose d.c.’s are —l/y/6 , 2 /^ 6 , —\/i/6
= ( - l / V 6 ) ( 2 -1 )+ (2 /^ 6 ) ( 4 - 2 ) + ( - W 6 ) (5 -3 )
= 1/V6.
The equations of S. D. (See § 2)
Theeqdations o fth ep Ia n eth ro u g h th e Iin e(I) and S. D. is
x —I y —2 2—3
2 3 4 =O or l l x + 2 , —7 2 ^ 6 = 0 ...(3)
-I 2 -1
And the equation of the plane through the line (2) and S.D.
U x —2 y —4 2—5 I
3 4 5 I=O or 7 jc+ , - 5 2 + 7 = u ...(4)
-I 2 -1
The equations (3) and (4) together are the equations of the
s.a
Shortest Distance 179
- M - D ' A t - U M ) ' }
= /( 2 - 1 ) + /« (3 -2 )+ n (4 -3 )
Since the length of S.D.=0, hence the given lines are copla
nar Le. intersecting.
Ex. 5. Find the points on the lines
—x y+ 9 z—2
—— = —(y —7) = z—4 and
which are nearest to each other. Hence find the shortest distance
between the lines and also its equations. (Bnndelkkand 1978)
182 Analy tical Geometry 3-D
6 + 6 = - 6 ^1 2 ~ 72=36
or I _ m _ / i _ V(JJ+/n* + «2) _ 1
Shortest Distance 18)
Clearly the line (I) passes through the point A(—a, 0, 0) and
the line (2) passes through B(0, —2a, a).
The length of S.D.=T he projection of join o f ^ a n d P .
on the line of S.D. whose d.c.’s are
I, m, /i.
= /(0 + a )+ /n (—2a—0 )+ n (a—0)
=(2/7) a —(3/7) ( —2a)+(6/7) a - 2 * .
Proved.
Ex. 7. Ifth ea xesa re rectangular, find the shortest distance
between the lines y= az+ b, z=otx+/3, and y= a'z+ b', z = a 'x + j8'.
Hence deduce the condition for the lines to be coplanar.
Sol. The equations of the given lines in symmetrical form
are given by
Ijx a = 1’ .( 1)
and x + F I* '_ y - b '_ _ z
Ijx' a' i - ( 2)
Let I, m, n be the d.c.’s of the line of S;D. The line of S.D.
being perpendicular to both the lines (I) aud (2), we have
/.(l/« )+ w .a + n . 1=0 and /(I/a ')+ m a'+ / 1.1 = 0.
Solving these relations, we get
I m
a—o' ( l/ « ') - ( t/a ; (1/a) a '—(I/* ').a
I m n
ot
xx' (a—a') a —a' a’x '—ax
_________________ I________________
ta —a 'j'+ t* —x')*+{a’x '—aa)*} ...(3)
C IearIytheIine(I) passes through the point Α(—β(χ, b, 0)
and the line (2) passes through 21(-/37*', b \ 0).
The length of S.D.=Tbe projection of the join of A and B on
the line of S.D. whose d.c.’s I, m, n are
given by (3)
= /( - /3 7 ^ + /3 ^ ) + //^ - 6 ) + /1 ( 0 - 0 )
I I
*.Γ ϋ
I
I I 2 J
Shortest Distance 185
or x ( 2 + 1 ) - ^ ( 2 + l) + z ( 1 - 1 ) = 0 .
or x —,y==0. —(4)
The equation of the plane through the line (2) and S.D. is
X y z —a
I -I 0
I I 2
or Jt ( - 2 - 0 ) ~ > ( 2 - 0 ) + ( z - a ) (1 + 1 )= 0
or x + y - z + a —0. -(5)
The equations (4) and (5) together are the equations of the
S.D. These equations are clearly satisfied by the point
x —y--^>z= —a.
By the symmetry of the co-ordinates of the point
X --y = Z = r -a ,
it follows that the other lines of S.D. between other two pairs of
opposite edges will pass through the point jc= j;=z*=»—o.
Hencethe three’lines of shortest distance intersect at the point
x= a.
Ex. 9. Find the length and position of the S.D. between the
lines
* 5x -- 2> —3z+ 6= 0 = x —3>>+ 2z—3.
ΙΓ the plane (3) is parallel to the line (I) [i.e. z-axis] then the
normal to the plane (3) will be perpendicular to the line (I) and
hence we have
0.(1 + 2A) + 0.(I + 3A)+1.(2+4A) = 0, or A = - J .
Putting thisvalue of A in (3), the equation of the plane
through the line (2) and parallel to z-axis [i.e, (1)] is given by
x 0 + ^/.(1—3/2) + z.O—3 + 2 --0 , or _y+2=0. ...(4)
Clearly z-axis i.e., the line (I) passes through the point
(0 .0 ,0 ).
Length of S.D. —the length of perpendicular from the
point (0, 0, 0) to the plane (4)
04-2 2
^ “ i
Ex. 11. Find the shortest distance between the z-axis and the
line
ax+ by+ cz + d —Q= a'x+ b'y+ c'z+ d'. (Meernt 1984)
Show also that it meets the z-axis at a point whose distance
from the origin is
(ab' — d'b) (be'—b'c) + (ca'—c'a) (ad’—a'd)
(6t' —b’c)a+ (ca'—c'af *
(Agra 1979; Allahabad 78; Boodelkhand 79;
Kanpur 75, 76, 79, 82; Lucknow 77, 82; Rajasthan 78)
Sol. The equations of the.z-axis are
5 - o - f - r*(say)* ...(I)
The equations of the other line are
ax+ by + c z + d ^ O ^ a 'x + b 'y + c 'z + d '. ...(2)
The length of S.D. We shall rind it by Method HL
The equation of any plane through the line (2) is
(ax + by+ cz+ d) +A (a'x + b'y + c'z+ d ')=*0
or x (fl+Ai/)+^ (6 + A6f) + z (c+Ac')+(i/+-A6f,)=aO. ...(3)
If the plane (3) is parallel to the line (I) [i.e. z-axis], then
0 (a + Aa ) + 0.(6 + A6 ) + 1. (c+ Ac;)— 0 or A= —c/c*
Putting this value of A in (3), the equation of the plane
through the line (2) and parallel to z-axis is given by
x (a—ca'/c')+ y {b -c b '/c ')+ z ( c - c ) + ( d —cd’lc')=*0
OT x (ac'—ca')+ y (b c '- cb')+(dc’—cd’) = 0. ...(4)
Clearly z-axis i.e., the line (I) passes through the point
(0, 0, 0).
188 Analytical Geometry 3-D
Ex. 12. Show that the shortest distance between the lines
x - x \ _ y - y i _ z - z i x —Xj _ y - y % _ *—*t
cos a, cos βι cos γ\ cos a2 aaCos /S2 cos y2
meets the first line in a point whose distance from (xlf yt, Z 1) is
[27 ( ( X 1 - x t) ( cos « i — cos B cos * » ) } ] / j / n * Θ
where B is the angle between the lines.
The epuations of the given lines are
* *' _ r - ? _ (say)
COS «1 COS βι COS yi x ...(D
x —x* y —y* z - z % ,
----- ?=> -— ~5 β -------- =>Γ| (say).
COSa2 COSjSa COSy2 ...(2)
Aoy point P on (I) is
(rt cos * i+ x ,. r, cos /3,+y,. rx cos y , + Z i )
and any point Q on (2) is
(r, cos cc2+ x 2, r2 cos /S2+ y 2, r%cos y 2 + z 2) .
Let the shortest distance meet the line (I) at P and the line
(2) at Q.
Now, siuce cos otlf cos /Slf cos y, are the actual d.c.’s of the
line (I). therefore r2 is the actual distance of P from the point
(x„ yi, z,) on the line (I). It is required to find rx.
D .r/s of PQ are rt cos a ,+Xj - r2cos a*—x2, T1 cos β ι+ yi
—r2 cos β i—y»,rι cos yx+Z i—r2 cos y2—z2.
Since PQ is the line.of shortest distance, therefore, it is per
pendicular to both the lines (I) and (2), and hence, we have
cos «i (rt cos Ot1-Hx, —rt cos a2—x2)
+cos βιXri cos β ι+ y i- r t cos βt - y t)
+cos yi (r, cos y i + Z 1- T 2 cos y2—z2)=»0 ...(3)
and cos ot2 (T1 cos a, + x, — r2 cos a2—x2)
+cos /J2 (r\ cos βι +y, —r2 cos βt —y»)
+cos y2 (Ti cos y,+ Zi —r2 cos y2—ζ2)=·0. ...(4)
Now
cos* a!+cos® /8|+cos* yi==l, cos® a2+cos* /S2+cos* y2= l I
and cos at cos a2+cos /S1 cos /S2+ cos γχ Cosy2= cos 6, f
-(5 )
B being given to.be the angle between the lines (I) and (2).
From equation (3/, we have
T1 (cos* Ot1 +cos® /S1+cos* Yi)—r%(cos Ot1 cos a2+cos /S1 cos /S2
+cos yi cos y2)+ (x i—Xt). cos y2) cos /S1
+ U i-z » ) Cosy1=O
190 Analytical Geometry 3-D
(fx—\ z — 1)+Ay=0
or Z x + U X y -A z-M = Q . .(3 )
If the plane (3) is parallel to the line (I), then the normal to
(3) is perpendicular to the line (I), and hence we have
0 .3 + 2 .12A—3 .( - 4 ) = 0 or A=» - J
Putting the value of Λin (3), the equation of the plane through
the line (2) and parallel to the line (I) is given by
3 * _ 6 y —4z—12=0. (4)
The line (I) clearly passes through the point (0, 2, 0).
The length of S.D .=the perpendicular distance of (0 ,2 ,0 )
from the plane (4)
0 —12—0—12 _ 24
(Numerically).
v/{(3)»+(-6)*+(-4)*}= V(^I)
Ex. 18. Prove that the S.D. between the\llnes
ax+by+cz+d=0=>Q'x+b'y+c,z+ d '
and ΛX+ βy+γz-^-δ= 0=Λ'x+ β'y+γ,z+ δ' is
d d' δ δ'
a a’ a d'
+ y/[Z (B C -P C )* ]
b b’ β β'
c c' γ y'
where A ^ b c '—b'c and Α '= β γ'—β’γ etc.
Sol. The equations of the given lines are
ax+by-\-cz-\-d=Q=‘Q,x + b’y + c ,z+ d ' -.--(I)
and ax+ /3 y + y z+ e= 0 = a'x + /3 '.y + y 'z+ 8 '. ...(2)
We shall use method IV. The equations of any planes through
the given lines (I) and (2) are
(ax+by+cz+d)+Xx (a'x+ b'y+ c'z+ d')—0 ...(3)
or x(a+ X la,)+ y (b + \ xb ')+ z(e + \le,)+(d+Xld ')» 0 ...(3')
and (oιx+βy + γz-\-δ)+λ9 (<ι'x+β'y+γ'z+δ')=0 ...(4)
or x (a+fl2a')+_y (/J+A^'J + z (y+Afy') + {?)+A.j8')= 0 ...(4')
If the planes (3') and (4') [l.e. (3) and (4)] are parallel, then
their corresponding coefficients are proportional and so we have
α+λ,ο'^ δ + λ ,δ ' ^ + Α , ι ' ,
a+Asa' β+ \φ’ y+A,y' K
From these relations, we get
0 + A1**' — ArA2Ocr= 0 ... (5)
196 Analytical Geometry 3 -D
b + X ib ’ - Ι ι β - ί Γ λ ύ S'=0 ^ (6)
c+Atc'—Ary-ArAiy“ 0. ---(I)
Eliminating Aj, -Ar, -ArA1 betv^een (3), (5), (6) and (7), the
equation of the plane through the line <1) and parallel to (2) is
given by vv x
ax+ by+ cz+ d a'x+ b'y+ c’z+ d ’ v 0
a a' a a' 0.
b b' β β'
C c' Y Ύ
Adding (—x) times second, (—>*) times third and ( —z) times
fourth row to the first row, we get
d d' —(«x+ fiy+yz)
a a' a a' . —0 ...(8)
b b’ β β'
C c' Y y'
Now we shall evaluate the coefficients of x ,y and z in the
expansion of the determinant in (8).
The coefficient of x
« - a a a' a' ( - « ') a a' a
h V β9 b b' β
C c9 y9 C c' Y.
e= a a! —a a ' a a' aa'
a a' a a'
b b’ β β'
C c' Y /
- r -^[(coefficient of x)*+(coeff. of y)*+(coeff. of z)2].
Putting values from (9) and (10), we get the required S.D.
d d' δ δ'
a a' a a'
b b' β β'
C c' Y y' Proved.
Ex. 19. Two straight
X- a i - y - β i ,
/. W1 fli It mt nt
are cut by a third line whose direction cosines are λ, μ, v. Show
that ‘d’ the length intercepted on the third tine is given by
d Λ Wi Πι - 0C|—OC2 β ΐ-β * yi—ya
It W2 W2 /i mx «1
λ V Ii W2W1
Deduce the length o f the shortest distance between the first two
lines.
Sol. The equations of the given lines are
(x -c tiV /i-iy -ft)/» » !—(z-y i)/n i= r! (say) • -(1)
198 Analytical Geometry 3-Z)
0μ + {\Si-/3.) W1J m%
dv f (y i—y.' «1 n2
Spilling this determinant into two determinants» we have
d A /i /a + «1 — a2 /1 /. « 0
μ mi ma 01 - / 3 . w, m.
V Wi W2 yi W2
I
e>
d A. /i /2 = — « 1 — «a /1 h
μ mi ma 01 — 02 Wli m2
V Wi W2 n - y* Wl Wa
d /l '« I Wi = — «1 — «2 0 1 -0 . y i - y 2
h m2 W2 /1 Wli Wl
A V ' /2 Wla Wa
Shortest D istt 199
/. HT1 H1
Z1 HT1 W
1
Exercises
I. Find the equations of the straight line perpendicular to both
of the lines
x - I j - 1 z+ 2 x + 2 y - S z+ 3
T ^ — T~ “ d T - = = T — 2 -‘
x —2 y - 3 z - 1
Aas.
7 “ 4 ~ —5 '
2. Find the length and equations of the common perpendicular
to the two lines
200 Analytical Geometry 3-D
* + 3 y —6 z . *+2 y z -7
- 4 —■— 3 -" 2 ----
*“d - 4 ~ i !Ί
(Meerot 1984; Andhca 68; Gorakbpnr 74; Madras 76)
Ans. The length of common perpendicular (i.e. S.D.)=9.
Tbe equations of S.D. are
32*+34,y+13z—1 0 8 = 0 = 4 * + 1lj»+5z—27.
3. Show that the length of shortest distance between the lines
* —2 .y + l z .
2 ~ 3 =4 ;
2*+3j’—5z—6 = 0 = 3 * —2y—z+3 is 97/(13^6)·
(Rajasthan 1975)
4. Find the length and the equations of the shortest distance
between the lines
5*— z = 0 = * —2_y+z+3
and 7*—4y—2 z = 0 = * —y + z —3. (Meerut 1986 S)
Ans. The length of S. D. is 13ISy/2.
The equations of S. D. are
17*+20j>—19z—39=0, 8 * + 5 .y -3 lz+ 6 7 = 0 .
5. Find the equation of the shortest distance and its length
between the lines
* —3 y —5 z - 2
I “ -2 " I ’
* —I y+ 1 z+1
'7 " - 6 “ Ί (M. U. 1990)
6
Volume of Tetrahedron
k x e lT S R T
or V= i la, b, cj. ...(4)
This is the required formula for the volume of the tetra
hedron.
202 Analytical Geometry 3-D
= i i X8- X 4 Ji2-JU Zi - Z 4
! X8- X 8 JU-JU Z3 - Z 4
Volume o f Tetrahedron 203
* 1 - *4 y i-> i Z 1- Z 4 0'
*2 -*4 y *-y 4 Z2 - Z i 0
£
I
*4 Z s-Z 4
CO
*4 y« Z4 I
X2 Z2 I
. ( 6)
Xs ys Z9 I
X4 >’4 Z4 I
λ*2 ys Z2 I
*3 Xs Zs I
*4 ) ’* Z4 I
UC mzc ZV8C
= £ abc Zi Wi Wi
Z8 W8 /I2
/3 m8 w3
Fi = - I f l W u WI1 W1 /1 W1 W1
Jo
U W8 W2 X Z2 W2 W8
Z3 W3 W8 /3 W3 W8
I cos v cos μ
=■·-T- fl‘CSC*
36
COS V I cos A [using the relations (I)].
cos μ cos Λ I
Λ F = ± J ubc I COS V COS μ *
cos v I cos Λ
COS μ COS A I
The negative sign will be neglected in calculating tbe magni
tude of the ycIume V.
§ 3. To find the volume V o f the tetrahedron when equations o f
its four faces are given.
Volume o f Tetrahedron 205
b» C» d, Q2 C2 d2
bt ct dt I at ct dt
i
Z -I
Q2 b2 ar2 Q2 b2 C2
Q2 b2 a as bi Cz
Qi bA d ot b4 Ci
Suppose Δ = ai bx Cl dx
Q2 bt C2 dt
Qz bt C2 dt
Qi bt Ci dt
I A t B% ^ \ , /^ i Β» C*V ( A t B t C t Y
DtV \Z V D i ' D t ) '
I i V D tt D t ) ' \ Dat ZV
Λ The required volume V of the tetrahedron is given by
At Bi Cl
I
Di Di Di
At Bt Ct
I
Dt D2 Dt
[See § I (C), equation (6)]
At Bt Cz
I
Dt Dt Dt
At B, Ct
I
Dt Dt Dt
I Ai Bi Cl Bi
6Z)| D] OgDi
At Bt Ct Dt
At Bt Ct Di
At Bt Ct Dt
_ A*-1 [since if A ' be the determinant of nth order
6Z)(DtDtDt formed by the co-factors of the elements of
the determinant A. then Δ '= Δ " -1]·
•'· V*< A8
IiDlDiDtDt
SOLVED EXAMPLES
Ex. I. Find the volume o f the tetrahedron, the co-ordinates of
whose vertices are (2, —1, —3), (4, .1, 3), (3, 2, —1) and (I, 4, 2).
(Punjab Ϊ981, 82)
Sol. The volume V of the tetrahedron, the co-ordinates of
whose vertices are (2, —1, —3), (4, I, 3), (3* 2, —l) and (1,.4, 2)
is given by
2 -I -3 I [using (6) of § I (C)]
4 l: 3 I
3 ’ 2 -I I
I 4 2 I
Vobune of Tetrahedron 207
I 3 2 0
-I 5 5 0
— i 2 2 6 expanding (he determinant
along the 4th column
I 3 2
-I 5 5
- _ 1· 0 12 16
1st row and adding 3rd row
0 8 7 to She 2nd row
-I 55
12 16 I, expanding with respect to 1st column
8 7 I
>i ( 8 4 - 1 2 8 ) = - (Numerically).
0 3
3 0 1 }
4 3 t. I
2 3 2 I
-3 I I 0 , adding (—1) times 2nd
row to each of the other
3 0 I I rows
I 3 5 0
—1 3 I 0
208 Analytical Geometry 3-D
-1 3 I
0 -2 --LΛ
· I ,ad d in g 3rd column to
the 1st column and
6 -1 2 5 ( - 3 ) times 3rd column
to the 2nd
0 0 I
0 -2 -2
6 -1 2 It expanding w.r.t., 3rd row
-I I I
0 0 -I
6Imn
2 0 I ,
0 2 I
adding 3rd
-P z 2' 0
6lmn 0 ■ 2 , expanding al<
= J= V = 2p*_
(numerically). .(6 )
6lmn ilmn
Proved.
To dedoce the area of the triangle. Let the area of the triangle
formed on the plane (4) be Δ· Also let the length of the per
pendicular from the opposite vertex O to the plane (4) be denoted
by rf. so that
’ ...(7)
Also the volume V of the tetrahedron is given by
Κ = Μ .Δ ·
_3V 3.2p* -\/(/2+/w*-fn2)
Λ “ d 3lmn' p
[Putting the values from (6) and (7)]
2p2y/ (I2+ m2+ n*}/( lmn).
Ex. 4. A9B 9C are three fixed points and a variable point P
moves so that the volume of the tetrahedron PABC is constant. Find
the locus of P and show that i: is a plane parallel to the plane ABC.
Sol. Let the co-ordinates of the fixed points Ay B9 and C be
taken as (a90, 0), (0, b, CMand (0, 0, c) respectively. Also let the
210 Analytical Geometry 3-0
variable point P be (Jitl, y it Z1). Now we are given that the volume
of the tetrahedron / yXBCaconstant, say k.
. I a O O I /=Jfc
*· 6
O b O I
O O c l
Xi yt zt I
0 I . J -X 1 0 0 I
I *
0 C I b 0 I
yt Zl . I I 0 C I
Xt yt Zi I
[Using (6) of § I (C)J
Volume o f Tetrahedron 211
0 I 2 I
3 -I -I 0
2 2 0 0
X\ T i - I zi—2 0
[V F = 6, as given. Also adding (—1) times 1st row to each
of the other rows]
or 36= -J 2 2 0 ,
: Xi T i-1 Zj —2
expanding w.r.t. 4th column
or 36 3 -4 -I
2 0 0 ,
—Χ ι+ Λ -1 «1—2
adding (—1) times 1st column to the 2nd
or 36=2 -4 -I
—*i+Ti —I «1—2
expanding w.r.t. 2nd row
or 1 8 = - 4 (zx—2) + 1 .( - X i + ^ , - 1 )
or 18= —4zi + 8 —Xi+y{—I
or Xi —^ i+ 4 z i+ 11=0.
The locus of P is the plane x —_>’+ 4 z + ll = 0 .
Ex. 6. A variable plane makes with the co-ordinate planes a
tetrahedron of constant volume 64k*. Find
(I) the locus of the centroid of the tetrahedron.
(Meerut 1977; Indore 76)
and (//) the locus o f the foot o f the perpendicular from the origin to
the plane.
Sol. Let the equation of the variable plane be
x/a + ylb + z/c= l. ...(I)
The equations of the coordinate planes are x = 0 , ^ = 0 , z= 0 .
Solving the above four equations taking three at a time, the
vertices of the tetrahedron are given by
<9(0, 0, 0); A(a, 0, 0); 5(0, b. 0); C(0, 0, c).
212 Analytical Geometry 3 D
0 b 0
0 0 C
64&3=& abc (V F = 64/c8 as given]
or e*c=384frs. ...(2)
(i) Now let the co-ordinates of the centroid of the tetrahe
dron OABC be (α, β , y), so that we have
O+a+O+O 0 04-0 + 6 + 0 · 0 + 0 + 0 + c
« - — 4-------. β -------- 4------ , y*=— 4— -
or a —4a, b —4β9 c= 4y.
Substituting these values of a9 b, c iu (2), v/e get
(4α) (4β) (4y)=384 fc3 or αβγ*=>6k*.
A the locus cf the centroid (a, β, y) is xyz*=6k*.
(ii) Let (.X1, y u 2,) be the co-ordinates of the foot of the per
pendicular from the origin O to the plane (I).
Thedirectionratiosofthis perpendicular are X1- 0, y \ ~ Ot
2i—0 t.e. x u y u Z1. Also the d.r.’s of the normal to the plane (I)
are I /a, 1/6, I/c. The two being parallel, we have
Ma I Ib I Ic .(3)
The foot of the perpendicular (xu yt9 Zi) lies on the plane
(I) and so we have xilo+ yilb+ 2ilc**l. ...(4)
From (3), we have
ill-* a - ' *va+ v + * i a ^ ^ι1+ ^ !1+ ^ 8
Xilo y jb ZlIc (*i/a)+(;Vi/6)+(Zi/c) 1
[using (4)1
or ^»Xi, + y ,a+ 2 ii .
Λ α= (Xi*+.Pif +Z18Vx1, b ^ ix x '+ y t'+ z M y u
c—(xit + yit + zi2)/zl.
Substituting these values of a, b and c in (2), we have
(Xi2+ J i2+ Zit )* = 384 X1 y x Z1 k 3.
Λ The locus of the foot of the perpendicular Ue. the locus
of (Xu yu Zi) is given by
(xa+ y 2+ 22)3= 384 xyz k3.
Ex. 7. The lengths of two opposite edges of a tetrahedron are
a, b, their shortest dista ice is equal to d and the angle between them
ts Θ. - Show that the volume of the tetrahedron is J abd sin Θ.
Volume of Tetrahedron 213
λ μ ' * ■ .(2 )
Sioce the point D is on the line CD [i.e. (2)] and is at a dis
tance 'b' from C (α, β, γ), hence the co-ordinates of D are
(A6+ «. μb+β, vfr+y).'
Now Θ is the angle between the lines AB and CD i.e. between
the lines (I) and (2), so that
sin θ=*\/ ημ)1]. ...(3)
Also d —the S.D. betweeo the lines (I) and (2)
«—0 /3—0 y -0 («IV—«μ)1]
I m η
A V
α β Y
I m η • ...(4)
λ V
al am on I al am an
X β y + a β y
b\ *>μ fcv i a β y
a β y + 0
•81*
I m η
D
I
X μ V
—J abd sin Θ [using (4) and neglecting —ve sign]
Ex. 8. A point P moves so that three mutually perpendicular
lines PA, PB, PC may be drawn cutting the axes OX, OY, OZ at
A, B, C and the volume of the tetrahedron OABC is constant and
equal to k*/6 Prove that P lies on the surface
(x, +y*-\-z*)3—ik 9 xyz.
Sot. Let the co-ordinates of the moving point P be
(xi, y\> z,). Since A is a point on the x-axis (i.e. on OX), there
fore, let the co-ordinates of ^fbe (a, 0, 0). Similarlylet the co
ordinates of B and C be (0, b, 0) and (0, 0, c) respectively.
N ow thedirection ratio so fth e lines PA, PB and PC are
Xi- a , y t, z,; x„ yx- b , z,; and X1, ylt Z1-C respectively.
It is given that the lines PA, PB and PC are mutually perpen
dicular, so using the formula +CiC*=0’, we have
( χ ,- .β ) χ 1+ Λ .(^ ,-6 )+ Ζ ι.ζ ,= 0
[PA and PB being perpendicular]
or Xi1+ ^ + Z i 1=UXi+ ¢^ 1 . -(1)
Similarly PB and PC; PC and PA are perpendicular, so we have
x * + y \%+z\*=byi+czi -(2)
Χι*+Λ*+Ζι*=»ΓΓι + a x i· -(3 )
The volume of the tetrahedron OABC is given to be k3/6.
a 0 0 k»
[using § I (B)]
i 0 ir o b
0 0 c
OT abc=>k*. -(4 )
Now in order to find the locus of the point P(xu y u z,) we
are to eliminate the unknown quantities a, b, c from the relations
(I), (2), (3) and (4).
VolumeofTetrahedron 215
a b C I
Ir mr nr 0
a' b' c' I
Vr' m'r’ nY 0
adding (-*1) times 1st row to 2nd and (—1) times
3rd row to 4th
^rr' a b c I
I m n 0
a' b' c' I
I' m' n' 0
~ J rr' a—a' b -b ' C — C* 0
I m n 0
a' b' Ct I
I' m' π' 0
adding ( —1) times 3rd row to 1st
= J rr' a —a b -b ' c c' .
I m
I' m' n' I
expanding along the 4th column. Proved.
Exercises
1. Find the volume of the tetrahedron, the co-ordinates of
whose vertices are (I, 0, ·;), (0,0, I ), (0,0, 2) and (I, 2, 3).
Ans. J.
2. Find the volume of the tetrahedron formed by the planes
y + z = 0 , z + x = 0 , χ+ .κ ~ 0 , and x - r y + z = l
Hence deduce the area of the triangle formed on th ; plane
x + y + z - 1.
Ans. Volume—2/3, area-=2/3.
7
Skew Lines
or
[ Z+ C J [
j 8—m2x*= 2*—c*.
Z -C J
Hence proved.
Ex. 2. Find the surface generated by the lines which intersect
the lines j= w x , z =c; y= —mxt z = —c and x-axis.
(Meerut 1973; Garhwal 81)
Sol. As in Ex. I above a line intersecting the first two lines
namely y ~ m x 9 z - c\ y ~ —mx, z=*—c is given by the planes
( j - wix)+A (z -c )= 0 , ...(I)
and (j-fm x)+ /* (z-i-c)=0. ...(2)
The planes (I) and (2) intersect in a line and if this line meets
the x-axis i e. j = 0 , z - 0, then putting j = 0 = z in (I) and (2), we
have
(0—mx) + A (O-C)=O and (0 + mx)-f μ (0+c) = 0
or A =—-mx/c and μ=» —mx/c.
From these two relations, we get A=μ. -. (3)
Therequiredlocusis given by eliminating A and μ between
(I), (2) and (3) and is given by
y —m x y jT t n x
z —c z+ c
or (j+ m x ) ( z - c ) = ( j —mx) (z—c) or cy=mzx.
Ex. 3. A variable line intersects the x-axis and the curve x = j ,
y2«=cz and is parallel to the plane x = 0 . Prove that it generates the
paraboloid xy cz
(Meerut 1972, 76, 89S; Kanpur 77; Kurukshetra 74)
Sol. Theequation of any pfane through x-axis i.e. j = 0 = z is
J e=sAz. ..( I )
The equation of any plane parallel to the plane x = 0 is
χ® μ. ...(2)
The planes(I) and (2) intersect in a line whichintersects the
x-axis and is parallel to the plane x = 0 . Ifthis line meets the
curve x = j , j 2=cz, we have by puttiug x = j in (2),
y= l·· -(3 )
From (I), A = - = —— [V c z~ y2]
z cz y*
220 ■Analytical Geometry i-D
or μλ*= —( \ + λ + μ λ ^
ΟΓ μλ*-\-μλ + 1 +A = Ot o r μ λ (A-I- I)-I- I .(A4 - 1 ) = 0
or (x—y —z ) ( x + y ) + z ( x + y —2o)=‘0
or x»—y*c=2az.
Ex. 7. Find .the locus o f the variable line which cuts the three
linesy= b, z=*—c ; z*^c, x ——a ; x=*a,.y— —b.
[Garhwal 1978 (S); Kanpur 73]
Sol. The equations of the given lines are
y—6= 0, z+ c= 0 . (1), Z - C = O t x+a~-0 . ( 2 )
and x —a=Q, y + b = 0 . .. .(3)
The equations of any planes through the lines (I) and (2) are
( y - b ) - X (z+ c)= 0 ...(4)
and ( ζ - ο ) - μ (x+ a)= 0. ...(5)
The planes (4) and (3) intersect in a line and if this line meets
the line (3) then we are to eliminate x, y, z between the equations
(3), (4) and (5).
From (3), x= a, y = —b. Putting these in (4) and (5), we get
—2b—λ (r+ c ) = 0 and ζ —ο—2αμ=0
Ot z«=(—2ί>/λ)—c and 2 = c+2o/i.
Equating the two values of z, the x, y, z eliminant of (3), (4)
and (5) is given by
(—2 ί>/λ)—c=*c+2au or au+ b/\+ c= 0. ...( 6 )
The required locus is obtained by putting the values of λ and
μ from (4) and (5) in (6 ) and is
Skew Lines 223
a j z - c ) b jz + p 0
x+ a ~ y —b
or a ( y - b ) (z -c ) + b (x+a) (z+c)+c (x+a) ( y - f t ) = 0
or ayz+bzx+cxy+abc·=* 0 .
Ex. 8 . How many lines can be drawn from a point to intersect
two non-coplanar lines neither of which passes through the point ?
Find the equations o f the lines or line which can be drawn from
the point (2, —1 , 3) to intersect the lines
<x—l)/2«=(>.—2 )/3 = (^-3 )/4 ;
(*—4)/4=y/5=(z+3)/3.
Sol. Let (α, βt y) be the co-ordinates of a given point. Let
the equations of the two non-coplanar lines be
y —m x= 0, z —c = 0 ...(I)
and y+mx-^-0, z + c = 0 . ...( 2 )
The equations of any planes through the lines (I) and (2) are
( y - m x ) —X ( z - c ) = 0 ...(3)
and (y+ m x)—p (z+ c)= 0 ...(4)
Since the planes (3) and (4) pass through the point (α, β, y),
so we have
(β - m a)~ Λ(y—c) = 0 and (β + .via) - μ (y + c ) = 0
or A= ()9—m a)/(y -c) and μ = (β+Μ<χ)/(γ+€).
Putting the values of λ and /· in (3) and (4), the equations of
the line through the given point (α, β, γ) and intersecting the
given lines (I) and ( 2 ) are given by
(y~ m x) ( y - c ) - ( z - c ) (j8 - / n a ) = 0 , I
and (y+ m x) (y + c ) - ( z + c) (β + Μ<χ.)τ=0. J
Thus only one line can be drawn from a point to intersect
two non-coplanar lines neither or which passes through the point.
Second part. The equations of the given lines are
x - \ y —2 z - 3 x - 4 y z+ 3
2~ = 3 ; ”4 “ 5.~ I
These lines can be reduced to general form as
i x —2y+ U-O= 2 jc—z + 1 .(1)
and 5x—4y—2 0 = 0 = 3 jc—42—24 ...(2)
The equations of any planes through the lines (I) and (2 ) are
f i x —2y+ \ )+X (2 Xr - z + 1 ) = 0 ...(3)
and (5.x —4y—20)+ μ, (3x—4z—24) = 0. ...(4)
The planes (3) and (4) pass through the point (2, —1 , 3), so
we have
274 . A n a lytica l G eo m etry 3 -D
( 6 .+ 2 4 η + λ ( 4 - 3 + 1)--¾ O or A = — 9/2 ;
(10+4- 20) + μ ( 6 - 1 2 - 2 1)=() or /*= -1 /5 .
Putting the values of A and μ in (3) and (4), the equations of
the line through the point ( 2 , —I, .'*) and intersecting the given
lines (I) rnd.( 2 ) arc given by
12x + 4v Oz+ 7 - 0, 11jc - IOy+ 2z— 28 = 0.
Ex. 9. A line of constant length has its extremities on two
fixed straight Urns. Prove that the locus of its middle point is an
ellipse whose axe* are equally inclined to the lints.
Sol. Let the equations of the two fixed straight lioesbe
given by
Jt/! = y /m = (z—c)/0= rT ·· Π)
and , ' xf\ =y/( ~ m )= (z+ c ) / 0 =^r2. -
The co-ordinates of any two points on (I) and (2 ) are
P (ru mrJ$ c) and Q (r2. —mr^ —c) respectively.
Let (α. β, γ) be the middle point of PQ9 so that we have
a-= J (γ! + γ2) , 0 -~■:* m Oil - r B), y = i (c~c) = 0. ...(3)
Also the length of PQ is given to be const ant, say 2d.
Then 2d=· PQ ·-- distance between P ami Q
or 4d*--“ (r 1 —r2 )2 + m2 (r, f r ,) a-f (c + c)*. -(4 )
Let locus of (α, β, γ) is obtained by eliminating the parameters
Tj and rt between (3) and (4). So putting the values o** Γι —r2and
ri + r 2 from (3) in (4), we get
4d2 (2film)2+ m2 (2a)*+(2c>2, y - 0
or m2(x2+fiilm2+c2=^d2t y - 0 .
Hence the locus of the middle point (a, β, γ) is given by
JWfJt*+y*/m*=rf* —cf, z -0. ...(5)
The equations (5) are the equations of an ellipse in the
jry-plane, whose axes lie along the axr;s of Jt and y. But we know
(see § I) that the axes of x and y bisect the angles between the
lines (I) and (2) Hence the axes of the ellipse (in the present case
axes of Jt and y ) are equally inclined to the gi ven lines.
Ex. 10. A point moves <o that the line joining the feet o f the
perpendiculars from it to twy given straight lines subtends a right
angle at the middle point of their shortest distance (S,D .). Prove
that its locus is a hyperbolic cylinder.
Sol. Let AB and AtB* be the two given linies. Let the
middle point O of the S.D between these lines be taken as origin.
ScewLm a 225
*—
or («*- m*/P)—e* (I +n^Jr/fl m*).
Λ The locus of P («, β , γ ) is
x*+ upy*^e* ( I J m *}*Tl —«*)-
This is clearly the equation of a hyperbolic cylinder.
Ex. 11. fa the above Ex. 10. i f M N is equal to the Jistaaee o f
Pfram the Gtiiin O. then prate that P lies on the surface
(I in1)*(**+.»*>=n+n^J*(4e*~ z*).
SnL We have evaluated in Ex. 10 above that
M is the point (rlt mr,. c) ; N is the point (rt, —nw*, - c)
T1= ( « 4 mp)/(l +iff*). r ,= ( i mfi)l { I +m*)
and the moving point P is fa, β, y ).
Now OP* - ** f I M N f (given). (I)
But ( M N i*=Ir1- r,)*+!·5 ^i+r.'H if+f)*
i+m/J * mfi V . - I m f i a—mfi
( T T rf-τ + rf) + r f I i T r f + i+ rf-
4-4c*.
T
Λ (l + #fi*|*(JlfJV)i =4iii^i*4i»i*.4a*+4c=(l lm*)1
or (I +Ifii/ («*+/!*+ 7 *)=4m* <a*ii-A*)+4cI (14-m*)*, using (I)
or (I +OttIt (*r-i-pt)=4mt (**+p*)=4e* (1+m*)*-?* (1+m*)*
226 Analytical Geometry 3-D
or {(I+m*)*-4iit*} (a*+/»*)=(4c*-y*) (I+m*J*
or
The locus of P (a, β, γ) is
(I —m*)* (**+J* )= (i+m *)* (4c*—z*). Prorei.
Ex. 12. Find the surface generated by a straight line which
meets two line y —mx, z—c\ y = —mx, z= —c at the same angle.
(Kanpor 1982)
Sol. The equations of the given lines may be written as
xfl=y!m ( z - c)/0=r, (say) ...(I)
and x/1 =y/( —m)=(z+c)IQ=rt (say).. ...(2)
Let the moving line meet the given lines (I) and (2) in the
points C (rb Iitr1, c) and D (rt, -Oirs, —c) respectively. The
d.r.’s of CD are r , - r*, m (γ,+ γ,), c+c. The equations to CD
are
X-T1 _ y —mr, z —c
r%—rM~ m (A + r,)” 2c - .(3)
Let the line (3) make angles a and β with the lines (I) and
(2) respectively, so that we have
COS H =
L (r,-r,)+i»i.m (r,+ r,)+ 0.2c_______
V(1 +m , + 0)V ((r1—r,)*+»n* Ir1+ Γ ,Ρ + φ )*} -.(4)
Lfti—rt)—m.m ftt+rt)+0.2c
and cos /1=-
V(»+"«, + 0 ) V t t ^ - fs), + « * fti+'t)*+(2c)*} -(5 )
But according to the given problem, cos a = + cos β. Hence
from (4) and (S), we get
Γ ,-Γ ,+m* ( ^ + ^ = ± [ ( ^ - ^ —111* (r,+ r,)],
the denominators beine same have been cancelled.
Taking + v e sign, we get
- 2m* (C1-Hri)=O or rt ——rx
and taking —ye sign, we get
2 (»i—r*)=0 or T1=C,.
Now when r , = —r1( the line (3) becomes
x - r , y —mrx z - c
2 T1 - 0 2c
X - T 1
Le. y ~ C i
Trx 2c
_ Z y_
Le. --1 = and r ,= —
'l . C m
X Z
Le. — = — and #
'i c m
Skew Lines in
Eliminating ru we get
xm __z_
— = — or mcx=yz. ( 6)
c
Again, when T2 =T1, the line (3) becomes
X -T 1 y -H ir1 z —c
0 ” I m r 1 ~~ 2c
i.e. x T1=O and y / m r ^ z / c ; i . e .·, T1=X and c y — m z r i .
Eliminating T1, we get cy=mzx. (7)
Multiplying (6 ) and (7) the required surface is given by
(mcx—yz) (cy - mzx)= 0.
Ex. 13. Show that the locus o f lines which meet the lines
x + a ___ y_____ z__ . x - a y __ z
0 ~~sin oc“~ —cos a’ 0 ~~sin a cos a
at the same angle is
N (xy cos %—az sin a) (zx sin cl—ay cos a)« 0 .
Sol. The equations of the given lines are
(x+ a)/ 0 = ^ s in a= z/( —cos a ) = ^ ...(D
and (x —a)f0=3yls\n a=z/cos a = r2.
Let the moving line meet the given lines (I) and (2 ) in the
points C ( - a , T1 sin a, - T 1 cos a) and D (a, r 2 sin a, rt cos a)
respectively. The d.r.’s of the line CJDare a+ a9 (r* - T1) sin a,
(r*+ ^) cos a. The equations to CD are
x+ a y ~ r x sin « . Z-Ir1Cosq
2 a “ (r 2 -T1) sin a ""(T2 -Hr1) cos a
Let the line (3) r.'ake angles 0 and φ with the lines (I) and
(2 ) respectively, so that we have
cos sin «.sin ot+ ira+ r^ cos « .(-co s a)
\/{( 2 a)2+ (T2 -T 1) 2 sin* a + (^ + T 1) 2 cos2 a}
V^{0 +sin 2 a+ cos 2 a}
...(4)
. __2 a. 0 + ( r t —T1) sin «.sin oc+(r2 +Ti) cos a. cos a
^""V U 2 f l ) 2 + ^ 2 - T 1) 2 sin2 α-Κ τ,+ Ti) 2 cos* a}
V{0+sin2 a+cos2 a} ...(5)
But according to the given problem, cos B= + cos φ. Hence
from (4) and (5), we get
(T 2 -Tj) sin2 OC--(T2 - T 1) cos2 a
= K i r 2 - T 1) sin2 « + (τ* + ^ ) cos2 a}.
Taking +ve sign, we get
2 (r»+^) cos2 a = 0 or τ, +T j —O or T1- - T 1
228 Analytical Geometry 3-D
Λ Pi=A/?, or Pi2=A2P,*
or [(w *i-> -i)2+ (m 2+ 1) (Z1-C )2J=A2 {(/η χι+ ρ ,)2
H-C"**+1) (Zi+c)*}
[using (3) and (4)].
The required locus of P (Xi., Z1) is given by
(mx—y)2+(/n2+ l ) ( z - c ) * = A * { ( m x 4 - y ) 2+ ( / H 2+ 1) (z+ c2)}
-(5 )
Ex. 14 (b). Find the locus o f a point which is equidistant from
two given lines.
Sol. Proceeding exactly as in Ex. 14 (a) above and putting
A=I [since here Pi=p,], the equation (3) of the required locus is
given by
mxy+c ( l + / n 2) z = 0 .
Ex. 15. P, P' are two variable points on two given non-inter-
secting lines and PP' is of constant length 2k. Find the surface
generated by PP’.
Sol. Let the equations of the given lines be
Jl
tr
O
(I)
N
Il
II
I
Jy ± mx x y —mx I 2c (y z-c m x )
I z+ c 1 Z - - C J m (z2—c2)
C fy+ m x y - mx \ 2c imzx cy)
and rx r2——
m ( z+ c z - c [ ~~ m ( Z 2 - C 2)
Substituting the values of rx+ r2 and ^1- T 2 in (3), the required
locus is given by
4c2 (m z x - c y )2 . 4c2 (yz mcx)2
+4c2=4A2
m2 (z2- c 2)2 ^ m2 {z2 - C 2) 2
or . c2 (mzx—cy)2+ c2m2 (yz- mcx)2= m 2 (A2--Ca) (z2- c 2)2.
Ex. 16. AP and AtPf are two given lines, A and Af being fixed
and P and P' variable points on them such that AP.A’P' is constant.
Find the surface generated by PPf.
Sol. Let the equations of the lines AP and AfPf be respecti
vely given by
x !\= y /m = (z-c)IO = r1 ...(I)
and x/\ =y/( —m )= (z+ c)/0 = r2. :..(2)
The co-ordinates of the fixed points A and Af on the lines (I)
and (2) are (0, 0, c) and (0, 0, - c) respectively.
Let the variable points on the lines (I) and (2) be P(rl9 mrv c)
and Pf (r2, mr2> —c) respectively.
According to the problem AP. AfPf=CowstdLnti say k.
:. V i i '! --0)2+ (/Wr1-O )a+ (c - c ) 2)V { (r2—0 )2 + ( - m r t - C ) 2
+(■-c+ c)2}= k
or V i r l2+ m zr12) .^ ( r 22+ m 2r22) = k
or rvr2 (I + m2)= k or ^ = ^ / ( 1 + m 2). ...(3)
The equations to PPf are
jc- r , y -mrx z - c
ri /*2 ///(/*i+ r2) = ~2c .. (4)
The locus of PPf is obtained by eliminating i\ and r%between
(3) and (4). Hence proceeding as in Ex, 15 above and putting the
values of rx and r2 from (7) of Ex. 15 in (3), the required locus is
given by
c (y+mx) c (y -m x ) k
m (z+c) ’ m ( z - c ) ~ i-\-m2
or c2 (l+ m 2) (y2—m2x2)= km 2 (z2- c 2).
Ex. 17. Find the locus o f a straight line that intersects two
given lines and makes a right angle with one of them,
Sol. Let the equations of the two given lines be
y= m x, z= e (I) and y = mx, z = —c. ...(2)
The equations of any line intersecting the lines (I) and (2) are
given by the planes
Skew Umes 231
A _ Z ± ^ l ^ i J ^ + Z ± « E l —2mC » 0
I z—e z+ c J I z c T z+c J
or A (2cy -Imzx) - B m (2jz —2mex)-2ntC (z* c*)=0
or >1 (<cjp—wizz) - JtM (fz —mcr)—mC (z*—c*)=0.
Ex. 19. Find the sufaee generated by a Une winchintersects
the Umesy=z=a; x+3z=a, y+z=a emdUyaraIIel to Aeplaee
jr + j= 0 .
SoL The equations of tike give· lioes are
j —e = 0 . z - o = 0 —(I)
I + 3z—o = 0 , y+z- o = 0 . -(2 )
The equations of any planes through the lines (I) and (2) are
( j - e ) —Jt1 ( z - e ) = 0 or j —J^z—o + o ^ = 0 —(3)
and (x + 3 z—o)—Jt (y+z—o )= 0
or z-JizF+ (3—ii) z —o+oJii=0. ---(4)
Any line intersecting the lines (I) and (2) is givenby the
intersection of the planes (3) and (4). Let Jt, p, » be its <Lr.*s. then
OJt+l.ji—V * = 0 , l-J -J ^ + (3 —Jt1) v= 0.
____A_______μ ____
·· 3 J .-J .J . -A 1 ~ - I '
Now the line with cLr.’s λ, μ, * is parallel to the plane
x + j = 0 Le., this line is perpendicular to the nom al to the plane
X-J-J=O whose d.r.’s are I, I. 0. So we have
l.(3 -J ^ -4 A H -M -A J + 0 L (-l)= 0
or 3 -A 1-Jt1-A 1A1=O. ...(5)
The required locus o f the line ir obtained by cfiaiaating
and J^ between (3), (4) and (S) hence is given by
3 I - · x + 3 z-a y—a x + 3 z - « Q
z —o y+z-a z—a y+z—a
or 3 (y + z - a ) (z—· ) —( j —o) (y+z-a)- (z—a) (x+3z-a)
(j-a) (x + 3 z -o )= 0
aaa
—j z - j*+2oz -x z + 2 e x —Xj=O
yz+j*+xz+xy=2az+2ax
(y+z) (x+y)=2m (x+z).
E r. 2 ·. Fimdthe Ioeas ofthe Umeawhkh smoteparallel to the
xy=r*, z = 0 ; j*—4cz, z = 0 .
SaL Let the equations o f any line be
(x -« )//= C r—Λ /ο * = (ζ-7 )/β . ...(IJ
According to the problem, the line (I) is parallel to the zx-
plane Le. the plane y = 0 , so that we have
Skew Lmt' 233
/.0φιιι.1+/ι.0=0 or IH==O.
Putting m =0 in (I), the line becomes
(x -α )//= 0 > -β )1 0 = (ζ -γ)/η . (2)
Now the line (2) meets Xy=C19 z=>09 so that we have
( x - a )//= ( ^ - f ) /0 = ( 0 - y ) /n
or ( x -(X)H=-Jfn9γ = β or x —a—(Ijfn)9 y= fi.
Putting the values of x and y in Xjr=C29 we have
(-?)— K > M ·
Agvn the line (2) meets y*=4cz, x = 0 , so that we have
(0—<*)//= O' -0)/O=(z—y)/n
or -* Ι Ι= (ζ —γ)Ιη, y = fi or z—y-(η*//), γ = β .
Pnttiag the values of y and z in y2=4cz, we have
> - f c ( r - " ) o r ( r - £ ) l- f . ...(4)
Muhiplying (3) and (4)9 to eliminate I and n9 we get
K k M -S U = '
or (a/? —c1) (4cy—/?*)™4ca/?y
or 4cafiy —β* (αβ-c*) -4c*y=4«/?y
or /P (αβ—c*)+4c*y=0.
Now the locus of the line (I) is the locus of the point (a, β, y)
and is given by
y* (xy—γ*)+4<^ζ= 0.
8
Change of Axes
§ I. Transformation of co-ordinates.
In th e study of analytical geometry, the co-ordinates of a
point, the equations of a curve or the equation of a surface are
always considered with regard to a fixed origin and a set of co
ordinate axes. The co-ordinates of a point, the equations of a
curve or the equation of a surface change when cither the origin
is changed or the directions of axes are changed or both are
changed. The process of changing the co-ordinates o f a point, the
equations of a curve or the equation o f a surface is said to be the
transformation of co-ordinates. This process of transformation of
co-ordinates will be of great advantage to tackle most of the
problems.
§ 2. Change of origin (Translation of axes).
To change the origin o f
L <
co-ordinates to another point
(a, /?, γ ) 9 whereas the directions
of axes remain unaltered. /V /y
Let O be the origin of co
ordinates and OX, OYi OZ be
the original co-ordinate axes L O1
Let O' be the new origin and 4 \
(a, β 9 yjits co-ordinates referred O
X
to theoriginal axes. / Y
Draw three lines OrX ', O 'Y \
and OtZ r through Of parallel to and in the same directions as OX9
OY and OZ respectively. Let P be a .point in the space whose
co-ordinates are (x9y 9 z) referred to the original axes OX9OY9OZ.
Again suppose that the co-ordinates of the same point P referred
to the new axes OfX f9 OfYt9 OfZ f are (*', y' , z').
Change o f Axes 235
X 1 y ’ Zt
X U U U
y /M1 m , Wj
Z ” 1 n2 «·
Change o f Axes 237
Let us suppose
D= k Wl1 W1 I
h m* »* j.
I
W3 W3 !
We have
The co-factor of Z1=(Wi2W3-W 1Wi), etc.
Hence the relations (5) show that e a c h constituent in D
= ± (its co-factor).
(C) To show that D= ± I .
We have Dt= h Wi Wl I I1 W1 »1
k W2 W2 X it W1 «I
U W3 W3 h W2 »«
or />*= Six' Shk £ / 1/3
Z hk ΣΙ* Skk
Zkk w1 Skt
OT Df= I 0 0
axes to another with the same origin, the expressions a + b+c aiul
a*+v*+tv* remain unaltered.
SaL' The given expression is
ax* T by*+ cz*+2fvz+ 2%zx+ 2hxy + 2ux+ 2vy+ 2wz+ d. ...II)
Let the given set of axes OX, OX, OZ be transformed to the
new set of axes OX', O Y', OZ'. Let d.c.’s of OX', O Y', OZ' be
I u m l t Ui; lt,m t,n t ; /,, m,, /1* referred to Ο Χ ,Ο Υ ,Ο Ζ . Now
replacing x by IiZ etc. [See § 3 relations (I)), the ex
pression (I) transforms into
o (/ix+/*y+/*z)*+& (mtx + /Wjj>-f.injz)2+ c (/I1X-I-Wi V+/¾-)*
+ 2 / (n^x+ m ^+ m jz) (ntx +thy+ /V )
■LigOhX+iv+/^z)( Ιχχ+ Iiy+ I,z) -{-2AiZ1X+ /,y+ UzKmlX+ m*y+ »W*z)
+ 2 a ( /jx - f /^ + /^ ) + 2 f (m1x+/»it y+e%z)
+ 2 ^(/1^+ /1^+ /1,2)+ //. -.(2)
Suppose the expression (2) is written as
Ax*+By*+Cz*+ 2Fyz+ 2Gzx+2Hxy+2Ux+2 V yJtlW z+ D
where A, B7 C7 F7 G, H7 U7 V7W and D are clearly given by
A -O l1* + Am,*+CTt1* + 2//11,/1,+2gn,/,+ 2U1Ot1,
B=Ol1t A-Amt*+ent* + Vm 1Iii + 2 g n ,/,+ Ihljn1,
C= e/|*+ Am3*+ c'/ij*+ + 2gW,/* + IM jn1.
¢/= a /,+ Vm1+MW1, K= a/.+vm ,+»'/»,,
H'=a/,+vm 3+M7»t> D—d.
:. A + B + C = a I l*+ b Σ ΐΒ ,+ ί Γ /r,*+ 2/2 /H1H1
+ ¾ J H1/, +2// Σ Uml
or ^ + e + C = n + A - |c
[using relations (3) and (4), § 4]
and U*-\- Vs+ IK*—(u/,+vm1+M’/r1)*+(a/,+vm,+M’H2)*
+ (u /9+vm, + Hw,»*
=«* Z /,*+v* Σ m ,*+iv* Σ n,2+2av Σ Ijn t
+ 2 ku 2' h1/,+ 2 vm· Σ m jtt
= m* + v* + h’* fusing relations (3) and (4) of § 4],
Hence the expressions α + Α + c and η*+ν* + Μ^ remain
unaltered.
The Sphere
We hare £7»=| CP |= r .
I ( x - a ) i+ ( y - b ) j+ (z —c) k | = r
or y/{lx - a f + { y b)*+(z -c)x}= r
or (x -a f+ (y -b )* + (z-c )* = r * . ...(I)
The equation (I) is the required equation of a sphere whose
centre is (a, b, c) and radius r.
Particalar Case. If the centre of a sphere is origin and
radius is r, then we have : OP I = r,
where O is the centre and OP ( = r) is the radius.
Hence we have
! x i+ y i+ zk ; = r Le. y/( x i + yt + zt)-=r
or x*+y* T-Zi --=rs ...(2 )
which is the rcquired’equation.
The Sphere 243
* i a+ .V i * + Z i * Xi yi Zi I
'" ·~·'Α v Q 3
***+ Λ *+ 2 ** *1 y% Zf i = 0. -.(8)
*» *+ Λ *+ *» * y* Zs I
* 4 yi Zi I
Remark. The equation ( 8 ) has *a determinant of 5th order
and hence in numerical problems, the evaluation of 5th order
determinant will require much labour and therefore, it is advi
sable, to solve four equations given by (7) for U9 v, w and d9 then
the required equation of t,he sphere is obtained by substituting
the values of t/, v, w and d in equation (6 ).
(E) Diameter form. To find the equation of a sphere on the
join o f two given points A U 1, y l9 Z1) and B (x2>y t%z2) as diameter.
(Berahampur 1981 (S); Indore 79; Lucknow 78; M. Dayanand 79)
Let P with co-ordinates (x, v, z)
be a current point on the surface of a
sphere drawn on the line joining the
given points
A Ui> Λ. Z1) and B (x2, y 2, z2)
as diameter. Then clearly the lincs^P
and BP are perpendicular to each other.
The Sphere 245
ron. Hence, solving (I), (2)f (3). we get the vertex O (0, 0, 0) and
solving (I), (2), (4), we get the vertex (o, b, —c).
Similarly solving other two triplets of the equations of the
planes the remaining two vertices are (o, - b9e) and (- a, b9 c).
Hencetherequired sphere passes through the four points
(0, 0, 0), (a , 6,— c), (a, —A, c) and ( - o, b9c).
Let the equation of the sphere be
&-\-yz+ zt +2ux+2vy+2wz+d=0. ·* (5)
If (5) passes through (0, 0, 0), we have d=.0. Hence (5) be
comes
x~+y*+z*+2ux+2vy+2wz=0. .(6)
If (6) passes through (a, b, —c), (a, —b9 c) and (--a9 b9 c),
we have
a2 F6*+c*+2ua+2v/>—2ivc=0, ...{V
a*+b*+c2+2ua -2vb+2\vc—09 - .(8)
and a*+b2 ±c2 -2ua+2vb+2wc=0. ...(9)
Adding (7) and (8),
2 (0^ + ^ + ^ ) + 41/0 =0 or 2u= —(o*+6*4-c2).o.
Similarly, we have
2v= —(o*+6*+c2;/6. 2w= —(o2+ ^ + ^ )//* .
Substituting the values of //. v, w and J in (5), the equation of
the required sphere is given by
JcM-J*2+ z2—(l a) (a*-J b2+c2) jc— (1/6) (οΜ-^M-c2) v
— (1/c) (uP+A^+c*) z—0
a-+ + 2+22 a* z
ΟΓ a2+b2+c- a b c
Ex. 8. F/W /Ac equation o f the sphere which passes through the
points (I, - 3, 4), (I, —5, 2), (I, —3, 0) and whose centre lies on
the plane x + y + z= 0 . [Jodhpur 1978]
Sol. Let the equation of the sphere be given by
jc2+ y* + z 2+ 2ux+2 vy + 2hz + d ^ 0. ... ( I)
If (I) passes through (I, —3, 4), then we have
I + 9 + l6+2w—6 v + 8 h + J —0. ...(2)
If (I) passes through (I, —5, 2), then we have
I + 25+ 4+ 21/- IOv+4 * + J=O. .(3)
If (I) passes through ( I, - 3, 0), then we have
1 + 9 + 0 + 2 ¾ /-6 r + 0 + Je=O. .(4)
The co-ordinates of the centre of the sphere (I) are ( —i/. —v,
—w) and if it lies on the plane jc+ j *+ j - 0, then we have
—u-—v —iv==0. (5)
250 Analytical Geometry 3-D
or w = l (54-33). ...(7)
Now it is given that the sphere (I) touches the plane
2x+2y - z = l l . ...(8)
·'· The length of perpendicular from the centre ( —u, —v,
—w) to the plane (8)—the radius of the sphere (I)
or 2 ( - « ) + ? . ( —νϊ --( -w) —11
= y/(ui + vi+wi -d).
V{(2}* + (2)* + ( - I)2}
Squaring, ( - 2u--2v + w*~l l)a= 9 (u*+v2+ w *- d). ...(9)
Putting the values of u, v, w from (5), (6), (7), we get
{d+ I- 3 4 + 13 + 1 (54—33)—J I}2= 9 {J (d+ 1.2+ * ( i d - 13)·
+ (1 /1 6 ) (5 4 — 3 3 )*— 4 }
or (1/16) {- 3</—21)2=(9/!6) {4 (4 + :)2+ 4 (34--13)2
+ ( 5 4 - 3 3 ) * - 164}
or (4+7)*=4 (42+ 2 4 + lj+ 4 (942 -784,-169)
+(ZSdi- H O d + 1 0 8 9 )-\6d
or 644*-6614+1720=0 or 842- 834+215=0
or 842—404 434+215=-0 or (4 -5 ) (8 4 -4 3 )= 0
or 4=5,43/8.
When 4= 5, the equations (5). (6) and (7) yield « = 3, v= I.
»v= - 2 .
Substituting the values of u, v, tv and 4 in (I), the equation of
the required sphere is
Xi + y 2+ Zi —6x+2y ■ 4 z+ 5 = 0 .
Similarly when 4=43/8, find the corresponding equation of
the sphere as above.
Ex. 15. A sphere of constant radius r passes through the origin
O and cuts the axes in A, B, C. Find the locus o f the foot of the
perpendicular from O to the plane ABC.
[Allahabad 1975, 79; Kanpur 81; Meerut 73, 83 S, 87, 89, 90
Rohilkhand 78. 79, 81: Nagpur 87]
Sol Let (a. 0. Ot, (0, b, 0) and (0, 0, c) be the co-ordinates
of the points A, B and C respectively. The equation of the sphere
OABC is [See Ex. 6 (a) above]
x2+ y2+ z2 ax by - c;= 0. - (1)
C learlytheinterceptsonthe co-ordinate axes are a, b, c and
so the equation of the plane ABC is
x/a+ ylb+ zjc= I. ...(2)
The radius r of the sphere (I) is given by
V'{(ia/ + (i*)2+ !$ c)2i = r or ai +bi +ci =4r-. ...(3)
254 A n a ly tic a l G e o m etry 3-D
Now the equations of the line through the origin and perpen
dicular to the plane (2) are
x —0 y - 0 z - 0
\la ~ \jb ~ ijc A’ (say)‘ .(4)
The co ordinates of any point on the line (4) are (X/a, λ/ft,
X/c). If this is the foot, say (α, β, γ), of the perpendicular from
the origin to the plane (2), then we have
Xfa=OL9 X /ft=0, X / c = y ο τ α = λ / α, ft=A//ϊ, C=Xjy. ...(5)
Since the foot of the perpendicular lies on the plane (2), we
have <x/a+$/b+ylc=\. .(6)
Now putting the values of a, ft, c from (5) in (3) and (6),
we get
λ2 ( l/α2+ 1//?2+ 1 Iy2i=Ar2 .. (7)
and (l/A)(a2+ £ 2+ y 2) = l . -...(8)
Eliminating λ between (7) and (8), we have
( a 2+ / ? 2+ ? 2) 2 ( o r 2+ / M - | - y - 2) = 4 r 2.
.·. The locus of the foot of the perpendicular ία, β, y) is
(x2+ y2+ z2)2 (x~2+y~2+z~2)= 4r2. Ans.
Ex. 16. Find the equation o f the sphere which passes through
the points (1 ,0 ,0 ), (0, 1,0) and (0,0, I) and has its radius as
small as possible.
Sol. Let the equation of the sphere be given by
X2 + y2+ z 2+ 2 u x + 2 v y + 2 w z + A=0. ...(I)
If(I) passes through the three given points (I, 0, 0), (0, I, 0)
and (0, 0, I), we have
I +2u+X =-0, I 4-2ν+λ=0, I-f-2w—λ= 0.
Solving, we have
U = V = W= -h ίλ-hl). ...(2)
Let r be the radius of the sphere (I), so that we have
r2=u2+ V2+ w2- Χ= μ (say)
or p = r2=± (λ-f I)2- λ.
[Putting the values from (2)]
If the radius r is minimum, then r2 i.e. μ is minimum. The
necessary condition for μ to be minimum is
dpfdX = 0 or (3/4).2 (λ+ l ) - 1 - 0 or A = - i
Also ^ = ' T = +ve at A= ~έ·
Hence μ is minimum at A = - J .
Putting A=*—I in (2), we have u=v=w =- —
The Sphere 255
or 2 (0 ^+ /^ + ^ )-2 * ! (α+β>+7)+(3α*—fc)=0.
Λ The locus of P (α, β %γ) is
2 (xH-J8+**)—2o (x+ y+ z)+ (3 a * -k)= 0
which is clearly the equation of a sphere.
Ex. 19. OA9 OB9 OC are three mutually perpendicular lines
through the origin and their direction cosines are I19 ml9 nx ; /* ml9 nt \
l%9 mlf /I1. IfO A=O 9 O B= b9 OC=C9 prove that the equation o f
the sphere OABC is
x*+^*+r2—x (a^+bU+cli) -y (am ^bm ^+ cm ^
z ( Ai i 1 + Α · ? * + ~ 0 .
[ Y I f+ m S + n f = ^
Similarly substituting the co-ordinates of B and C in (I), we
get
b + 2 m/ , + 2 vwr, + 2 m7i , -= 0 , -(3 )
and c i- 2i /4 i- 2vm%+ 2m//3=0. - (4)
The Sphere 257
Now I1, Ii, Ia ; mx, mt, mt ; nt, nt, na are the d.c.’s of OX, OY.
OZ respectively with reference to the three mutually perpendicular
given lines.
Λ Λ*+ /»*+ /**= W1*+ m**+ m»*= «i*+ «**+ = I.
and I1My^mlatrtt“\~ΙζΜ$~
Multiplying (2), (3) and (4) by I1, I9 and I9 respectively,
adding and using the above relations, we get
οΙχ-\· blf\-cl9-\· Iu=O or 2u = —(ali~\~ bla~\~da) ■
Similarly multiplying (2), (3) and (4) by mu m% and mlt and
then by nu nt and n3> and proceeding as above, we get
2v— — (ami+bnta+cma), Iw = — (e » i+ b « i+ c n *).
" « / ( I* _ V2
Ex. 2. FiW the radius and centre of the circle o f intersection of
the sphere x* +y*+ z*—2y - 4z -= 11 and the plane x + 2y + 2z = 15.
/Agra 1976; BundeIkhand 78; Madras 74; Meerut 84 S, 85 (S)
Sol. The equations of the giver, ciicle are
x * + j> * + z * -2 y ~ 4 z -l!= 0 . ...(I)
aod x + 2 y + 2 z —15=0. ...(2)
See figure of § 3 above. The centre of the sphere (I) is the
point O (0, I, 2) and its radius
O/*= t/{(0)*+( I )*+ (2)*- ( —I >)}=4-
Now OC—t he length of the perpendicular from the centre
O (0. I, 2) to the plane (2)
0 4 - ? . J+ 2 7—I s
3 (numerically).
A T heradiusofthecircle
--=CP= V iO P i - OCi) = V ( I *·-9) -- V7 ·
To find the co ordinates of the centre C of the circle. The line
OC being perpendicular to the plane (2), is parallel to the normal
of the plane (2) and hence d.r.’' vi the line OC are I, 2 2.
Λ The equations of the lire OC the equations of the
line passing through O (0, I. 2) and having d.r.’s I, 2, I arc
x —0 y - z-2
r (say).
1 "~'~2 2 -(3 )
Any point of (3) is (r. 2r j-l, 2r+ i) If this is the foot of
the perpendicular (i.e. the centre C of the circle) then it must lie
on the plane (2), so that we have
r + 2 (2 r+ l) + 2 (2 r+ 2 )—15—0 c r r = J .
Putting the value of r, the co-ordinates of the centre C of the
circle are (I, 3,-4)..
Ex. 3. I f r is the radius of ike circle
x* A-Pt -^ zi -Trlux y 2 v y - \- 2 * : - \- d -0, I x + m y + n z '= 0,
prove that
(r*+J) (/* -j- niJ + rt*) =.(wu> —,,r)1+ (iiu— ^ -f (/r—m uf.
(Rohiikhttnd 1982; Delhi 75/
The Sphete 261
ϊ )· num" l“ "y'
Λ The radius of the circle
= C P = V (O P t - O C t)
1=3Vt I —I) =*V d ) ■*I·
This proves the 1st part of the problem.
For the second part proceed exactly as in Ex. 11 above. The
equation of the required sphere is ^iven b>
166 Analytical Geometry 3-D
-I
or Γ(ϊ*+ * + ? ) — 5 or r= 2 (a-*+b-*+c-*)
. a , r ^ja I _a* (a- , +b-*+<r*)—I
” 2 a = 2 2α (a-*+b-»+c-*) “ 2α (a-*+tr*+c-*)
_ a* (6-*+c-*) _ a (fe-«+c-«)
2a (a-»+l»-*+c-*)“ 2lo-*+fr-*+c-V ‘
Similarly - + —=> ^ (<τ*+α~*)
y 2 b 2 (a-*+6-*+c-*)
and c (a- *+&-*.)
T + · c 2-(e-*+ft-»+c-*)
Putting these values in (4), the co-ordinates of the centre of
the circle are given by
I g (b-*+c-*) b (c_,-fa_i) c (a - , -fb_2)
\2 (fl-*+h-»+c-»)' !(a -'+ b ^ + c -* ) * I (a-*+h~*+c-> )
To find the diameter of the circle. Let r be the radius of the
circle and R the radius of the sphere. If p be the length of the
perpendicular from the centre of the sphere to the plane (2), then
r*=R*—p*. ...(5)
Now /{«radius of the sphere (l)=$-v/(a ,+ £ * + c*)
and /!«the length of the perpendicular from ($a, $c) Le.,
the centre of the sphere (I) to the plane (2)
________________________________ I________
Vtl !/«>■+ (I /W+d/*)*} 2V {o-*+ b-*+ c-·)
Putting the values of R and p in (5), we get
i _ a 2+ i 2-fc2 I
' ~ 4 4 (a-*+hr··+ c-*)
a*^2C1 ___
or 4r2= a 2-f b*-f c2—
h*c*+c*a*+a*/>*
(b*+c*) (c»+a*) (a'+b*)
b»c*+c«a* +a*b%
The required diam eter«2r
If the plane (2) touches the given sphere (I), then the length
of the perpendicular from the centre ( —u, —v, —h’) of the sphere
(I) to the plane (2 )= the radius ■v/(a*+v*+*·'·—</)of the sphere (I)
l ( —u)+m (.—»)+ n ( - w ) - p
Le., y/(u*+ V2+ a»·—d ).
V (/x+»n*+n*)
Squaring, the required condition is given by
(lu+mv+nw+p)*=^*+v*-\-w*—d)
Corollary. The condition that the plane lx+m y+nz=p
touches the sphere x2+ j*+ z*= e* is given by
p*=e* (Proceed as in § 7 (E) above].
SOLVED EXAMPLES (C)
Ex. I. (a). Find the equation of the sphere described on the line
joining A (2, —1, 4) and B (—2, 2, —2) as diameter and find also
the equation of the tangent plane at A.
Sol. The equation of the sphere described onl the line
joining the points A (2, —1, 4) and B (—2, 2, —2) as diameter
is given by
( * - 2 ) {*—(—2)}+{y—(—1)} O’—2)+ (z—4) ( z - ( - 2 ) } = 0
or *, +.)’, +z*—y —2z—14=0. Ans.
The equationofthe tangent plane at A (2, —1,4) of the
above sphere is given by
* 2 + ^ .( - 1 ) + 2 .4 - * O'- Ι ) —I .(2 + 4 )-1 4 = 0 (See § 7 (A)]
or 4 x —3^ + 62—3 5 = 0 a u b .
is 6 x -3 .y -2 3 + A (32+2)=0
or 6x—3.y+3Az+(2A—23)=0. ...(2)
If the plane (2) is the tangent plane to the sphere (I), then
the length of the perpendicular from the centre (—I, 2, —3) of
the sphere (I) to the plane (2 )= the radius of the sphere (i)
i e 6 ( - 1 ) —3.2+3A (— 3)+2A —23 m \w , / ι _τ»\
i e · --------V{(6)*+(3)*+ ( 3A)*T-------- ν { ( Ι ) · + ( - 2 ) + ( 3 ) - ( - 7 ) }
or - 7 A - 3 5 = v /(21)v/(45+9A»).
Squaring and simplifying, we get 2A*—7A—4 = 0
or 2A*—8A+A—4 = 0 or A=4, —$.
Putting the values of Ain (2), the required equations of the
tangent planes are given by
2x —y +42—5=0 and 4x—2y—z—16=0.
Ex. 2. (b). Find the equations o f the tangent planes to the sphere
**+y*+z*+6x—2 z + l= 0 which pass through the line
3 (16—x)=3z«=»2y+30. (Punjab 1977)
Sol. The equations of the given line may be re-written as
3 (16—x )—3z=0=2_y—3z+30.
Now proceed as in Ex. 2 (a) above, the equations of the tan
gent planes are given by
x + 2 y —2z+ 14=0 and 2x+ 2y—z= 2.
Ex. 3. Find the’equatlons of the tangent planes to the sphere
x*-\-yi +z* ==24 which are parallel to the plane 2 x + y —z=*0.
Sol. The equation of the given sphere is
*, +3’*+z, =24. ...(I)
Let the equation of the tangent plane parallel to the plane
2x-f y —z - 0 be 2x+.y—z+A=0. . (2)
If the plane (3) touches the sphere (I), then the length of the
perpendicular from the centre (0, 0, 0) of the sphere (I) to the
plane (2)= ± th e radius of the sphere (2) 1
. __ 2 .0 + 0 —0+A __ .
■’ V{(2)* + ( !)*+( —I )*} - ± V U '
or A= ± V (2 4 ) . v '6 = ± ( 2 .
Putting the values of A in (2), the equations of the required
tangent planes are given by
2 x + ^ -z ± 1 2 = 0 .
Ex. 4. (a). Show that the plane 2x—2_y+z+12=0 touches the
sphere x t + yt + zt —2 x —4y+ 2z—3 = 0 and find the point of contact.
(Avadb 1981; Agra 82; Berahmpor 81; Gaohati 77;
The Sphere 277
^ + ^ + ^ + 2 ^ -^ 4 1 + ^ (2 ^ -^ -2 2 -4 )= 0 . ...(3)
If (3) passes through the point (I, —I, 0), then we have
1+ 1 + 0 + 2 + 6 + 1 +A ( 2 + 1 - 0 - 4 ) = 0 or A = Il.
Putting the value of λ in (3), 'the equation of the required
sphere is given by _\ ' ^ '
* * + ^ + z * + 2 4 x - l7 V - 2 2 r - 4 3 = 0 .
Ex. S (b). Prove that tfte equation of the sphere which touches
4 (x*+y*+2*)+1O x - 25y - 2z= 0 at (I, 2, - 2 ) and passes through
the point (—1, 0 ,0 ) is \'Λ \
x*+y*+z?+'^x—6 y + l= 0 . (Meerut 1978)
Sol. Proceed as in Ex. 5(a) above.
Ex. 5 (c). Find the equation o f the sphere which touches the
sphere x*+y, + r* + 2 x —6>>+l=0 at (I, 2, —2) and passes through
the oHgin.
Sol. Proceed upto equation (3) of Ex. 5 (a) above.
If (3) passes through (0, 0, 0), then we have
0 + 0 + 0 + 0 - 0 + 1+A ( 0 - 0 - 0 - 4 ) = 0 oj"VA= 1/4.
Putting the value of A in (3), the equation of the required
sphere is given by n^
(λ', + Ρ , + ζ* + 2 χ —6 y + l) + i.( 2 x —y —2z—4)=0
or 4 (χ’ + ^ + ς^ + ΙΟ χ —25y—22=0.
Ex. 6. Find the equations o f the spheres through the circle
**+ ,^ + Z i =aI. 2x+ 4y+ 5z= 6
and touching the plane 2 = 0. (Gorakhpur 1974; Meerut 85S)
Sol. The equation of any sphere through the given circle -is
ju*+j,*+zs _ l + a (2 x + 4 ^+ 5 2 —6)=0
or xi +y*+2*+2Ax+4A^+5Ar—(1+6A)=0. ...(I)
If the sphere (I) touches the plane 2 = 0, then the length of
the perpendicular from the centre ( —A, —2A, —J.5A) of the
sphere (I) to the plane 2= 0 = + th e radius Of the sphere (I)
ie - V O iT “ ± * /{ λ*+ 4 λ ί+ ^ ’ +6λ)}
25A1 45A*
Squaring = - ^ —+6A+1 or 5A*+6A + 1= 0
or 5A*+5A+A+1=0 or (5A+1) (A +1)=0 or A =—1/5, - I
Putting the values of A in (I), the equations of the required
spheres are given by
5 (x*-+y2+ 2*)—2x—4y—5 z + l= 0
and x*+3'*+z*—2x—4>’—5z+ 5=0.
The Sphere 279
Ex. 7 (a). Find the equations o f the spheres which pass through
the circle **+>>* +z*—2 x + 2 y + 4 z -3 = 0 , 2x + .y + z= 4 and touch
the plane 3x+ 4y—14=0.
Sol. The equation'of any sphere through the given circle is
x * + y * + z* -2 x + 2 y + 4 z-3 + X { 2 x + y + t-4 )= 0
or xt + ^ , + 2 * -2 x (l-A )+ y (2 + A )+ * (4 + A )-(3 + 4 A )« 0 ....(l)
If the sphere (I) touches the plane
3x + 4 y —14=0; ·- -(2 )
then the length of the perpendicular from the centre
(I-A , -(2+A )/2. —(4+A)/2)
to the plane (2) = + th e radius of the sphere (I)
. 3 .f l- A ) - 4 .j( 2 + A > —14
- ± V { ( l- A ) * + i (2+A)*+i (4+A)*+3+4A)
or -<A+3)=±*v'(6A*+20A+36).
Squaring and simplifying. A1—2A=0 or A=0, 2.
Putting the values of A in (I), the equations of the required
spheres are **+>*+**—2x+2y+ 4z—3= 0
and χ*+^*+ζ*+2χ+4>»+6ζ-ΙΙ=·0.
Ex 7 (b). Find the equations o f the spheres which pass through
the circle x1+y*+ z, = 5 , 2*+3,y+z=3 and touch the plane
.N 3 ^ + 4 ^ -1 5 = 0 . (Punjab 1982)
Sol. Proceed as id . βχ. 7 (a) above.
Ex. 8. Find the equations o f the tangent line to the circle
3x*+ 3y*+ 3z*—2x—3^—4z—22= 0, 3x+ 4y+ 5z—26=0
at the point (1,2, 3).
Sol. The tangent line to a circle at a point is the line of
intersection of the tangent plane to the sphere at the point and
the plane of the circle.
The equations Qfthe given circle are given by the sphere
**+>>»+z*-ix-.V-fz-*e*«0 -.(I)
and theplane 3 χ $ % ^ 5 ζ —26=0. ---(2)
The equation of the tangent plane to the sphere (I) at (I, 2, 3)
is x .l- h y .2 + z .3 - J ( x + l ) - J (.y+2)—S ( 2 + 3 ) - ^ = 0
or 4x+9.y+14z—64=0. ---(3)
Thusthe non-symmetrical equations of the required tangent
line are given by (2) and (3) together. To find its symmetrical
form, let /, m, n be its d.r.’s, so that we have
3/+4m -|O/i=0 and 4 /+ 9m+ 1 4 = 0 .
280 AnaJyticaI Geometry 3-D
Solving, wc get ,%
I __m _ n Z_ m _ n
—11_ 22*" —lT ° r T - 2 Γ
Λ The equations of the tangent line at (1, 2, 3) and having
d.r.’s 1 . - 2 , I are
(*—1 )/1 = (3 '-2)/(—2 )= (z—1)/1.
Ex. 9 (a). Ifthreemutually perpendicular chords of lengths
du dt, d3 be drawn through the point (xit y t, Zi) to the sphere
xi +y*+z*c*ai, prove that
dt* + da*+d3* = l2o*—8 (x1*+J’i*+2l1).
Sol. The equation of the given sphere is
x*+y*+z*=a*. ...(I)
Let /,, mlt Hj; /*, mit η» and /8, m3, M3 be the actual d.c.’s of
the three mutually perpendicular chords drawn through the same
point A(xit y u z,). The equations of the first chord are given by
(x -.x 1)//l = ( ^ - 7 ,)/Mii=(z-Zi)/n1= r (say). . (2)
Any point on (2) is (ZiM-Xlt m ,r+ yu Bir+Zi).
Substituting the co-ordinates of this point in (I), the points
of intersection of the chord (2) with the sphere (I) are given by
(Λγ+Χ ι ^ + Ο πιΓ-Η ',^-Η λιΓ+Ζι )*=^* ...(2)
or rs(Zi2+ IWi1-I-B1*)+ 2 r (/iXi+miyi+BiZi)
+ (*I*+>1*+ V - ύ*)= o
or r*+2r (ZiXi+fWiyi+BjZi)+ ^ - + ^ + - 2 ^ - 0 ^ = 0 . ...(3)
Let the chord (2) meet the sphere in the points P and Q so
that AP=rit AQ=rt where Ti and r8 are the roots of (3).
A dt= P Q = A P —AQ=Tt - T i
or </i*=(T1- T i)*=(rt + γ8)*=4 γιΓ8
or </i*={—2 (I1X1^ m ty 1 +Mi Zi )}*-4 (χι2++ι2+ ζχ2- α 2)
o r. </x*=4 (/i Xi +»Wi^ i +Mi Z1)* -4 ( x ^ + y ^ + z ^ -a * ). ...(4)
Similarly </**=4 (/3Xi+m8+i+w8zi)*-4 (Xi*+yi2+Zi*-a*) . (5)
and <Z8*=4 (/8Χι+Βΐ3^ ι+ η 3Ζι)*—4 (Xi*+,yi*+Zi*—a*). . (6)
Adding (4), (5) and (6) and using Zi*+/8*+/3*<=l etc.,
/itMi+ Z 2IW 8 +Z3Wi3=O etc., we get
<Zi *+</3*+ </3*=4 (Xi2++i*+Z12J - 1 2 (Xi2+ y i2+Z i*-fl2)
= 12e*—8 (Χι*+Λ*+Ζι2).
Ex. 9 (b). Show that the sum o f the squares of the intercepts
made by a given sphere on any three mutually perpendicular straight
lines through the fixed point is constant. \
Sol. Let the equation of the given sphere be x*+>*+ z*= ar
and the fixed point be (xx, y lt Z1).
The Sphere 281
or 2r·—16r+30=»0, or r2—8r+ 1 5 = 0 ,
or (r—3) (r—5 )« 0 or r= 3 , 5.
Substituting the values of r in turn in (I), the equations of
the two required spheres are given by
s* + ^ * + * * -6 x -6 y —6 z+ 1 8 « 0 f
and x 2+ y2+ z2—IOx —IO y -10z+50*=»0.
Ex. 14 (a). Find the equation o f the sphere inscribed in the
tetrahedron whose faces are
y=0, 0 and 2*+6y—3 z+ 9= 0.
Sol. The equation of the fourth plane is t2x+6y—3z+9«=»0.
This plane clearly meets the x 9y and zaxes in the —ve, —ve and
+ve directions. Hence the sphere touching the three co-ordinate
planes and the plane 2x+ 6y—3z+9*=0 lies in OXtYtZ octant and
so the centre of such a sphere is of the form (—r, —r, r) where r
is the radius of the sphere.
Since the sphere touches the plane
2 x + 6 y -3 z + 9= 0, ...(I)
therefore the length of the perpendicular from the centre ( —r, —r,
r) to the plane (1) = the radius r of the sphere
—2 r - 6r—3r+9
i.e. V (4+36+9) - « r
or —I lr + 9 = 7 r or r«=J.
Λ Thecentre ofthe required sphere is ( — $, — $, and
radius is £. Therefore its equation is given by
(* + *)f+(y + i)a+ (* --* )f- U ) e
or x 2+ y*+ z*+ x+ y—z + $ = 0 . Ans.
Ex. 14 (b). Find the equation of the sphere inscribed in the
tetrahedron who.se faces are
x = 0 , .v^-0, z - Oand 2 * + o y + 3 z—14—0.
Sol. Here the sphere lies in OXYZ octant. Proceeding
as in Ex. 14 (a) above, the equation of the required sphere is
given by
81 (*a+ y 2+ z 2) - 126 (x + y + z)+ 98 = 0.
Ex. 15. Prove (hat the centres of the spheres which touch the
lines y —mx, z=c; y = —mxt z=>—c lie upon the conicoid
mxy+cz (l+ //i2)= 0 .
(BundeKkhand 1979; Lncknow 74, 81; Kanpur 79; Meerut 89S)
Sol. Lu the equation of the sphere be
JC2+ > a + z 2+ 2 w x + 2v#K + 2 n ’z + J = i 0 .
The Sphere 285
the polar,plane of A (xlt j^, Z1) with respect to the sphere (I) is
given by
*i*+y\'·+ zi*+ 2«x,+ 2vyt + 2h»z, + d
= —{ (* -* i)(* i+ « 0 + 0 '-J'i) (y t+ v )+ (z -zt) (2|+W)}
or xx\+ yyi+ zzt+ u (x+*i)+v(y+yi)+»*’ (2 + 21) + ^ = ° - (6)
To flod the pole of a given plane. Let the pole of the given
plane be (X1, y u Z1). Write the equation of the polar plane of
(X itji, zr) with respect to the given sphere. Now compare the
given plane with this polar plane and then obtain X1, y r, Z1.
§ 10. The properties of the pole and the polar plane.
Property I. Salmon’s Theorem. The distances of two points
from the centre of a sphere are proportional to the distance o f each
from the polar plane o f the other.
Let the equation of the sphere with centre O (0,0, 0) be
xt + y t +z*=a*. ···(!)
Let P (xlf y u zt) and Q (x%, y t, z*) be the two points whose
polar planes with respect to the sphere (I) are
xxi+ y yi+ zzi-a * —<2)
add xxt+yya+zztesa*. —(3)
J^ow the distance of P from (3)
the distance of Q from (2)
(xtx*+ yiyt+ zrzt —a*)l ViXt9^ y t t +Ztt )
“ (** *i+ y ty i+ ZtZl - a*)l^(X1"jCyit + zx*)
OP
^V iX t'+ ya'+ z»*) ~OQ Proved.
Property II. ThepolarpIaneof apointP with respect to a
sphere Is perpendicular to the line joining the point P to the centre o f
the sphere.
Let the equation of the-sphere with centre O (0, 0, 0) be
x*+y*+z*z=a*. ...(I)
Let the co-ordinates of the point P be (X1, y r, 2i) and so the
equation of the point plane of P (X1, y u zr) w.r.t. the sphere (I) is
x x i+ y y i+ ZZ1^ a t. .(2 )
The direction ratios of the normal to the polar plane (2) are
Xi, y u zt. Also the d.r.’s of the line OP are X1-O , J11-O , Zj -O
i.e. X1, y r. zt. Hence the normal to the polar plane (2) is parallel
to the line OP i.e., OP is perpendicular to the polar plane (2).
Property III. I f the line joining the centre O o f the sphere and
a point P meets the polar Of P in the point Q, then
OP. OQ =»(radius)*.
288 Analytical Geometry 3-D
+(W^1-W a)*
or 2i/1w*+2 v1vi + 2 h’iWf=^Z1+</,, ...(3)
which is the required condition.
§ 13. Tooching spheres.
If the distance between the centres of the two spheres is equal
to the difference of their radii, then the two spheres touch
internally.
If the distance between the centres of the two spheres is equal
to the sum of their radii, then the two spheres touch externally.
SOLVED EXAMPLES (D)
Ex. I. Prove that the equation o f the polar plane of (xlf ^1, Z1)
with respect to the sphere a 1+;'*+Z1=A1 is
xxi+yyi+zzi'=** (Delhi 1974)
Sol. proceed as in § 9 above.
Ex. 2. Find the pole o f the plane lx+ m y+ n z~ p with respect
to the sphere x*+y2+z*=a*. (Gorakhpur 1976; Rajasthan 78)
Sol. The equation of the sphere is
^ + / + Z1= A1. ..-(I)
The equation of the plane is lx+ m y+ nz= p. .(2 )
Let the pole of the plane (2) w.r.t. the sphere (I) be (X1, yl9
Z1). The equation of the polar plane of (xlf yu *i) with respect to
the sphere (I) is **i+XVi+ZZ1=A1. - (3)
The planes (2) and (3) are the same, so comparing the coeffi
cients of like terms, we get
*1 yi ^ z l A1
A1/ i orn
or , zx= — ·
The pole of the plane (2) is (a*l/pf a*mlp, a*njp).
x —I y —2 z —3
Ex. 3. Show that the polar oj ~ ~ =»-^— with respect
to the sphere x*+y*+z*=9 is given by
x+ 2 y+ 3 z—9<=0, 2x+3y+4z*=0.
Sol. ThisprobIem isbased on § 11. The co-ordinates of
any point on the line
are (2r+ l» ir+ 2 , 4r+3).
Its polar plane w.r.t. the sphere x*+<y>^2 * = 9 is
The Sphere 291
(2 r+ l) * + (3 r+ 2 ) y + (4 r+ 3 ) z= 9
or (x+2j»+3z—9 )+ r (2*+3y+ 4z)=0.
This plane for all values of r passes through the line
x + 2 j'+ 3 z - 9 = 0 . 2 x + 3 y + 4 z= 0 .
Thus these are the equations of the polar line of the given
line.
Ex. 4. Prove that the polar plane o f any point on the line
jx = $ Cp - I ) = * U + 3) With respect to the sphere 0 + 0 + 0 = 1
passes through the line
(2 x + 3 )/l3 = (y —I)/(—3)=z/(—I). (Allahabad 1976)
Sol. This problem is based on § 11.
The co-ordinates of any point on the line
|x = $ O' —1)=·! 0 + 3 ) are (2r, 3 r + l, 4r—3).
Its polar plane w.r.t. the sphere 0 0 + 0 + 0 = 1 is
oc.2r+,y.(3r+l)+z.(4r—3)=1
or (y—3z—l)+ r (2x+ 3y+ 4r)= 0.
This plane for all values of r passes through the line
y —3z—1=0, 2oc+3.v+4z=0. .··(!)
The symmetrical form of the line (I) is
* + 3 /2 _ y -l_ z -0
13 ~ - 6 “ - 2
-2oc+3 y —I z 2x+3 y — I z
or
26 -6 -2 13 -3 - I
Ex. 5. Find the angle of intersection o f the spheres
x * + 3 ', + z* - 2 x - 4 > ' - 6 z + 1 0 = 0
and oc*+0+z*—6x—2 ^ + 2 z + 2 = 0 .
Sol. The equations of the given spheres are
* * + 0 + 0 - 2 o r - 4 j> - 6 z + 1 0 = 0 , ...(I)
and x*+y, +z*—6x—2 ^ + 2 z+ 2 = 0 . ...(2)
The centre of (I) is Ci (I, 2, 3), its radius
r ,= V ( l , + 2 * + 3 * -1 0 )= 2 .
The centre of (2) is C* (3, I, —1), its radius
= ^ + 0 + (-1 ^ -2 )= 3 .
Now CiCt=the distance between the centres C> and C»
={(3—l)*+ (l —2)*+( —I —3)*}= -v/(21).
Let Θ be the angle of intersection of the spheres (I) and (2),
then
292 Analytical Geometry 3-D
2
cos 9 _ r«*+ r,* - (C1C2)*_ (2)*+ (3 I*- 21 - 8
^ 2r,r, " 2.2.3 12 3
= } (taking the acute angle).
9=cos_1 (}).
Ex. 6. Show that the two spheres x* +y*+ z, + 6 y + 2 z + 8 = 0
and x*+y, +z*+6jc+8.y+4z+20=0 are orthogonal.
Sol. The equations of the two spheres are
x, + y*+ z, + 6 y + 2 z + 8 = 0 ···(!)
x, + 7*+ z* + 6 x + 8 ^+ 4 z+ 2 0 = 0 . ...(2)
For the sphere (I), we have t/,=0, v,=3, Wi = It </ι=·8, and
for the sphere (2), we have «,--=>3. v,=4, tv,=2, </,=20.
We know that the condition of orthogonality of two spheres
is [See § 12, equation (3)]
2«iW»+2v,Vj + 2 h',mv-</|+</,. ---(3)
Putting the values in (3), we have
2 .0 .3 + 2 .3 .4 + 2 .1 .2 = 8 + 2 0
nr 0 + 2 4 + 4 —28 which is true.
Hence the given spheres are orthogonal.
Ex. 7. Two points P and Q are conjugate with respect to a
sphere S; prove that the sphere on PQ as diameter cuts S ortho
gonally.
Sol. Let the equation of the sphere S be given by
S s x 1 + y * + z* + 2ux + 2vy+ 2 m»z + < /= 0 . - - - ( I)
Suppose the co-ordinates of the given points P and Q are
(xi. .V i. Ti) and L r , , y t, z f ) respectively.
T heequationofthe polar plane of P (Xtf yt, Zj ) w.r.t. the
sphere (I) is
* * ,+ v y i+ zz,+ t/ (x + x ,)+ v (y + y ,)+ iv (z+zO+</=0.
The point Q (xt, y t, z,) will lie on this plane, since P and Q
are conjugate points and hence, we have
*ix*+yiy»+z,z,+i< (*!+**>+v O't+y»)+»' t o —z,)+</=o
The equation of the sphere on PQ as diameter is
(X -^i) (x -x » ) + ( y - y i) (y -? * )+ (z -z i) (z-z,)*=*o
or x, +y*+z*—x (x i+ x ,)—y (y i+ y » )-z (zi+z,)
+(xiX,+yiy,+ZiZ,)=0. ---(3)
-. Now the sphere (3) will cut the sphere (I) orthogonally, if
-2 -2 _ 2 < h+ I*l.w
=</+(x,x,+yiy,+ziZi)
or xix»+yiy»+z,z,+u ( χ ι+ χ ,)+ ν (>i + y ,)+ w (zi+z,)+</=o
which is true by virtue of (2). Heace proved.
The Sphere 293
ϊ Γ ΐ ; ΐ Α'Η ·
The sphere S=O will cut the sphere A1S1-I-A4S2=O [i.e., the
sphere (3)} orthogonally if
(*¾¾*)+*')
_ j i / iM i I^ A 4 i
V A1 , A4' I
The Sphere 297
0Γ 2 ( / ( A 1 IZ j + A 2M 2) + 2 v ( λ ι V 1 + A j V i ) T 2 w
(A1+A2)+(/Z1A1+ J 2A2)
or A1 (2IO/1 + 2W I+ 2 IVW0 +A 2 (2uu2+2vv2+2ww2)
t^A1 (J+ d|)+ A 2 (J + d 2)
or Aj (J+ J|)+ A a ( J + J 2) -- A1 (J+ Ji)+ A 2 (^/+^/2)
fusing (I) and (2)]
which is true for all values of A1 and Aa. Hence the result.
Ex. 15. Prove that the tangent planes to the spheres
X2 + y 2+ Zt + 2ujjc + 2vjy + 2W1Z + J1^ 0
and X2 + y %+ z2H- 2i/2jc+2v2y + 2 w2z + J 2 0
at any common point are at right angles if
2t/iM2+ Zv1V2+ 2 W1w2= J 1+ J2. (Allahabad 1975^78)
Sol. By § 12, we know that at a common point the angle
between the tangent planes to the two spheres is defined as the
angle of intersection of the two spheres. As the angle between the
tangent planes is given to be a right angle, the given spheres will
cut at right angles (i.e. orthogonally). Hence this is the same
problem as § 12.
Ex. 16. Find the equation of the sphere which cuts orthogonally
each o f the four spheres
x 2+ y2+ 2%^ a 2+b2+c2, x2+y* + z2+2ax*=a%9
x2+ y2+ z2+ 2by fca, x2+ y%+ z2+2cz—c2.
Sol. Let the equation ol the required sptiete be
x2+ y2+ zt -{2u\ \ .lvy+ :wz+ct^O. ...(I)
The equations ol the given sphetes arc
x 2+ / - + z * - ( a 2+ b 2+ c * ) = 0 . ...(2)
x1+ > a+ z* + 2ίΐλ - a2= 0, ...(3)
x 2+ y 2+ z 2+ 2 b y ~ i > * ~ 0. ...(4)
and **Hr.)^ + ^ + 2 ^ - c2<=0. ...(5)
It the sphere ( !) cuts the sphere (2) orthogonally then apply
ing the condition ' 2ulu.1-\-2vxV%+ 2 w l \\2 ·--Ji + Ja\ we get
2u.0+2v.0+2w .U =J - (a2’\-o2+ c i )
or d = a 2+ b 2+ c \ >..(6)-
Similarly it the sphere (I; cuts the sp)ku> ii), (4) and (5)
orthogonally, wc respectively hai.«
lUQ*=*d-Q2 ...v. ; 2\b d -b* ...(«)
u ν’.
298 Analytical Geometry 3-D
Patting the value of d from (6) in (7), (8) and (9), we get
2u=(b*+c2)la, 2v=(c*+a*)/b, 2w—(a*+b%)/c.
Substituting the values of 2u, 2v, 2h>and d in (I), the equa
tion of the required sphere is given by
x*+y*+zr+{(bi +c1i)la} x+{(c*+a*)lb) y+{(a*+b*)lc) z
+ (a»+6*+c*)=0.
Ex. 17. The variable sphere x t + y*+ zt + 2ux+ 2vy+ 2w z+ l*= 0
always cuts the sphere 3x*+3y* + 3z*—6x+ lOy+z—8 = 0 orthogo
nally. Show that the point («, v, w) moves on a fix e d plane.
Sol. The equations of the given spheres are
x t + y*+ zt + 2 u x + 2 vy + 2 w z+ l= 0 ...(I)
and x H ^ + z * ^ * + j ? y + | z-* = 0 . (2)
The spheres (I) and (2) will intersect orthogonally if
2«.(—l)+2v.(5/3)+2H \(l/b)= I —8/3
or —όΜ+ΙΟν+Η'= —5 or 6u—IO v -w+5=>0.
The locus of the point (u, v, w) is
6x— 10y—z + 5 = 0
which is a fixed plane.
Ex. 18. Show that the spheres
x* +y*+r* =s25, x *+>* + 2*—I S x - 24y—40z+225=»0
touch and show that their point of contact is (9/5, 12/5, 4).
(Berahampur 1981S)
Sol. The equation of the first sphere is
x*+yi + zi —25. ...(I)
Its centre C1 is (0, 0, Oj and radius ri= 5 .
The equation of the second sphere is
■**+>’2+ 2 i —18x—24y —40z+225—0. ...(2)
Its centre Ct is (9, 12, 20) and radius
r*= V(9i +12»+208- 225)«=20.
The distance between the centres- CiCi
= V U9—0)»+ ( 12—O)2+ (20 - 0)* = V(625)= 25.
Also the sum of the radii = r,+ r* = 5 + 2 0 ^=25.
Thus the distance between the centres of the spheres (I) and
(2) is equal to the sum of their radii. Hence the given spheres
touch externally. (See § 13).
To find the point of contact. Let (xt, y lt Z1) be the point of
contact. The equations of the tangent planes at (xi, yu Z1) so the
spheres (I) and (2) respectively are
'The Sphere 299
Hence the d.r.’s of the line joining the centres* are Iz1- if2,
V1-V 2, W1- wi which are same as that of the r.orihal to the radical
plane.
Property II. The radical plane of two spheres passes through
their points of intersection.
Let the equations of the two given spheres be given by Si=O
and S2eaO. Theequation ofthe radical plane is given by S1- S 2=O.
Clearly the points which satisfy both S1-O and S2*=0 also satisfy
S1- S 2=O.
Hence the radical plane S1- S 2=O passes through the common
points of intersection of the spheres Si=O and S2-O .
§ 15 (C). The radical line (or radieal axis). (Avadh 1982)
Definition. The radicat planes o f three spheres taken two at
a time pass through a line which is said to be the radical line (or
radical axis) of the three spheres.
Let the equations of the three spheres in their standard form
be given by S1=O, S2=O and S3=O.
Taking two spheres at a time, the radical plane of
S1=O and S2=O is Si —S2=*0.
Similarly the radical plane of S2=O and S3=O is S2- S 3=O
and the radical plane of S3 =0 and Si=O is S3- S 1= 0.
These three radical planes clearly pass through the line
S1= S2= S 2
which is the equation of the radical line (or radical axis) of the
three given spheres.
§ 15 (D>. Radical Centre.
Definition. The radical lines of the four spheres taken three at
a time meet in a point which is said io be the radical centre o f the
four spheres.
Let the equations of the four spheres in their standard form
be given by S1=O, S2=O, S1=O, Si=O.
Iaking three spheres out of these four spheres 4C3 i.e. 4
radical liues will exist. The equations of these four radical lines
are given by
S1= S 2= S 3J S2=S3= S1J S3= S 3= S 1j S3= S 1= S 2.
These lour radical lines clearly intersect in a point, called the
radical centre, given by Si = S2=S 3==S3.
Clearly the radical centre will also lie on all the radical planes
of these four spheres taken two at a time.
§ 16. Coaxial system of spheres.
Definition. A system (or family) of spheres is called a co-axial
302 Analytical Geometry 3-D
system o f spheres If for all spheres any two o f them have the same
radical plane.
Remark. We have proved in property I of § I S (B) that the
line joining the centres of any two spheres is perpendicular to
their radical plane. In the case of co-axial system of spheres any
two spheres have the same radical plane, hence the centres of all
the spheres of a co-axial system always lie on a straight line.
Note. Some authors write co-axat for co-axial.
(A) To find the general equation of the system of co-axial
spheres.
Let 5,==0 and Si - 0 be the equations of two spheres. The
radical plane of these spheres is given by S i - S i = 0.
Now 5 ,+ μ ( S i - S i )-O ...(I)
represents a system of spheres for different values of μ and any
two members have the same radical plane. Let μχ and μ» be the
two values of μ and so two members of the system (I) are given
by S i+ μι (S1- S 1)=O and 5 ,+ μ , (5 ,—5,)= 0.
Subtracting, the radical plane of these two spheres is given
by (μ ι—μ») (S i—5 ,) - 0 or 5 , - 5 , = 0 ( V μιΦμ»].
Hence any two members of the system of spheres given by (I)
have the same radical plane. Hence (I) represents a co-axial
system of spheres.
Again (I) may be re-written as
(1+ μ ) 5 ,—μ 5 ,—0 or S i—{μ/(1 +μ)} 5, = 0.
T h isiso ftb efo rm 5 ,+ λ 5 ,= 0 ...(2)
The equation (2) is the general equation of a co-axial system
of spheres.
Similarly if 5 , - 0 be a sphere and P=O be a plane then
S+XP=0 ...(3)
represents a co-axial system of spheres.
(B) To find the simplified form of the equation of co-axial
spheres. (Punjab 1980; Madras 76)
Let us take the centres of all the spheres of the system to lie
on the x-axis so that the y and z co-ordinates of the centres of all
the spheres of the system are zero whereas the x-co-ordinate is
different for different spheres.
The equation of such a system of spheres is given by
**+y*+ i*+ 2kx+ </=»0. ...(I)
The Sphere 303
Sol. . Let the centre of the circle be (α, β, γ) and let it cut
intercepts p, q, r on the axes so that (ο, β, γ ) lies on the plane
x v z ,
— + - - + — =1
p~ q~ r •••(I)
u B y ,
—+ — + — = 1.
P q r - ( 2)
Since radius of the circle=*».
Distance of («, β, y) from (p, 0, 0 )= a
( —cr.+p)2+fi!+ f = a i
^ P = V id t - β2—? )+ *
Similarly ς=ΒΛ/( α*~γ»-α*)+ β
r= V io * —ot*-fi*)+y
. J _ _________ I U - V jd t -Pt - / 2)
P u + V (tP -β * —Ύ*)ί **—(β*--β2—'·*)
U - V ia t Pt - tY*)
~ (ο*+β2+ γ2) - α 2
c . ·, , I β —ν ( & — Yt - **)
Similarly3 q—=7ixt. +fii
. Λ +, ty ic~
* )-d .t
I__ y—Vfrta—a» - β2)
r ~ (χ*+β2+)*)—α*
Put these values in (2;
« { * - V(O2^ f i i - V i )) +β {β -V (O 2- Y t -O t))
+7 I r - V io t- a* - β*)) =(α*+β2+ γ* - a 2)
Λ Locus of (α, β, γ) is
χ V (o t —y 2—zt)+ y V i a*- ζ* —χ)+ ζ V (ο* - Xt - y 2)= a2
Exercises
Find the equation of the sphere which passes through the
origin and makes equal intercepts of unit length on the
axes.
Ans. V + y 2+ z2- χ - y —z = 0.
2. A point moves so that the ratio of its distances from two
fixed points is constant. Show that its locus is a sphere.
3. Find the centre and radius of the circle
xt + yt + 2t + x + y + z —4=0, x + y + z = 0 .
Ans. Centre is the point (0, 0, 0) and radius is 2.
4. Find the equation of the sphere through the circle
**+,y*+2* =9, x+3y+4z<=5
The Sphere 309
The Cylinder
§1. Cylinder.
(A) Definition. fKnnpnr 1979 ; Lucknow 77)
A cylinder is a surface generated by a moving straight line which
mares parallel to a fixed straight line and intersects a given curve or
touches a given surface.
The curve is called the. getting curve and the fixed straight
line is called the axis of the cylinder.
Any line on the surface of a cylinder is called its generator.
(B) To find the equation o f a cylinder whose generators are
parallel to the line x/l= ylm = zln and intersect the conic
ax*+2Jhxy+by*+2gx+2fy+c=Q9 z=0.
(Agra 1981 ; Garhwal 78S ; Kanpnr 77, 81; Rohilkhand 81)
The generators of the cylinder are given parallel to the line
xll= ylm = z/n. ...(I)
H encetheaxis of the cylinder is given by the equations (I);
'tLet P (xl9 yl9 Z 1) be a point on the cylinder, then the straight line
through the point P (xlt y l9 zx) and parallel to the axis (I) is a
generator and so its equations are
X - X 1 y - V 1 z -Z1
I m n . (2)
The equations of the conic are given to be
ax*+2hxy+ by*+2gx+2fy+c=09 z=0. ...(3)
The generator (2) meets the plane z = 0 in the point given by
( * - ¾ .* - ¾ 0 ).
Since the generator (2) meets the conic (3), hence the point
(Xl - I z lIn, y i—m zjn, 0) will satisfy the equations of the conic
given by (3), and so we have
The Cylinder 311
+ ^ ( * · - 'τ ) + ν ( Λ - ν - ) +£“ °
or a (Wjc1- Zzx)2+2A (Wjc1-Zz1) (Wy1-Wiz1)+ ^ (WV1-Twz1)2
+ 2gn (WJC1-Z z1) + 2/w (W^1-/WZ1)+w*c=0.
Λ The locus of P(JC1, yx, Z1) *·*· the required equation of the
cylinder is given by
a (nx /z)2+ 2A ( wjc - Zz) (wy - wiz) + b (wy - wiz)2
+ 2gw ( wjc - lz)+2fn (wy—wiz)+cw2= 0. ...(4)
Coro1Iary I. If the axis of the cylinder be taken as z-axis
whose d.c.’s are 0, 0, I, then putting Z= 0, wi=0, w*=*l in the equa
tion (4) above, the equation of the cylinder becomes
0Xa+2Ajcy+Ay2+2gje+2/y+c==O. *··(5)
Remark. The readers should recall that the equation (5) repre
sents a conic in two dimensional geometry whereas in three dimen
sional geometry the equation (5) represents a cylinder with its
generators parallel to the z-axis (the co ordinate which is absent in
the equation).
Corollary 2. In view of corollary I above, the equation of
the cylinder whose axis is the z-axis and whose generators inter
section the circle xa+ y 2=w2, z —0 is given by
*2+ y 2—a2.
(C) The equation o f the form f ( jc, y )= 0 represents a cylinder
with its generators parallel to the z-axis. (Rajasthan 1975)
Consider a curve in the xy-plane w'hose equation m two
dimensional geometry is / ( jc, y)= 0. If P (a , j? ) be any point on
it, then we have / ( a , /?)=0.
Now the co ordinates of P in three dimensions are (α, β , 0).
Draw a line through P ( a, 0, 0) parallel to the z-axis. Take
g(a, 3, z) any point on this line. The co-ordinates of the point
Q clearly satisfy the equation / ( jc, y)«=»0. Therefore the co-ordi
nates of every point on the line PQ satisfy the equation/(jc,y)= 0
and hence the whole line PQ lies on this locus.
Thus the assemblage of lines (parallel to the z-axis drawn
through the points on the curve in the jcy-plane) is the required
locus which is, therefore, a cylindrical surface having generators
parallel to the z-axis.
Similarly, the equations / ( y , z) =:0 and / ( z , jc) = 0 represent
cylinders with their generators parallel to jc-axis and y-axis res
pectively. u
312 Analytical Geometry 3-D
or {n (y - β ) - m (z -?)}*+{/ (z ~ γ ) —η (x - a)}2
+{m (x —a)—/ (y ~P)}*=r* (l*+m*+n*) ...(2)
The equation (2) is the required locus of P and hence this is
the required general equation of the cylinder.
Corollary. If the axis of the right circular cylinder be z axis
then putting a=0=y® O , l= m = 0, and n= I in the equation (2),
the equation of the right circular cylinder in its simplest form is
given by x*+ jp—-r*.
§ 3. Tangent plane to a cylinder.
To find the equation o f a tangent plane to the cylinder whose
equation is
ajf*+2hxy+by*+2gx+2fy+ c= 0
at th° point Pixt, y u zx). Also to prove that this tangent plane touches
the cylinder albrig a generator. (Gorakhp'ir 1982)
The equation of the given cylinder is given by
ax*+2hxy+by*+2gx+2fy+c=0. -..(I)
Since Pfx1, y x, Z1) is a point lying on the cylinder (I), we have
OXi*+2hxxyx+ by i*+ Igxi + Ifyl +C=O. ...(2)
Let the equations of any line through the point P (X1, y x, Z1)
be ( x - x 1) / / = i v - y 1)/m =(z *z1)/«= r (say). ...(3)
The co-ordinates of any point on the line (3) are
(Ir + .V1, m r+}\, nr+ zt).
Therefore the points of intersection of the line (3) with the
cylinder (I) are given by
a (lr+ xt)*+2h (lr+xi) (mr+yx)+ b (m r+ ^ )8
+ 2 g (lr+xx)+ 2 f (mr+yi)+c=;0
or r* (aP + 2hlm+bm*) + 2r {/ (axi +hyt+g)+m (hxx+ byx+ f )}
+ (axx*+2hxxyx+byx*+2gxx+2fyx+ c)= 0
or r* (al*+2hlm-{ bm*)+2r {/ (axx+hyx+g)+m (hxx+byx+J ))= 0
-(4 )
[using (2)]
The equation (4) being a quadratic in r, gives two values of r,
Clearly one value of r is zero. Hence, if the line (3) is a tangent
line to the cylinder (I), then the other value of r must also be
zero and the condition for the same is that the coefficient of r
must zero
*·«· I (axx+ hyx+ g)+ m (hxx+byx+ f) = 0 . ...(5]
The tangent plane to the cylinder (I) at the point Ρ(χχ, y x, zx)
is the locus o!' the tangent line (3) for all values of /, m, n and is
obtained by eliminating the variables /, m, n between the equa
314 Analytical Geometry 3-jD
tions (3) and (5). Hence the equation of the tangent plane to the
cylinder (I) at the point P (xl9 y l9 Z1) is given by
(x - x i) frxi+hy1+ g ) + ( y - y j (Ax1+ ^ y 1+ / ) = 0
or x (axi + hyt+ g)+ y (Ax1+Ay1+ / ) H a x 12+ by12+2Ax1y1
+gXi+fyi)=Q
or x (axi+hy1+g) + y (H x ^b y 1+ / ) + (gxi+fy!+c)=0 [using (2)]
or axxx+h (X jh+ *]/)+ byyx+g ( x + x ^ + / (^ + J 1Rc==O. . (6)
Second part. The equation (I) of the cylinder is of the form
f(Xy y)=~-Q and hence, in view of § I (C) above, the generators of
the cylinder (I) are parallel to the z-axis. Again P (x\9 V15 Z1) is a
point on the cylinder (I) and, therefore, the equations of a gene
rator through the point P (X 1, y l9 Z1) are
{x - x 1)/0=(y yi)/0= (z -Z1)/1= r ' (say), ..(7)
The co-ordinates of any point on this generator are
(Xi>yi>'r'+Zi)·
Therefore, using the formula (6), the equation of the tangent
plane to the cylinder (I) at the point (xl9 yl9 / - '+ Z 1) is given by
Oxx1+ h (xyi+ x1y)-\ byy1+g ( x + x /) + / (v + J i)+ c = 0 . ...(8)
Clearly we see that the equations (6) arid (8) are identical.
The equation (3) of the tangent plane remains the same for all
values of r, it being free from r and, therefore, there is same tan
gent plane at every point of the generator (7). Hence the tangent
plane to the cylinder (I) at XhepointP(Xl5Jh izI) touches the
cylinder (I) along a generator through the same point
Pfru yu zi)·
§ 4. Envelopiag Cylinder.
(A) Definition. The enveloping cylinder is the locus of the
tangents to a surface (sphere or conicoid) which are parallel to a
given line or in other words the enveloping cylinder is the cylinder
whose generators touch a given surface and are directed in a given
direction.
(B) To find the equation o f the enveloping cylinder of the sphere
x*+j>2-J-Z2=A2 whose generators are parallel to the line
xll=ylm-=*zln.
^Rohilknand 1982 ; Kanpur 76 ; Meerut 84)
Consider a point P (a, :i, γ) on the enveloping cylinder.
Since the generators of the cylinder are all parallel to the line
xjl=yjm —zln9 therefore the equations of the generator of the
cylinder through the point P are
The Cylinder 315
Since the generator (3) meets the conic (I), hence the point
(*y+Zi, ^i+2zj, 0) will satisfy the equations of the conic given by
(I)9 and so we hav*
jM*t+ zx)%+ 2 O’i+2zj)*=* I
3 (X12+!*!*! +Z1*)-1-2 0'ι*+4-νιζ1+4ζι2)=1
or .W -f 2^!2+ 11Zlt I-ZylZx+ 6zxxx- 1=0.
.·. The locus of P (X 1. y u Z1) i.e. the required equation of
the cylinder is given by
3x2+2j;2+ I lz2+ 8yz-t-6zx -1 = 0 .
Ex. I (b). Find the equation o f the cylinder whose generators
are parallel to the line x =^y/( 2)= ζ β and passing through the
curve x2+ 2y2= I, z - 0. (Agra 1982; Gorakhpar 74;
Meerut 83 S, 84 S; Kanpar 78; Lackiow 79)
Sol. Proceeding exactly as in Ex. I (a) above the required
equation of the cylinder is given by
3 (x2+ 'j ,3+ z 2) 2zx4 8yz 3=-- 0.
Ex. 2 Find the equation to the cylinder whose generators are
parallel to the line x =yj{ 2) = z/3, and the guiding curve is the
ellipse x* f 2_v2= I, z= 3.
(Avadh 1982; Allahabad 80; Agra 78; Garhwal 799 81;
Kurukshetra 76; Meerut 77. 89; Poojah 77; Rajasthan 77)
Sol The equations of the guiding curve are
x2-f2>>2= !, z = 3. ,..(»)
The equations of the given line are
x/\= y/( -2)=z/3. (2)
Consider a point P (xl9 ylt Z1) on the cylinder. The equations
of the generator through the point P (xl9 yl9 Z1) which is a line
parallel to the given line (2) are
x - Χ ι y - y i Z- Z1
I ~ - 2 3 ...(3)
The generator (3) meets the plane z= 3 in the point given by
(3 x - z + 3 ) * + 2 ( 3 .y + 2 z - 6 ) * « 9
or Q-X2-I-z* Π -9— 6 z x + 1 8 x —6z
+ 2 (9y2+4z2+36+I2yz -3 6 j> -2 4 z ) = 9
or 9 * 2+ 1 8.y2+ 9 z 2+ 2 4 ; ; z — 6 z x + 1 8 x — 72>>— 4 8 z + 7 2 ^=O
or 3 x 2+ 6 > '2 -|-3 z 2+ 8 >yz 2 z x + 6 x -2 4 jr -1 6 z -f2 4 = 0 .
Ex. 3. Find the equation of the circular cylinder, whose gene
rating lines have the direction cosines, I9 m\ n and which pass through
the f ix e d circle x*+Z1=G2, in ZOX plane. (Agra 1980)
SoL The guiding curve (circle) is given to be in the ZOX
plane Le. y —0 plane and hence its equation are
x2+ z2^a*9y= 0. .. (I)
Consider a point P (X1, ^1, Z1) on the cylinder. The equations
of the generator through the point P (xlf y 1%Z1) and with d.c.’s
I9m9 n are
(x —xi)/l= iy —.V1M m ^iz -ZlMn. . (2)
The generator (2) meets the plane ^=O in the point given by
X Xi 0 —.Pi z—zi i.e. (x, Iyl Sm, 0, Z1 - n v ^ m ) .
I m n
Since the generator (2) meets the curve (I), hence the point
(xi—lyilm, 0, Zl -H y Jm ) will satisfy the equations of the curve
given by ( I), and so we have
(*i - /.Vi/m)H (Z1- nyjm )2—a*
or (mxt—/vl)2+(m z1- ηγ^*·=α*ηι2.
Λ The locus of P (.X1, y x, Z1) i.e. the required equation of
the cylinder is given by
(imx—ly)*+(mz—ny)2~m*a*·
Ex. 4. Find the equation of the surface generated by a straight
line which is parallel to the line y= m x, z —nx and intersects the
ellipse X2Sa- + y2jbl = I, z = 0. (Avadh 1981 ; M .U. 90)
Sol. The equations of the guiding curve (ellipse) are
X2Ia2F y 2Sb2= :, z=0. ...(I)
The equations of the given line may be written as
x/1 =y!m=zjn. .. (2)
Consider a point P (xt, y lt Z 1) on the cylinder. The equations
of the generator through the point P (x„ y x, Z1) which is a line
parallel to the given line (2) are
(x -X1V I=C p- y ^ i m - f z - Z1Mn -.(3)
The generator (3) meets the plane z= 0 in the point given by
X-X1 ,P-JP1 C -Z 1
I m "(* ■ - Ϊ - - " M
318 Analytical Geometry ?-£>
Since the generator (3) meets the co :ic (I), hence the point
(xi—zjn , y x—MZ1In90) will satisfy the equation of the conic given
by (I), and so we have
(IIati) (^1-::,//0+(1/*2) ( ji—m zjn)2= I
or ft2 (nxx - z , ) 2+ 3 2 (η γ τ m z ,)2= a 2ft2n 2.
Λ The locus of P (xl9 V 1 , .Z1) i e. the required equation of
the cylinder is given by
b2 ( n x - z ) 2+a2 (r\y—mz)2 =Iib1U1.
Ex. 5 (a). Find the equation o f the cylinder which intersects the
curve ax2+by2+CZ1= I 9 lx+ m y+ nz= p and whose generators are
parallel to the axis o f x. (Meerut 1978, 82 , Lucknow 82)
Sol. Thfe equations of the guiding curve are
ax 2+ ftV2+ C Z 2 = X ... ( I)
and lx+my+nz=p. :-(2)
Now the equation of the cylinder whose generators are para
llel to x-axis will not contain the terms of x. [See § I (C)]. Hence
the required equation of the cylinder is obtained by eliminating
x between the equations (I) and (2), and so is given by
[ v Fmm (2,. χ
or a (p my - nz)2+ ft/2+ !+ cl2z2=- /2
or a (p2+ m2y *- \ n Z 2 -''lpmy -2pnz-{ 2mnyz)+bl2y1+cl1z2= l 2
or (am2+ bl2) y'+ (an2+cl2) z* + 2amnyz—lampy
—2anpz+(ap —/2)=0. Ans.
Ex. 5 (b). Find the equation o f the quadric cylinder which
intersects the curve ax2+by2+cz' = I, lx+ m y+ nz—p and whose
generators are parallel to z-axis.
(Kanpur 1982, 83 ; Bundelkhand 79 ; Meerut 85 P)
Sol. Th^ equations of the guiding curve are
ax2+ft;r+C22= l ...(I)
and lx+ tn y+ n z= p. .. (2)
Since the generators of the cylinder are parallel to the z-axis,
therefore the required equation of the cylinder is obtained by
eliminating the z co-ordinate between the equations (I) and (2),
and so is given by
a x 2+ b y 2 +c { ( p — l x — m y ) In}2= \
or an% x + b t v y 2 + c ( P 2 j i- V x 2-1 i;ily 2— 2 / t i x + 2 l m x y — 2 p m y )= /r
The C y lin d e r 319
~ V S W c W o W V if0-O-OO)'+(I.ο - 1.(3-2)}"
+ {0-(3-2) -1.0}·]
= 1/V2-
Consider a point P (x, y, z) on the cylinder. The length of
the perpendicular from the point P (x , y, z) to the given axis (I)
is equal to the radius of the cylinder. Hence the required equation
of the cylinder is (See § 2 (B)]
(l.y- 0.z)2+ { l.z—l.(x—2)}2+{0.(x—2)—l.y}2= (l/\/2 )* (1 + 0 + 1 )
of y*+(r—x + 2 )2+y2= l *
or x2+2y2+ z 2~ 2 zx —4 x+ 4z+ 3--0. , Ans.
Ex. 10 (a). Find the equation o f a right circular cylinder des
cribed on the circle through the points A {a, 0, 0), B (0, a, 0) and
C (0, 0, a) as the guiding curve.
S61. We are given three points A (a, 0, 0), B (0, a, 0) and
C (0, 0, a). Let the fourth point be taken as O (0, 0, 0). The
equation of the sphere OABC is [See Ex. 6 (a), set (A) of the
chapter on spherel
x2+y*+z*—flx—ey— b0. -.(1)
The equation of the plane ABC is xja+ yla+ z/a—l
or x+y+ z= e. --(2)
Theguidingcircleis given by the equations (I) and (2)
together. '
Since the cylinder is a right circular cylinder, hence the axis
of the cylinder will be perpendicular to the plane (2) of the circle
and thus the.d.r.’s of the axis of the cylinder are I, I, I.
Let P (X1, yu Z1) be any point on the cylinder. The equations
of the generator through P (X1, ylt Z1) having d.r.’s 1,1, I are
(X-X1)/! «-(y—y i)l\—{z—Z1)/1= r (say). ...(3)
Any point on the generator (3) is (r+ x l5 r+ ylf r + z j.
Since the generator (3) meets the guiding circle, hence the
point (r+ x „ r+ ylt r+Zj) will satisfy the equations (I) and (2)
of the circle, and so we have
('■+•«i), + (i-+yj)!!+ ( r + z 1)*—a (r+ xfjihe ( r + y ^ - f l ( r + z ^ - O
and ('•+*i)+(>’+ y 1) + ( r + z 1) = a
or 3r2+ r {2x1+2y1+2z1—3a)+(x1t + y1, + z1i —ax1—ay1—az1)=Q
...(4)
322 A n a ly tic a l G e o m e try 3 -D
The Cone
Ex. 4. Prove that the equation o f the cone whose vertex is the
origin and base the curve z = k ,f(x ,y )= 0 isf(xklz, ykjz)=0.
(Kanpur 1979, 81; Meerat 87, 89 S)
Sol. Let x/l—ylm=zln -(0
be a generator of the cone.
Since it meets the base curve z = k , f (x, y)= 0, therefore
x y k ... I , m ,
-r = — m=—, SOthat X·=- k, V=-- AC.
I in it η n
Putting these values of x, y i n / (x, y)= 0, we get
' ( H - H - 0· .( 2)
Eliminating /, m, n between (I) and (2), the locus of
xll= ylm = zln is the cone
' ( T t- T i H '
Alternative Solution. We know that the equation of a cone
whose vertex is at the origin is homogeneous in x, y and z. From
z —k, we have kjz·= I. Therefore m a k in g /(x, y )= 0 homogeneous
with the help of z= k , the equation of the required cone is
f(x k fz , yk/z)—0.
Ex. 5. Find the equation of the cone whose vertex is the origin
and base the circle x —a, y*+z*=*b2 and show that the section o f the
cone by a plane parallel to the plane XO Y is a hyperbola.
(Meerut 1973, 84)
Sol. The equations of the circle are
ya+ z* = p ...(I), and jc/e-1. (2)
Making (I) homogeneous with the help of (?.), the equation
of the required cone with the vertex at the origin is
y*-\-za= P (x/e)* or a* ()ΐ*+ζ*)=Ρχ*. ...(3) Ans.
The section of the cone (3) by a plane parallel to the plane
XO Y Le. by the plane z —c is the conic given by
a* Cys-S-Ca) -- z= c
OT P t f - O aP t = O i C*, Z e C
which is clearly the equation of a hyperbola.
Ex. d. The plane x/a+ y/b+ z/c= I meets the co-ordinate axes
in A, B, C. Prove that the equation of the cone generated by the
lines drawn from O to meet the circle ABG is 1
yz (b/c+clb)+zx (c/a+a/c)+xy (a/b'+bia)=0.
(Agra 1979; Bundelkhand 78; Punjab 82; Nagpur 78;
Rohilkhand 77)
The Cone ' 329
or 4zS( ^ - + £ - + ^ - ) = ( i i - + ^ ) ·
This is the required equation of the cone with the vertex at
the origin.
Ex. 9. Planes through OX, OY include an angle a. Show that
their line o f intersection lies on the cone
z2 (x2+ y2+ z 2) = x 2y 2 tan2 a.
(Agra 1978; Meerut 76, 87, 89; Lucknow 76, 80; Guruiianakdev 76;
Nagpur 76)
Sol. The equation of any plane through OX i.e., the line
y = 0 , z==0 is y +λίΖ—0
or ψ 0.x+ I ^ + A 1Z = O . -O )
ySimilarly the equation of any plane through O Y i.e., the line
x = 0 , z^=0 is
*+Aaz = 0 or l . x + 0 ^+A 2Z=O. -. (2)
The angle between the planes (I) and (2) is given to be V ,
hence we get
_ 0.1+ 1.0+Aj.Af A1A2
cos a“ v ( i + v j v ( T + v r V i(H -V ) (Γ+αλ Γ
• Squaring, we get A12A22 sec2 α = (1 + λ ι2) ( I +A22)
or A12A22 sec2 a= -1+A1s^ A 2H A 12A22
οι A12A22 (sec2 a 1)= I +A a2+A22,
or Ai2A22 tan2 a = I +A12+A22, .. (3)
Eliminating A1 and A2 between the equations (I), (2) and (3),
the locus of the line of intersection of the planes (I) and (2) is
( - y j z ) 2 ( xlz)2 tan2 a = l + (-+ 7z)2+ ( - x / z ) 2
or X2/ tan2 a= za (x2+ y 2+ z 2).
This is a homogeneous equation of 4th degree and hence
represents a cone with its vertex at the origin.
§3. The line x /l—y\tn=-zjn is a generator oj the cone whose
equation (homogeneous) is given by
/ ( x , y 9 z)= ax2+ by2+ czz+ I fy z + 2gzx+ 2hxy= 0,
i f and only if its directiori ratios /, mf n satisfy the equation of the
cone i.e., i f and only i f / ( / , m, /i) = 0.
The equation of the given cone is
/ ( x , y , z ) = A X 2+ b y 2+ cz2+ I f y z + 2gzx+ 2 h x y = 0. / .(I)
The line (2) is a generator of the cone (I) if and only if each
point on the line (2) lieso n (I) Le., if and only if the point
(Ir, mr, nr) satisfies (I) for all values of r Le., if and only if
(al2+bm2+cn2-\-2fmn -\-2gnl+ Zhlm) r2= O for all values of r Le., if
and only if al2 \-bm2+cn2s±2fmn+2gnl+2hIm = 0 i.e., if and only
if/ ( / , m, n) = Q.
§ 4. To find the general equation of a cone of second degree
which passes through the co-ordinate axes, the axes being rectangular.
(Avadh 1980; Agra 76, 78; Garhwal 78, 82;
Kurukshetra 76 ; Meerut 87P)
Since the cone passes through the co-ordinate axes, its vertex
will be at the origin and hence its equation will be homogeneous
of second degree. Let the equation of the required cone be given
by ax2+by2+cz2+2fyz+2gzx-¥ 2Axy=O.' ...(I)
The d.c.’s of the x-axis are I, 0,0. The x-axis will be a
generator of the cone (I), if the d.c.’s I, 0, 0 of the χ -axis satisfy
(I) [See § 3 above] and hence we get a = 0.
The d.c.’s of the y- axis and z-axis are 0, I, 0 and 0, 0, I
respectively. Similarly the y and z axes will be generators of the
cone (I) if ft=0, C=^ O.
Substituting the values of i/, b .c in (I), the required equation
of the cone which passes through the co-ordinate axes Le., the
cone which has co-ordinate axes as generators is given by
fyz+ g zx + hxy=0.
SOLVED EXAMPLES (B)
Ex. I. Show that a cone can be found to contain any two sets
of three mutually perpendicular concurrent lines as generators.
Or
Show that a cone of the second degree can be found to pass
through any two sets o f rectangular axes through the same origin.
(Rohilkhand 1979)
Sol. L e to n e se to f three mutually perpendicular lines be
chosen as the co-ordinate axes and the other set of three mutually
perpendicular lines through the same origin have their direction
cosines Ilf mx, nx ; I2f m2f n2 and I3, m3f n3.
Thegeneral equation of the cone passing through the co
ordinates axes (/.<?., one set of rectangular axes) is
fyz+ gzx+ hxy= 0 ,..(I)
[See § 4 above]
332 Armfftical Geometry I-D
If the cone (I) passes through the first two lines of the second
set, then we have
fm xnx+gnxlx+ Iiltfn1=O9 ... (2)
and fni2n2+gn2l2+hl2mt =0. ...(3)
Adding (2) and (3), we get
Z im lHi ±m 2n2)+g ( n ^ + n f i ^ + h (Itfn^U m i)=O
or f ( —m% nz)+g (—/I3Z1)+ * ( - / 8m3)= 0
[V W1H1-I-WaWa+W3H8=O, etc., the lines being mutually
perpendicular]
or /w aWa+gw8/3 +/i/am8= 0 . ...(4)
The relation (4) shows that the cone (I) also passes through
the third line of the second set.
Hence a cone can be found to pass through any two sets of
rectangular axes through the same origin.
Ex. 2. Show that the equation o f the cone which contains the
three co-ordinate axes and the lines through the origin having direc
tion cosines I19 W 1. W1 and I2i w2, n2 is Σ I1I2 (W1Wa-W 2Wi) yz= 0.
Solution. First we shall show that a cone of second degree
can be found to pass through five concurrent lines.
Take the point of concurrency of the lines as origin. We
know that the equation of a cone with the vertex at the origin is
ax2+ by2+ Cz2+ Ifyz+ Igzx+ Hixy = 0. ' ... (I)
Let α^άΟ. Dividing throughout by ‘a \ we get
xt+ib/a) y 2+(c/a) z2+ 2 (f/a ) y z + 2 (gja) zx+2 (h/a) xy= 0
or X 2+ b y 2+ cfz2+ 2f ' y z + 2g'zx+ 2h'xy = 0, ... (2)
where b'==b/at c '^c/a etc.
The equation (2) of the cone contains five arbitrary indepen
dent constants na ely b \ c \ f \ g' and h' and as such a cone can
be found to satisfy five independent 'conditions. Therefore, a cone
can be made to pass through five concurrent lines.
The general equation of the cone through the three co-ordi
nate axes is [See § 4]
fy z+ g zx+ Iixy=0. ...(i)
If the cone (I) also passes through the lines whose d.c.’s are
I11 W 1, W1 and I29 w2, n2 then these d.c.’s will satisfy the equation
(I) of the cone [See § 3] and hence we have
fnixnx+gnxlx+ Mtfn1^O 9 ...(2)
and f i n & t + g n J t + M t f n t =0. .... (3)
Solving (2) and (3), we get
The Cone 333
/{ M -T -a -» ·
Sol. Let the equations of the line OP be
XlIi=Plmt =Zln1 ...(I)
and that of OQ be x/lt =ylm t =z/nt. (2)
It is given that the lines OPand OQ are perpendicular
therefore
hi»+t»itnt+«i«i= 0. ...(3)
Now it is given that the plane OPQ (i.e. the plane containing
the lines OP and OQ) passes through OZ (i.e. x= 0= y). The
equation of any plane through OZ is
χ+ λ+ = 0 . .. (4)
Let the equation of the plane OPQ be given by (4). Since
the plane OPQ given by (4) contains the lines OP and OQ, there
fore the lines OP and OQ both will be perpendicular to the nor
mal of the plane (4), and hence we get
/j.l+ m 1A-|-«i.0=0 and 4 1 + ^ ,^ + / ^ . 0 = 0
or X = - I lIml= - I tImt. ...(5)
Again it is given that the line OP given by (I) describes the
c o n e /(ylx, z/x)=0. Therefore the d.c.'s I1. mlt w, of the line OP
will satisfy the equation of this cone, so that we have
f i m j l i , /1,/4)=0. ...(6)
Dividing (3) by Iu we have
4 + («»/ 4 ) »»i + ( « j /4 ) « * — o
Hence the line (2) i.e. the line OQ whose d.c.’s are It, mt, nt
generates the cone
Proved.
336 Analytical Geometry 3-D
§ 5. The equation of the cone with a given vertex and a given conic
as base. To find the equation o f a cone whose vertex is the
point (α, β, y) and base the conic
ax*4-2hxy4-by*+2gxA-2fy-\-c=0, z= 0.
(Agra 1980; Delhi 75; Indore 76; Lncknow 78; Kanpnr 76)
The equations of the base of the cone are given to be
ax* Af^hxy Jr by2 f 2gx+2fy+c=0, z= 0. · ( I)
The equations of any line through the vertex (α, β, y) are
(x -a )//= (^ -p )/m = (z -y )/n . ..(2)
The line (2) meets the plane z = 0 at the point given by
x —a y —β 0—y . ( Iy my n\
—r = -—- = — - i.e. at the point I a —— , B---- -, Ol
I m ■ η \ n r n /
If this point lies on the given conic (I), then we get
a,
The relation (3) is the condition that the line (2) intersects
the conic (I), and hence the locus of the line (2) i.e. the required
equation of the cone is obtained by eliminating /, mf n between (2)
and (3).
From (2), we have 1
/ x- λ , m y -β
— =----- , and — = - — —·
η ζ-γ η ζ —γ
Putting these values in (3), the required equation of the cone
is given by
+ V { P - ( y: :_ y ) v } + C- 0
Multiplying throughout by (z y)2 and simplifying, we get
a 1<χζ~γχ)*+2Ι] ( j ~ γχ) (βz - y y ) + b (βζ—yy)*
+2g'(az—yx) ( z - y)+ 2f ( β ζ -y y ) (z - y ) + c (r - 7)*= 0 .
§ 6. Tofind the condition for the general equation o f the second
degree to represent a cone and to find the co-ordinates o f its vertex.
(Avadh 1982; Allahabad 77, 80; Jodhpur 78; Rajasthan 75)
The Cone 337
g f C W
U V W d
If the condition (7) is satisfied, then the equation (I) represents
a cone. The co-ordinates (α, β, γ) of its vertex are obtained by
solving the equations (3), (4) and (5) and are given by
X - β _ y -I
A g u a g «II a Λ u a h g
b f V A / v I H b v A b f
f e w g C W I g f W I g f C
338 Analytical Geometry 3-Z>
M ^ W 1= -K -= J W
or {γγ-βζ)*= 4α {&γ—βχ) (y—β)
or { β ζ - yy)*= 4a (β- y ) (βχ - ay). Proved.
Eu. 2 (a). Find the equation o f a cone whose vertex is the point
(α, β, y) and whose generating lines pass through the conic \ ·
x*la*+y*Jb* = I , r= 0 . (Gorakhpur 1975; 82; Gaahati 77)
Sol. The equations of the given conic are
XtIat -^yiIbi = I t Z=0. Ο)
The equations of any line through the vertex (α,.β, y) are
(x -a )//= ( y - 0 ) //n = ( z - y ) /« . -(2)
The line (2) meets the plane 2«=0 at the point given by
X - a y —β O y . ' . . ■, I W a mY nV
I m n \ π n /
If this point lies on the conic (I), then
ie . (αV ) + ( 4 V + α*β* - a W )= 0
or Ρα·+α*(β*+γ*) -a*4*.
Λ The locus of the point P (a, β, γ) is
4*x*-fe* (y*-Vzt)= atbt or x lja2-V(y* -Vzt) jb*= I. Proved.
Ex. 3. Find the equation to the cone whose vertex is the point
P (a, b, c) and whose generating lines intersect the conic
px%-\-qy*= I, z=0. ' '
(Agra 1982; Avadb 81; Calcutta 77; Meerot 85)
Sol. The equations of the base curve are
PXi Vqyt = I t Z=O. ...(I)
The equations of any line through the vertex P (a, b, c) are
( x —a)ll=(y—b)fm=(z—c)ln. \·- (2)
The line (2) meets the plane z= 0 at the point given by
x —a y —b O—c . . . ( Ie . me A
_ l.e. at the potat ( a - - , b - - , o)
This point will lie on the conic (I), if
p (a—Icjn)*-Vq(b —mc/n) = I. ...(3)
Eliminating I, m, n between the equations (2) and (3), the
equation of the required cone is given by
' Η ^ * ϊ ,+« Η £ - ί Μ ν ι
or p (az—exp+ q (bz—Cyf=(Z-C)*
or c* (px* + qy*)-V(p(P-Vbtq - \ ) z*—2c (apzx+bqyz—z)=c*. Aos.
Ex. 4. Find the equation o f the cone whose vertex is (I, 2, 3)
and guiding curve is the circle Xt -Vy2-Vzt = 4, X-Vy-Vz= I.
(Lucknow 1982)
Sol. The equations of the guiding circle are given by
*, +3'*+2, ==4 ...(I) and x -Vy+z= I. .. (2)
The equations of any line (generator) through (I, 2, 3) are
x — I _ y —2_ z —3_^x-Vy + z ~ 6
I m n ϊ+m+n .--(3)
[Note]
The line (3) meets the plane X-Vy-Vz=I at the point given by
x —I_y —2_z —3_ 1—6 ^ —5
I m n I-Vm-Vn l+m-Vn
. . /. SI „ 5m ^ Sn \
U . at the PO,at 2 - + ^ -
t m-Vn—41 2l+2n—3m 3 /+ 3 m —2n\
V + m + » ' I-Vm-Vn ’ l+m-Vn /
The Cone 341
This point lies on the base circle given by the equations (I)
and (2), hence this point will satisfy the equation (I) and so we
have
(m + n -4/)*+(2/+2n-3m )*+(3/+3m -2n)*«=4 (/+m +n)* ...(4)
Eliminating /, m, is between (3) and (4), we· get the IocUs of
the line (3) i.e., the equation of the required cone. We observe
that the equation (4) is homogeneous in /, m, n, so to eliminate /,
m, n, we can put even the proportionate values of I, m, n from (3)
in (4). Then the required equation of the cone is given by
{(y—2 ) + ( 2 - 3 ) - 4 (x—1)}*+{2 ( x - l ) + 2 ( 2 - 3 ) - 3 0--2)}*
+ {3 (x—1)+3 0 - 2 ) - 2 (2—3)}*=4 { ( x - 1 )+ 0 - 2 ) + ( 2 - 3 )} *
or 0 + 2 —4x—l)*+ (2x+22—3.y—2)*+(3x+3y—2 2 - 3>*
= 4 (x+>-+ 2 —6)*
or 5χ*+3^*+2*—6yz—4rx—2 x y + 6 x + 8 y + 1 0 r—26=0.
Ex. 5. Find the equation o f the cone with vertex (5, 4, 3) and
with 3x*+2>*=6, y + 2 = 0 as base. (Indore 1979)
Sol. The equations of the base curve are
3x*+2>*=6, and y + 2 = 0 . ...(I)
The equations of any line (generator) through (5, 4, 3) are
x —5 y —4 2—3 y + z —7
I m n m+n ...(2)
[Note]
Ih e line (2) meets the plane y + 2= 0 at the point given by
x —5 y —4 2—3 0 —7 —7
/ β m ^~n~ ~ m+ n ^ m+n
Tm , Tn \
i.e., at the point ' I s - J U -
V ' m +n m+n
/ Sm+5*»—Tl 4«-3m 3m—4n\
or
\ m +n * m +n ’ m + n J
If this point lies on the base curve given by (I), then this
point will satisfy 3x*+2^---^6 and hence we have
3 (5m +5n—7/)*+2 (4n —3m)*=6 (m+n)*. ...(3)
Putting proportionate values of I, m, n from (2) in (3) and
thus eliminating /, m, n between the equations (2) and (3), the
required equation of the cone is given by
3 {5 0 - 4 ) + 5 ( 2 - 3 ) - 7 (x—5)}*+2 {4 ( 2 - 3 ) - 3 0~4)}*
= 6 (0 -4 )+ (2 -3 )} *
or 3 (5y+52—7x)*+2 ( 4 2 - 3 ^ / = 6 0 + 2 —7)*
or 147x*+87j'* + 10l2*+90j»2—210rx—210x)'+84<y+842—294*^0.
342 Analytical Geometry 3-D
is the locus of the tangent line (2) under the condition (5) and is
obtained by eliminating /, m, n between the equations (2) and (5).
Thus the equation of the tangent plane at P (α, 0, γ) is given by
(a«+-A0+gy) ( χ - α ) + ( A*+A0+/y> (y- β )
+ - ( g a + /0 + cy) (2 — y ) = 0
or x (ax + hβ+gγ)+ y (Ιιχ+ϊβ+]γ) + ζ (ga+/0 + cy)
—(aa*+ 60* + Cyt +-2/0y-j-2gya + 2A*0)*=»0
or x (act.+-hβ+gγ)+y(hx+bβ+fγ)+z^gx-{-fβicγ)=0 ...(6)
[using (3)]
Note. The equations (S) and (6) can be more conveniently
remembered and written with the help of differential calculus as
follows :
Differentiating (3) partially w.r.t. a, 0, y respectively, we
have
BFjdx = 2 (a* + A0 fgy), BF/Ββ 2 (hx+όβ -\-fy)
BFjdy —2 (gx +/0 + cy).
Thus the condition (5) that the line (2) is the tangent line to
the cone (I) may be written as
. BF , B F , BF n
' - I S m Te+ " Tr - * ...(S')
The equation (6) of the tahgent plane at the point P (a, 0, y)
to the cone (I) may be written as
B F , B F 1 BFi .
x T S y Te+ 2 Tr - 0- (e/)
Corollary. To prove that (he tangent plane at any point
P (ά,β, γ) is the tangent plane at every point o f the generator through
P (α, β, y). In other words, the tangent plane at P touches the cone
along the generator through the point P.
The vertex of the given cone (I) is O (0, 0, Uj, and the tan
gent plane at P (α, β, γ), i.e. (6) clearly passes through the vertex
O (0, 0, 0).
The equations of the generator OP are clearly given by
X -Q n J - O = 2—0 j 2
——k (say).
a —0 )3-0 y —0 €' α β V
Any point on this generator is (/ca, Ar0, ky). The equation of
the tangent plane at (kx, A0, ky) to the cone (I) is
X (akx+hkfi+gky)+y (hkx+ bkβ+ fkγ)+ z (gkx+fkβ^\-ckγ)■=0
or * (aa+-/j0+gy)+y (A a+60+/y)+z (ga+ -/0 + cy J-0
which is the same as the tangent plane at P (a, 0, y) given by (o).
The Cone 347
h b f V
g f C W
U V W 0
348 Analytkal Geometry 3-D
h b f
g f C
Thus
» ~ b c - f* - JMf , B = c a - g * J J ± .C = a b - h * J £
(IO)
F ^gh- α / ^ ψ . G ~ h f-b g -i
§ 9. The Reciprocal cone.
Definition. The reciprocal cone o f the given (one is the locus
of the lines through the vertex and at right angles to the tangent
planes o f the given cone, Le. the reciprocal cone a f the given cone is
the locus o f the normals through the vertex to the tangent planes o f
the given cone.
(A) To find the equation of the reciprocal cone of a given
cone.
Let the equation of the given cone be
ax*+by*+cz*+2fyz+ 2gzx+ 2hxy =*0 ...(I)
and let ux+vy+wxeaO ...(2)
be the equation of a tangent plane to the cone ( I). The condi
tion that the plane.(2) touches the cone (I) is given by [see equa
tion (9), § 8]
Au*+Bv*+Cw*+2Fvw+2Gwu+2Huv=*H. ...(3)
The d.r.’s of the normal to the plane (2) are u, v, w. Hence
the equations of the normal to the plane (2) passing through the
vertex (0,0, 0) of the given cone (I) are
xlu=ylv*az/w. ...(4)
Eliminating u, v, w between (3) and (4), the locus of the
normal (4) is given by
Ax*+ By*+Cz*+2Fyz+2Gzx+2 Hxy =- 0. .. .(5)
The equation (5) being a homogeneous equation of second
The Cone 349
degree represents a cone with the vertex at the origin. Thus the
cone (5) is the reciprocal cone of the given cone (I).
Working rule. In order to find the reciprocal cone of the
given cone ox*-|-hy*+er*+2/yz+2£ZX+2/ury=»0, we have to simp
ly replace the small letters a, b, c,f, g, h by their corresponding
capital letters A, B, C, F, G, H, where the capital letters are the
co-factors of the corresponding small letters in the determinant
a h g
h b f
Z f C
(B) To find the reciprocal cone of the cone
Axl +Byt +Czt +2Fyz+2Gzx+2Hxy=‘0. ..:(6)
To find the reciprocal cone of the cone (6) we should only
replace A, B, C, Fi G, H by their co-factors in the determinant
A H G
H B F
G F C
Let A', B', C , F', G', H be the co-factors of A. B, C, F, G, H
in the above determinant. We have
A '^ B C - F t ^ ( c a - g t) (ab—h*)—(gh- a/)*
[See equations (IO) of § 8]
—aibc -f- 2afgh—a’/ 1- Obgt -Ocht
■»« (abc+2fgh~ a ft —bgt —cht) - a Λ.
Similarly B'=*bA> C’—c A ,
and F '= G H ^ AF=f A tG ' HF—B G -gA * H'= F G -C H ^ h A-
Now the reciprocal cone of the cone (6) is given by [See
working rule of § 9 (A) above]
A'xt + B 'yt 4-C z t + 2F'yz+2G'zx+2H'xy*= 0
or A (ox*+ by1+ cz*+ 2fyz+ 2gzx+ 2hxy)*=* 0
or axt +byt +czt +2/yz+2gzx+2hxy=0 .. (7)
[as A ?*0 because ( I) represents a cone].
But in § 9 (A) we have proved that the reciprocal cone of (7)
is given by (6). Hence we see that the cones (6) and (7) are such
that each Is the locus of the normals through the vertex (the origin)
350 Analytical Geometry 3-D
Ex. 6. Find the condition that the plane ux+ vy+ w z= O may
touch the ccne axi +byi +czi =0. (Meerut 1984 S)
Sol. Tbe equation of the cone is
<7X*+'6}>*+ C2*= 0. -.-(I)
The equation of the plane is ux-l-vy+wz-O. ...(2)
We know that the condition that the plane ux+vy + wz^O be
a tangent plane to the cone ax i + b y i + c z i + 2 / y + 2 g z x + 2 h x y = 0
is (See § 8)
Au*+ Rv*+Ctvi + 2Fvw+ 2Gwu+ 2Huv ^=0. .(3)
Here a = -a ,b = b ,c = c ,f= 0 ,g = 0 ,h = 0 .
.·. A = i,b c ~ fi"=>bc—0=1be.
Similarly B=ca,C=ab.
Also F= "gh—a f” =0.
Similarly G -O and H = 0.
Putting these values in (3), the required condition of tangency
is given by
bcu*+ cav*+ab w*+ 0 + 0 + 0 -- 0
or u*/fl+ v*/^+M’i/CKr 0
Alternative method. In view of the Mmpottant note’ to § 9,
the plane ux+vy+w z—0 will be a tangent plane to the cone
ax*+6.y*+cz*~0 if the normal to this plane through the vertex
of the given cone i.e., through the origin in this case is a genera·
tor of the cone reciprocal to the given cone namely
UX3 + (>>>*+ C Z 1 :=0.
•hows that the plane (I) cute the cone (2) in two lines. Let the
direction cosines of these two lines be h, mt, nt and Z1, mt, π».
Thus the roots of the equation (6) are Iilmt and Zs/ma. By theory
of equations, using the formulae for the sum and product of the
roots of the equation (6), we have
Zi . Zt —2 (cuv—fuw—gvw+hw1)
Cut -^awt -Ig u w
Zimt-f ZtTW
l_______ !",In,____ » Zsav)
or —2 (cw —fuw—gvw+hw^^ciA+aw^—lguw 1 3
...(7)
I1 It__ b w * v w
and mt /η*- eu*+aw*—2guw
Z1Zf iw,m, WfW, A
·* hw, +cv*+2/rw"!aeii, +eH^—igsiW- ^ + * ! ^ - 2hwi “ '
...(8)
keeping in view (7) and writing the third fraction by symmetry.
(Z jlW f— Z1WJ1) 1 = (Z j IW f - |- ZfTW j)*— 4 Z jZ f IW1IW f
=4λ* (cuv—fuw—gvw+ Aw*)*
—4A* (Am^ + cv*—2/iwv) (« ι· + αη4—2guw)
~4A»w» { - ( ^ + ^ + 0 ^ + 2 ^ + 2 ( 7 ) ^ + 2 ^ ) }
=4λ·»ν·0*
where the expression within the brackets is denoted by Dt and is
given by the determinant
a h g U
h b f V
g f C W
u v w O
Also the capital letters A, B, C, F, G, H are the cofactors of
the corresponding small letters a, b, c ,f, g, A in the determinant
a h g
h b f .
g f c
Hence Umt - Zf/w1=>2AK'Z). I
Similarly Wi1Wl—/Wfij1■»=2\uD and ntlt —HtIi ^iXyD. J .. (9)
356 Analytical Geometry 3-D
The equation (7) therefore gives the angle between the gene-
rotors of the cone cut by a plane passing through the line
*/1= ^/1= z /l. It is required to prove that this line is the axis of
the given cone. It will be so if we prove that the angle between
this line namely $/1=^/1 = 7/1 and any generator of the cone is
half of the angle Θgiven by (7) Le 9 vertical angle of the cone
contained between the generators in which a plane through the
axis cuts the cone.
Now if x/l= ylm —zln is a generator of the cone
y z + z x + x y = 0, then mn+nt+lm=0. ..(8)
Let a be the angle between this generator and the line
x/l==y/l==z/l then we have
_______L I H-m. I + n. I _____
cos “ - λ/{(/*+«*+«*)
_________ l+m-\-n
V $- y/{(l’+ m+n)*—2(mn+nl+lm)}
I
since mn+nl+lm=*0 from (8).
λ/3·
and A1, Mt, Vi be the d.r.’s of these two lines so that* from (2) we
have,
the product of roots= —. —=»^—~ r ~ i r *
Mt Mt an*+ c l*
• <b* <«*>·
..(3)
Also the sum of the roots=— + — · lime
Mi Mi an2+cl2
*iMi+*iMt _ MiMa = AT[from (3)]
—2Imc
Miva+Mivi ν>*»+ν**ι (by symmetry) .(4)
. —2mna —2nib
Now the equations of the two lines of section are
*/Ai «=j7mi = z/ vi and χ λ a=JVmi= */vi.
Therefore, the equations of the planes through the origin
perpendicular to these lines of section are
A,*+Mi.?+viZ= O and A2X+ Ma^+V2Z=O.
Hence the combined equation of the planes through the origin
perpendicular to the lines of section is
(λι*+Μι.V+viZ) (A2X+ p% y + v2z)= O
or A1A1X1+ /! ,^ * + V 1ν ^ + ί ^ ν * + ^ ) yz
+ (A1V2+A2Vj) zx + (Aj/z2+Aj/A|) X^==O
or (bnv+cma) x*+(cl*+an2) y 2+(am2+bl2) z 2
—2amnyz—2 b n lzx -Iclmxy=O9 using (3) and (4).
Proved.
Ex. 10. Prove that the conditions that the lines o f sections oj
the plane lx+ m y+ nz= 0 and the cones
ax2+by2+cz2*=*0, fyz+ gzx+ hxy= Q may be coincident are
bn2+cm2 ^ c l 2+ an2^ a m 2+bl2
fmn gnl him (Meerut 1977)
Sol. Let the equations of a line of section of the cone
ax2+by2+cz*«=0 by the plane lx+ m y+ nz^ 0 be given by
χ/Λ=^//χ=ζ/ν. Then, we have
a\2+bp2+ cva=0, /A+m/t+nv=0.
Proceeding as in Ex. 9 above, we get
*i*i _ MiMi _ VjVi
btP-t cm2 cl2+an2 am2+bl2 -...(I)
366 Analytteal Geometry 3-D
Pt Ρ* qiq» V i
bn*+cm* _ c/2-i-an*_ am*+ bl*
Jjl “ g/m “ h/n
Multiplying the denominators throughout by lmn, the required
conditions are given by
bn*+cm* cl*+an* am*+bl*
fm n β gnt ~ him ·
Ex. 11. Show that {he planes which cut ax*+ by*+ cz*—0 In
perpendicular generators iouch the cone
x*l(b+c)+y*l(c+a)+z*l(a+b)=o: (M.U. 1990)
Sol. The equation of the given cone is
ax2+^y2H-CZ1=O. ..( I )
Let the equation of any plane through the vertex (0, 0, 0) of
the cone (I) be
Kjf+vy+H>z=0. .. (2)
Suppose the plane (2) 4puts the cone (I) in perpendicular
generators. Then proceeding as in Ex, 3 above, we have
(h+c) u*+(c+a) v*+(a+h) W2=O. -(3 )
The Cone 367
From (3) we observe that the normal to the plane (2) through
the origin i.e. the line x/u= y/v= zlw lies on the cone
(6+c) x*+(c+a) y*+(a+b) z*=0. ...(4)
Now the plane (2) is a tangent plane to the cone which is
reciprocal of the cone (4). The equation of the cone reciprocal
to the cone (4) is
**/( b + c) +y*l(c+a)+ z*j[a+ b)= 0 ... (5)
[See Ex. 4 page 347].
Hence if the plane (2) cuts the cone (I) in perpendicular
generators then it touches the cone (5).
Ex. 12. Show that the locus of the line of Intersection o f tangent
planes to the cone ax*+by*+Czt =O which touch along perpendicular
generators Is the cone
a* (b+c) x*+b* (c+a) j^+c* (a + b )r* = 0 .
(Kanpur 1977, 81)
Sol. The equation of the given cone is
ax*+by*+cz*=0. ...(I)
Let x//= yIOi=Zjn · ...(2)
be the line of intersection of the two tangent planes to the cone
(I) which touch the cone along two generators O Fand OQ where
P and Q are the points («,, βι„ yx) and (a*, )3*, y*) respectively.
The tangent plane to (I) at (Ot1, βu yt ) is
aoti x + bfiiy+ CytZ= 0.
Since it contains the line (2), therefore
a*1l+bfi,m+cyln=0. . ...(3)
Similarly since the tangent plane to (I) at the point (a2, /3*, y*)
also contains the line (2), therefore
OixiI+bfitm + Cyjt=O. —(4)
From (3) and (4) it is clear that both the points («*, JS11 yt)
and (α*. β·2, y*) lie on the plane
alx + bmy+cnz= 0 (5)
through the origin. Hence (S) is the equation of the plane OPQ
containing the two generators OP and OQ thetangent planes
along which intersect in the line (2).
If we put al=u, bm=v, cn=w, the equation of the plane (5)
becomes
ux+ vy+ w z= 0. ..(6)
The plane (6) cuts the cone (I) inperpendiculargenerators
OP and OQ. Proceeding as in Ex. 3 above, the condition that the
plane (6) cuts the cone (I) in perpendicular generators is given by
368 Analytical Geometry 3-D
(can*+abmt) +·(abP+bcn*)+(6cwi*+fe/*)«=0
or a(b+ c) l9+b ( c + q) m*+c (o-\-b) n*=*Q.
.·. The required locus of the line of intersection (2) is given
by
a (b + c )x * + b (b+a) y*+c (a+b) 2*=0
which being a homogeneous equation of second degree represents
a cone with the vertex at the origin.
Ex. 14. A line OP Is such that the two planes through OP, each
o f which cuts the cone aXt +byt +czt = 0 in perpendicular generators
are perpendicular. Prove that the locus o f OP is the cone
(2a+b+c) xt +(a+2b+c) y*~+(a+b+2e) Zt =O.
(Jodhpur 1976)
Sol. Let xll= ylm = zln -(1)
be the equations of the line OP.
The equation of any plane through OP may be given by
ux+vy+wz=Q -*(2)
where ul+vm+wn=0. -(3)
Now proceeding as in Ex. 3 above, the condition that the
plane (2) cuts the cone axt +by*+czt = 0 in perpendicular genera
tors is given by
(b+c) 11*+(<·-(a) +6) We==O- —(4)
The d.r.’s 11, v, w of the normal to the plane (2) are given by
the relations (3) and (4). Also the equation (4) being a quadratic
in 11, v, w shows that there will be two planes like ux+ vy+ wz= 0.
Let Hi, Vj, Wi and ut , v2, w* be the d.r.’s of the normals to the two
planes. Since the two planes are given to be perpendicular, there
fore we have
Ui Ut + ViVi -I-WiWi =O. .(5)
Now eliminating w between (3) and (4), we have
(b+c) u*+(c+a) v*+(a+b) {—(ii/+vm)/n}*—0
or {(b+c) n»+(a+b) /*} (u/v)*+2 (a+ b) Im (u/v)
+ {(c+a)«*+(a+b) m*}=0,
which is a quadratic in u/v.
Its prodnct of roots' u%_ Μ* _ (c+a) n*+(a+b) iw*
ν ,· ν 2 - (b+c) n '+ le + b ) /*
. ___________ U l Ut _____________________________ V 1V 2 ___________
(c+o) «*+(a+b) m* “ (a j-b) P +(b + c) n*
______ w,wt ___
(by symmetry).
“ (b+c) m*+(c+a)P
370 A nilytkal Geometry 3-D
Ex. 5. Find the equation of the right circular cone whose axis
is x = y = z , vertex is the origin and whose semi-vertical angle Is 45°.
(Beriwin 1974)
SM. Proceeding as in Ex. 4 above the required equation of
the cone is given by
x?+y*+z*—4 (yz+zx+xy)*=*0.
Ex. 6. Find the equation of the right circular cone with vertex
ut (I, —2, —I), semi-vertical angle 60® and the axis
(*=* I)/3** —O'+2)/4 =*(z+1)/5. (Gorakhpur 1980)
SoL The vertex of the cone is A(I9 —2, —I). The equations
of the axis of the cone are (jc—1)/3=0^+2)/(—4)=»(z+1)/5.
The d.r.*s of the axis of the cone are 3* —4, 5. The semi
vertical angle of the cone is 60°.
Consider a general point R{x9y 9 z) on the cone and so the
d.r.’s of the generator AR are x - 1 , j + 2 , z+ 1 . Hence the requi
red equation of the right cone is given by
cos 60 3 . ( x - 1 ) + ( - 4).(y+ 2 )+ 5A z+ 1)
Λ/{(3>*-h ( —4>*+ ( 5)*}
. 3x—4 j+ 5 z —6
or
V W W ( x * + y * + z %-2 x + 4 y + 2 z + b j
Squaring and cross-multiplying, we get
25 (x*+.y, + z f -2 x + 4 y + 2 z + 6 )= 2 (3x 4y+5z-6)*
or 7x* -T y t - 25z*+ 80yz—60zx+ 48xy+ 22x+ 4y-|-170z+78** 0.
Ex. 7. Find the equation of the cone formed by rotating the line
2x+3y=6, Z=O about the y-axts.
Sol. A generator of the cone is given by
2x+3y«=»6. z= 0 or 2 (x—3 ) = —3.y, z = 0
or (x —3r/3=j^/(—2) =z/0. ...(I)
The axis of the cone is. y-axis whose d.c.’s are 0 ,1 ,0 . The
vertex A of the cone is the point of intersection of (I) with the
y-axis (i.e. x = 0 , z =0) and is given by (0, 2, 0).
If 0 be the semi-vertical angle of the cone then it being the
angle between the generator (I) and the y-axis (t.e, the axis of the
cone) is given by
_________ 3 .0 + (-2 ).1 + 0 .0 _________ j —2
V{(3)t + ( - 2 ) t +(O)t}V{Ot + l t +0*) “ V(13V ...(2)
Consider a general point ϋ (χ 9y 9 z) on the cone and so the
d.r.’s of the generator AR are x —0, y —2, z —0 i.e. x, ^ - 2 , z.
Also the angle between the generator AR and the axis of the cone
(i.e. y-axis) is 0. Hence we have
384 A n a lytica l G eom etry 3 -D
χ.Ο+Ο»—2).1 + z 0
cos
V{*’ + iy 2)»+2*}V(0*+ i *+ o»>
_2 y _2
or V(T3)“ V ( ^ + ^ + z * - ^ + 4 ) * ue,Dg (2)
Squaring and cross-multiplying, we have
4 (jc*+ j * + z*-4 ^ + 4 )= 13 (>·-4^+4)
or 4 x e— 9 y * + 4 z f + 3 6 y — 3 6 = 0 .
This is the equation of the required cone.
Ex. 9. Find the equation o f the cone generated by rotating the
line x ll^ y /m ^ z /n about the line xla=*ylb~z/c as axis.
Sol. The equations of the axis of the cone are
xfa=ylb=zlc. ...(I)
The equations of a generator of the cone are
xll—y lm ^ z/n . ...(2)
Let Θ be the semi-vertical angle of the cone then it is the
angle between the lines (I) and (2; and so we have
al+bm +cn
cos 0=
..(3)
Consider a general point R (x9y, z) on the cone. Nowthe
vertex A of the cone being the point of intersection of (I) and (2)
is given by (0, O9 0). Thus the d.r.’s of the generator AR are
x —O9y —0, z - 0 i.e. x, y %z. Also the angle between the axis (I)
and the generator AR is Θand hence we have
c o s 9 e _______ ax+ by+ cz_______
y/{a*+6*+ £*)-*/(** +y*+Z)* (4)
Equating the two values of cos 0 given by (3) and (4), the
required equation of the cone is given by
ax+ b y+cz al+bm+cn
Vixt+ jP + ViP+nP+n*)
or (ax+by+cz)* (l9+mt + rf)= (xi + y%+ z*) (al+bm+cn)*.
Ex. 9 (a). Ifa right circular cone has three mutually perpendi
cular generatorst then show that the semi-vertical angle is tan"1
tPonjab 1982 S; Madras 76)
Sol. Let us consider the equation of the right circular cone
with the vertex at the origin, axis the z-axis and the semi-vertical
angle 0. Its equation is given by ISee § 13 (B) case II]
x*+y*+z* tan* 0 = 0; ...(I)
Ifthe cone (I) has three mutually perpendicular generators,
The Cone 385
then the sum of the coefficients of x%,y 2 and z* in the equation (I)
must be zero.
.·. 1+ 1+ ( (an2 0)--=0; or tan2 0=2, or ta n 0 = -* A
or 0 = tan "V 2 ·
Ex. 9 (b). I f a right circular cone has three mutually perpendi
cular tangent planes, then show that the semi-vertical angle of the
cone is given by cot_I \/2.
Sol. Let us consider the equation of the right circular cone
with the vertex at the origin, axis the z-axis and the semi-vertical
angle 0. Its equation is given by [See § 13 (B) case 11]
**+y2- z 2 tan* 0=0. ...(I)
The cone (I) will have three mutually perpendicular tangent
planes if
,bc+ca+ab=f*+g*+hv [See § 12]
or. I . (-ta n * 0 ) + ( - tan* 0).1 + 1 .1 = 0 + 0 + 0
or 2 tan2 0=1 or cot* 0 = 2 or 0=θοΙ-1·\/2.
Ex. 10. Lines are drawn from O with direction cosines propor
tional to ( 1,2, 2); (2, 3, 6); (3, 4, 12). Show that the axis o f the
right circular cone through them has direction cosines —I/V 3. I/V^*
l/\/3 and that the semi-vertical angle of the cone is cos'1 (l/y/3 ).
(Meerat 1973)
Sol. Clearly the vertex of the cone is at the origin 0(0, 0, 0).
Let the direction cosines of the axis of the right circular cone
be I, m, n. Let 0 be the semi-vertical angle.'
The cone is passing through the given lines drawn from O
and so these given lines are generators of the cone. Hence each
of these given lines with d.r.’s (I, 2, 2); (2, 3, 6); (3, 4, 12) i.e.
d.c.’s (1/3, 2/3, 2/3); (2/7, 3/7, 6/7); (3/13, 4/13, 12/13) is inclined
at an angle 0 to the axis of the cone whose d.c.’s are /, m, n.
We have
cos 0=(1/3) /+(2/3) »!+(2/3) n ...(I)
cos 0=(2/7) /+(3/7) m + (6/7) n ...(2)
and cos 0= (3 /1 3 )/+(4/13) m+(l2/13) n. ...(3)
Subtracting (2) from (I), we get
0= (1/21)/+ (5/21)01-(4/21)0 or /+ 5 0 i-4 « = O . ...(4)
Again subtracting (3) from (I), we get
4/+1401-100= 0. -(5 )
Solving (4) and (5), we get
386 Analytical Geometry 3-D
I m n vW*+»»*+»*) I
—1 1 = I _ V{(-D*+i*+'*} " V 3 ‘
Λ The d.c.*s of the axis of the cone are
- I I V h I I V h 1/V3.
Patting the values of I, m, n in (I), we get
CM " - ^ 7 3 + 3 ^ 3 + 3 ^ 3 = 3 ¾ = ^ or ψ ί
§14. The Enveloping Cone.
Definition. The enveloping cone o f a given surface is the locus
o f the tangent lines drawn from a given point to the given surface.
The enveloping cone Is also called the tangent cone to the surface
with the given point at Its vertex.
(A) The equation of the enveloping cone. To find the equa
tion o f the enveloping cone of the surface (conlcold) ax*+by*+ cz*=*I
with the vertex at the point (xu y u Z1)'. (Kanpur 1976, 78)
The equation of the given surface is
ex*+6y*+cz*=l. --(I)
The equation of any line through (X^y1, zt) are
( x - x 1)//= (y -y ,)/m = -(z -z ,)//i= r (say). . (2)
The co-ordinates of any point on the line (2) are
{lr+xu m r+ yu nr+z,). . (3)
Let the line (2) meet the given surface (I) at the point given
by (3). Then the co-ordinates of the point given by (3) will
satisfy the equation (I) and so we have
a. (Ir+ X1)*+ b (mr+ y, )*+ c (nr+ Z1)* = I
or (al*+bm*+cn*) r*+2 {alxi+bmyx+cnzx) r
+(ox,*+by,* + C z 1* - 1 ) = 0 . ...(4)
If the line (2) is a tangent to the given surface (I), then it
will meet the surface ( I) at two coincident point and hence the
two values of r obtained from (4) should be equal. The condition
for which is iBt =-AAC'
or 4 (alxx+bmyx.+ cnzx)*=4 (al*+bm*+cn*) (Ox1*+by,*+Cz1* - 1)
• or (alxx+bmyi+cnzx)*—(al*+bm*+cn*)(axx*+byx*+czx*—l) ...(5)
The enveloping cone of the surface (I) i.e., the locus of the
tangent line (2) is obtained by eliminating I, m, n between the
equations (2) and (5) and is therefore, given by
{a ( X - X 1) X1 + 6 ( y - y ,) yt + c (Z -Z 1) Z1)*
= { a ( X - X 1)1 + 6 ( y - y j ’ + c (Z -Z 1)*) (Ox1*+ ^y1*+ Cz1*—I )
The Cone 387
or {(axxx+ byyx+ Czz1- 1 ) - (axx*+byt*+ czx*- 1)}*
**{(ax*+by*+cz* —1 ) - 2 (fljcxj + f t ^ + c z z i — I)
+ (α*χ*+ byi*+czx*—I )} X (axx*+byi*+czi*—l). .. .(6)
For convenience let us set
Saax*+by* + cz* —I, S1=OX1H h y 1H i z 1* - Il
and T = axxx+byyx+czzx— I. J -·;(7)
Using the above notations, the equation (6) of the enveloping
cone becomes
( T - S i)* = (S -2 T + S 1).S 1
or T * -2 T S x+ S x*=SSx- 2 T S 1+ S l*
or SS1=T*
where S, Si and T are given by equations (7).
SOLVED EXAMPLES (H)
Ex. I. Find the equation o f the enveloping cone o f the sphere
\ x H y H z 2= a 2 with the vertex at the point ( x , , J11, Zi).
(Punjab 1975; Allahabad 75)
Sol. Proceeding as in § 14 (A), the enveloping cone is given
by
(X2+y* + z*—a*) (X iH y ,2+ Z1* - a*)=(xxx+ yyx+ zz,- a*)*.
Ex. 2. Find the equation o f the enveloping cone o f the ellipsoid
x*/a*+y*lb*+z*lc*=‘ I with the vertex at the point (xt, y x, zt).
Sol. Proceeding as in § 14 (A), the equation of the enve
loping cone is given by
S = X i + y2+ z2—2az= 0.
Λ Sx= χ χ*+ γλ*+Zxi - I a z i and T=xxx+yyx+zzx—a {ζ+ ζχΧ
Tbe equation of the enveloping cone of the sphere S= 0 with
the vertex at P (x lt ylt zt) is given by SS1= T i
l e. (χ2+ γ*+ ζ%~2αζ) (Xxi +yxi +zxi—2azi)
= {xxx+ y y i+ z z i-a (z+ zt)}2 ...(I)
The plane z = 0 cuts the cone (I) in the conic given by
(*'+y*) (Xxi +yx2+z12-2azi)=(xxx+yyi-a z i)i, z= 0
or Xi {yxi + z x i — 2 a z x ) — 2 x x y l x y + y 2 ( χ ^ + Ζ ι 2— 2 α ζ χ)
+ 2 αΧ χ Ζ χ Χ + ^ a y j Z i y - O i Zx2= O t Z = 0. ...(2)
Now we know that the conic
ax2+2hxy+by2+ 2gx+ 2fy+ c= 0, z= 0
represents a parabola if h2=ab.
Applying this condition the conic (2) will represent a para
bola if (—x Jy i)t =(yxi +ZLi -2 a z 1) {Χχ2+Ζχ*—2αζχ)
or Xtiyti r=xJiyxi +yxiZxi -2aziyxi +ZtiXii +Zi*—2az19- 2 a z iXx2
—2αΖχ*+4α2Ζχ*
or Z12 (Xx2+ y t i + z i i — 4 a z i + 4 a 2) — 2 a zi (Xi*+ ) ^ ) = 0
or Z1 (Xxi + y x i + z Ji - 4 a z i + 4 a 2) - 2 a (Xii - ^ i i)=O.
The locus of the point P (X1, y Xt Zi) is
z (x2+ y2+ z2—4az+4o2) —2a ( X 1 - J - ^ 2) = O .
Ex. 7. The section o f the enveloping cone o f the ellipsoid
x i/a2+ y2!bt = zilci = I whose vertex is P (x Jt y u Z i ) by the plane
Z=Ois(I) a parabola, (it) a rectangular hyperbola, (Ui) a circle.
Find the locus of P in the above three cases. (Meerut 1984R)
Sol. The equation of the given ellipsoid is
S s x iIa2+Ji2Ib2+ZiIci - 1 = 0 .
Therefore S i= Xi*/a2+yx2/b2+ Zx2Ic2- 1,
and T=xxxla2+yyx/b2+zzxlc2—l.
The equation of the enveloping cone with the vertex at
P (X uyuZ i) ^ SS1= T 2
or l ^ + y l + t .. i \ /½!+*%?!? _ i \ = i H i + y j i + 2Si- I Y
\ a 2^ b 2 Ci M o* b2 T c * J \ a2 V ^ ci }
...(I)
The section of the enveloping cone (I) by the plane z= 0 is
or
390 Analytical Geometry 3-D
ΟΓ
...(3)
But XitIat +yS/b*+ZltIc2— I O since thp point P (xu Vi. Z1)
does not lie on the given ellipsoid.
Hence (3) gives Zi2Ict - 1=0 or z4*=»c* or zj = ± c.
.·. The locus of the point P (Xi, y u Zi) is z==±c.
Case (ii). If the conic (2) represents a rectangular hyperbola,
then we should have
th e c o e ffic ie n t O f x 1- H b e c o e ffic ie n t o f V2= O
··· (3)
and Xi1Vi=O i.e. X1=O or Vi=O.
If Xj=O then from (3), we have
or Z i l + 5l / L - U = L - L
a2b2 c* \a a b2J a2 b2
or c*yi*+(b*—a2) Z12^ i b 2-Q 2) ca.
in this case the locus of P (xle y u zx) is
c2y 2± (b 2—a2) 2*«=t2 (b2—a2),
The Cone 391
or
ΜΜ-ρβ,+*-')
c2Xi*+(a2—b*) 2,2= c2 (a* - b2).
in this case the locus of P (x,, y„ z t) is
c*x®+(a* -ft2) Zt - C t (a*—b*)t y = 0.
Ex. 8. Find the locus o f a luminous point which moves so that
the ellipsoidXtJa2-^y2Ibt +ZtIct =] casts a circular shadow on the
plane 2 = 0.
Sol. Let P (x,, yu 2,) be the luminous point. It is required
to find the locus of P when the section of the enveloping cone
of the given ellipsoid with the vertex at P (x,, y it r,) by the
plane z —0 is a circle. Now proceed as in the case (iii) of Ex. 7
above.
Ex. 9. Show that three mutually perpendicular tangent lines
can be drawn to the sphere x t +y*+z2= ri from any point on the
sphere χ 2+>»2+ 2*=(3/2) r2.
Sol The equations of the given spheres are
x 2+ y2+ z2= r2 ...(I) and x t + y2+ zt =(3l2) r*. ...(2)
Let P (x,, y,, z,) be any point on the sphere (2), then
*ι, + Λ , +*ι*-(3/2) r·. .. (3)
Now three mutually perpendicular tangent lines can be drawn
to the sphere (I) from the point P if the enveloping cone of the
sphere (I) with the vertex at P has three mutually perpendicular
generators.
For the sphere (I), we have
S = x t + yt + zt - r t, Sl= x 1i + yl2+ zl3- r 2
and T = X X 1 + yyt + 22, —r*.
The enveloping cone of the sphere (I) with the vertex at
P ( x i , y u Zi) is tSS i = T 2'
or ( x 2 + y 2 + z 2 - r 2) (Xi* +.V,2+ z *—r2) =(xx, + y y x + 22, —r *)*.
Ex. 10. Find the locus o f points from which three mutually
perpendicular tangent lines can be drawn to the paraboloid
ax*+by*=2cz.
Sol.' Let P (xi. y u Zi) be any point whose locus is required.
Here S=ax*+by*—2cz, S ^ a x S + b y S —2czlt
and T = axxi+ byyt —c ( z + z t).
A The enveloping cone of the given paraboloid with the
vertex at P (Xu yu Zi) is tSSi = Tv ,
i.e. (ax*+by*—2cz) (axS+ byt*—2czt)
= {axxt +byyt - c (z+ z,)}*. ...(I)
Now three mutually perpendicular tangent lines can be drawn
from P (X|, P1, Z1) to S=O if the enveloping cone (I) has three
mutually perpendicular generators, the condition for which is that
in the equation (I) the sum of the coefficients of x2, p2 and z2
should be equal to zero
i.e. a (byS —2czt) + b (axS—2czt) —c2=0
o r. ob (Xi2+Pi*) —2c (α+h) Zi =C*.
:. The locus of P (Xi, yit Zi) is
. ab (X2-Hp*)-2c (a+b) Z=c*.
Ex. II. Find the locus o f the points from which three mutually
perpendicular tangent lines can be drawn to the conicoid
ax*+by*+CZ2=»i.
Sal. Proceeding as in Ex. 10 above the equation of the
required locus is
a (h-Hc) x*+h (c+a) y*+c (β+h) z*=a+b+c.
Exercises
1. Find the equation to the cone whose vertex is the origin
and whose base curve is the circle z=3, X2-Hp2-=9.
Ans. X2-Hp2- Z 2=O.
2. Find the equation of the cone with the vertex at the
origin and which passes through the curve
Ux2-Hhp2-Hcz2= I , ax2+/3p2=2z. (Kanpur 1981)
Ans. 4z2 (ax*+by*+cz*)=(c^x*+βy*)*.
3. Find the equation ot the cone with vertex at the origin
and direction cosines of its generators satisfying the relation
3/2—4m2-Hin2=O.
Ans. 3x2—4p2-Hiz2=O.
The Cone 393
Ue. the plane The d.r.’s of the chord PQ are 2xu 0, 0 Ue.
I, 0, 0 showing that the chord PQ is perpendicular to the plane
YOZ. Hence we conclude that the co-ordinate plane YOZ bisects
all· chords perpendicular to it. Similarly we can prove that the
co-ordinate planes ZO X and JfOFalso bisect all chords perpendi
cular to them respectively.
Hence the central conicoid (2) is symmetrical with respect to
all the three co-ordinate planes and these planes are called the
principal planes of the central conicoid.
The three principal planes (f.e. the co-ordinate planes in this
case) taken in pairs intersect in three lines (ie. the co-ordinate
axes in this case) which are called the principal axes of the central
conicoid.
(c) The standard form (2) of the central conicoid can take
the following'three forms each of which represents a surface with
a definite name assigned to i t :
x* y* 2* ,
(i) 3 Ϊ+ 5 5 + 3 Ϊ"= 1 (Ellipsoid)
* ' y2 2* ,
(ii) flV (Hyperboloid of one sheet)
X* y % Zi
(iil) (Hyperboloid of two sheets)
a1 ~ b*~ c- “ -
Now in the following articles we shall discuss the above three
surfaces (i.e. central conicoids) in details.
§ 2. The I7Ilipsoid.
The standard equation ot the ellipsoid is given by
X2Ia9+ y 2/b2+ z%jc2--1. .. (I)
(i) The Origin is the centre of the ellipsoid given by (I). Pro
ceeding exactly as in § I (a) we see that all the chords passing
through the origin are bisected at the origin. Therefore the surface
(I) has the origin as it centre.
(ii) The co-ordinate planes bisect all chords perpendicular to
them. Proceeding exactly as in § 1(b), we see that the co-ordinate
planes bisect all chords perpendicular to them. Hence the ellipsoid
(I) is symmetrical " ith respect to all the three cc-ordinate planes
and these planes arc called the principal planes of the ellipsoid.
T hethreeprincipalplanestakenin pairs intersect in three
lines (i.e. the co-ordinate axes in this case) which are called the
principal axes of the ellipsoid.
396 Analytical Geometry 3-Z>
with respect to all the three co-ordinate plaoes and these planes
are called the principal planes of the surface (I).
The three principal planes taken in pairs intersect in three
lines (/.^. the co-ordinate axes in the present case) which are called
the principal axes of the surface.
(iii) The intercepts on the co-ordinate axes. The equations of
the x-axis are .V=O, z=0. It meets the surface (I) in the points
A(a, 0, 0) and A \ —af 0, 0). Therefore, the surface (I) intercepts
a length 2a on the x-axis. In a similar manner the surface (I)
intercepts a length 2b on the .y-axis. The z-axis i.e. x = 0 , y= 0
meets the surface (I) in imaginary points [ z ^ ± V ( - " c)] *-e· l^e
z-axis does not meet the surface (I) in real points.
(iv) The sections of the hyperboloid of one sheet by the planes
parallel to the co-ordinate planes. The equation of any plane
parallel to AOir plane is given by 2 ^=»λ. Thesection of the sur
face (I) by the plane z=*A is the ellipse given by
x 2/a2+ y 2lb2= I + X 2Ic29 z=A. ...(2)
The lengths of the semi-axes of the ellipse (2) are
a ^ (\+ X 2tc2) and by/(l+ X2j'c2)
and its centre is (0, 0, A) i.e. centre lies on the z-axis at a distance
A from the origin. Since the quantity VO + A2/c2) remains real
for all real values of A, we get an infinite number of elliptical
sections as A increases numerically from 0 to oo. Also the sizes
of these ellipses increase as the value of A increases i.e. as we go
away from the XOY plane.
In a similar way the sections of the surface (I) by the planes
Z= —Aare the ellipses on the other !sides of the plane XOY.
The section of the surface (I) by the plane jc=A (i.e. plane
parallel to YOZ plane) is the hyperbola given by
y2/b2—Z2Ic2= I - X2Ia29 x=X.
Similarlythesectionof the surface (I) by the plane ^=A
(i.e. a plane parallel ZOX plane) is the hyperbola given by
X2Ia2- Z 2Ic*= I - A 2Ib29y = A.
<v) Sketch. From the above discussion it is possible to draw
a rough sketch of the hyperboloid of one sheet and is given
below :
Central Contcotds 399
Ifthe line (2) is a tangent lin etoth e conicoid (I) at the point
(*ι> Λ . zi) then the two values of r should be coincident. Clearly
one value of r given by (5) is zero and hence in order that the
line (2) is a tangent line to the conicoid at (Jt1, y 1% Z1), the other
value of r should also be zero the condition for which is
alxi + bmyx+ cnzt = 0. ... (6)
The tangent plane to the conicoid at ( X 1 , y l9 Z1) is the locus
of such lines through (xl9 y l9 Z1) that satisfy the condition (6) and
so its equation is obtained by eliminating /, m, n between the
equations (2) and (6). Hence the required equation of the tangent
plane to the conicoid (I) at the point (X1, ^1, Z1) is given by
a (X -X 1) X1*4"b ( y - y x) y x+ c (Z -Z 1) Z1=O
or axxx+byyx+ Czz1= axxf + b y f + c z f
or O x x ^ b y y 1+CZZ1= I . using (4).
Rule. To write the tangent plane to the conicoid at the point
(*i, Z 1) replace x8 by XX1 y 2 by y y x and z2 by z z v
TH+ - ^ i + W r 4 " 2 - 4 + 6 - 4 or 4 - 4 .
which is true. Hence the given plane (2) touches the given coni
coid.
Ex. 3. Find the equations to the tangent planes to the hyper
boloid 2x*—6y*+3z*==5 which pass through the line
3 x -3 y + 6 z -5 = 0 = x + 9 p -3 z .
(Agra 1976; Rajasthan 74; M eant 86)
Sol. The equation of the given hyperboloid is
2x*—6y»+3z*=5 or (2/5) Xs- (6/5) y*+(3/5) z* ~ l. ...(I)
The equations of the given line are
3 x -3 p + 6 z —5=0, x+ 9 y—3z=0. ...(2)
The equation of any plane through the line (2) is
3x—3p+6z—5+A (x+ 9p—3z)=0
or (3+A) x + ( —3 + 9 λ )ρ + (6 —3λ) z= 5. ...(3)
If the plane (3) touches the hyperboloid (I), then applying
J f rthe condition /*/a-|-m*/6+/i*/c=p* [See § 6 (A)], we have
(5/2) (3+A)*+(—5/6) (—3+9A)*+(5/3)(6—3A)*=(5)*
or 15 (9+6Α+Λ*)—5 (9-54A+81A*)+10 (3 6 - 3fA+9A*)= 150
406 Analytical Geometry 3-D
or 3 (9+6λ4-λ2)~ (9 -5 4 λ + 8 1 λ 2) + 2 (36-36A+9A2)= 3 0
or —60A2-f 60—0 or A2= I or A= ± I.
When A= I, from (3) the equation of the tangent plane is
4jc+6y+3z = 5.
When A= - I , from (3) the equation of the tangent plane is
2 x -1 2 y + 9 z = 5 .
Ex. 4. Atangentplanetothe ellipsoid x^ja^+y^lb2+ Z2Ic2= I
meets the co-ordinate axes in points Pj Qand R. Prove that the
centroid o f the triangle PQR lies on the surface
flVjc2+Aa/>2+ c 2/z2= 9.
(Rohilkhand 1978; Kanpur 77; Agra 75; Meerut 85S, 89)
Sol. The equation of the given ellipsoid is
jrlaP+y^/P+z^lc2= I. ···G )
The equation of any tangent plane to the ellipsoid (I) is
[See Note to § c(B)]
lx+ m y+ nz= V (a*P + b2mz+cin?). .(2)
The plane (2) meets the co-ordinate axes in the points given
by
P {y/irr+ P m t+ c'n*)/!, 0, 0); Q {0, y/(a*F+b*m*+cV)//w, 0}
and R {0, 0, + b2m2+ c2/i2)/n}.
Let (α, β , γ ) be the co-ordinates of the centroid of the triangle
PQRj then
«- 0+0 I V i f P + M + c< n*)t
* (¥ + ? + ¥ -* ) ■ ( $ + T + ?? - λ * )
Now the equations of the lines through the origin and perpen
dicular to the planes (2) are
xll=y!m=z/n. ...(4)
Eliminating I9 m9 n between (3) and (4), the required locus of
the perpendicular (4) is given by
xr (a2—r*)+y* (b*-r*)+z* (c * -r2)= 0 . Proved.
Ex. 10. Show that the tangent planes at the extremities o f any
diameter o f an ellipsoid are parallel. (Rajasthan 1978)
Sol Let the equation of an ellipsoid be
x*!cfi+y*lb*+z*/c*= I. ...(I)
W eknow that the centre of the ellipsoid (I) is the origin.
Hence every line passing through (0, 0, 0) and intersecting the
ellipsiod (I) is a diameter of (I). Thus let the equations of a
diameter of (I) be
x/l= y Irn=^z/n. ...(2)
Any point on (2) is (Zr, wr, nr). If it lies on (I), then
r* (/Vn8+ m2/6*+ ;i2/c2) = I
or r= ± \/y/iP /tP + m P /P + P Ic9)= ±A (say).
Now the extremities of a diameter are
(/Af m \9 ηλ) and ( —/A, —wA, —ηλ).
The equations of the tangent planes to (I) at these points are
/Ax m \y ηλζ —I i lx my nz_ I
a~ΊΓ + P ' + C ,.2* · * &a* +5*"
‘ Λ* +■ 7i~
c* -(3)
—/Ax mky wAz . lx my nz__ _1_
and
a* “ 6* Ci = .......*
i e" a*
a* + b3 + o*~ -----λλ ..,(4)
The equations (3) and (4) are the equations of two parallel
planes.
Ex. 11. I f P be the point of contact of a tangent plane to the
ellipsoid which meets the co-ordinate axes in A9 B and C and PL9
PM9 PN are the perpendiculars from P on the axes9prove that
OL.OA=a\ OM .OB=P9ONOC=C2.
Sol. Let the equation of the ellipsoid be
jp/a*+y*lb*+z*lc*= I. ... ( I)
Let the co-ordinates of P be (a, /?, ;>), so that the equation of
the tangent plane to the ellipsoid (I) at P (a, β 9 y) is
a x/a'+ fiy/P+yz/c*= I. ... (2)
The plane (2) meets the co-ordinate axes in A9Bi and C and
so OA9 OB and OC are the intercepts made by (2) on the co-ordi
nate axes hence we have
OA=QiIoL9 O B = P t f9 OC=C2Iy.
412 Analytical Geometry 3-D
h * + Wtf=Pu * = 0. -(3 )
Now if the plane ( 2) touches the ellipse (I) then the line (3)
will touch the ellipse (I) in the xy-plane and the condition for
the same is
O2I12+ Fnti2=Pl2 or Pi=zV ia0I t+ P m l2).
Putting this value of pv in (2), the equation (2) becomes
i1x+ m 1y-\-n1z = V ( a2li2+ b'ni12). ...(4)
Similarly the equations of the other two tangent planes to
the ellipse (I) are
l2x+rn2y-\- n2z = V ia 2U2+ b2m% 2) .. .(5)
and Z3* + m9y + w3z= V ia 2U2+ b2ma2). .. -(6)
Squaring (4), (5) and (6) and adding, we get
*2 TZ12+ / 1 Σ πί^ + ζ- Σηχ2+ 2γζ Σ η ι^ + ΐζ χ Tzi1Z1
+ 2 x y TZ1ZW1=A2 TZ1H h 2 Στηχ2
or x2A + y l.\+z-.\-\-2yz.0+2zx.0 + ?xy.0=a2.\ +Zr.l
or *H pH *W +h8 t v TZi2= I i TZ1W1=-Oetc.]
which is the required locus.
§ 8. The Polar Plane. (Kanpur 1982, Meerut 84 R)
Definition. Let ax2+by2+cz2= I .(1)
be the equation o f a conicoid and Afa9β9 γ) be any point. Draw lines
APQ to meet the conicoid (I) in the points P and Q. The locusof the
point R such that AR is t h e harmonic mean of AP and AQ (i e.9 AP9
AR and AQ are in harmonic progression) is called the polar plane of
the point A w ith respect to the given conicoid.
(A) The equation of the polar plane.
The equations of any line through the point Α(<χ, β9γ) are
( χ - α )/l= iy ~fi)/m =iz - y)/zi=z* (say), ..(2)
where Z, m, n are the actual d c.’s of the line.
The co-ordinates of any point on (2) distant r from Λ(α, β 9 γ)
are Qr+α, mr+β, nr+ γ). Therefore, the distances of the points
P and Q where the line (2) meets the given conicoid (I) are given
by the quadratic in r2
a (Zr+«)2+ 6 im r+fi)'+ c (nr+y)*= I
or r2 (al2+bm2+cn2) + 2r ial&+bm$+cny)
+(A aH tyJH cy2- 1)=0. ...(3)
L etthetw o values of r be T1 and ra and let Z1= ^ P a n d
T9=sAQ. Now AP9 AR9 AQ are in harmonic progression.
416 Analytical Geometry 3-D
The equations (2) and (3) should represent the same plane
hence comparing their coefficients, we have
« /(* + k )J l / ( P + t o _ y l(c*+k) I
I m a H p
·*♦ « = * {<d+k)jp, β = « ι (**+*)//>, j = n {<r+k)jp
OtI Ik m il· m k ml· ok
W a ----=—, ft------ ——, y ----- = ----
* P P P P P P
Therefore, we clearly we have
a —QtIjp β —mb'fp j —mPlp t
I = ' m η p
•V The locns of the pole («* β , j) is
-T- OtIfp^ y -B ttitIp Z-HtPjp
/ “ m ~ η
This is a straight line with its direction cosines proportional
to /, in,« which are also the direction ratios of the normal to the
plane (2). Hence the locos is a straight line perpendicular to the
given plane (2).
Εκ. 4. Find the loots o f straight lines drawn through a fixed
point (m. β, γ) a t rigjht angles to their polar Hith respect to the
conicoid at*+ byh+cz *= I.
(Lnrknew 1976, 80; Kanpnr S2; Meerat 89)
SoL The equation of the given conicoid is
ajp+byr+cz*'= I. -.-(I)
The equations of any straight line through the point (a, /I, y)
are (x —x )!I= iy —fi)!m = {z -y);n = r (say). --(2)
The co-ordinates of any point on (2) are (lr+ α, ra rjA n r 'r j).
The equation of the polar plane of this point w.r.t. the conicoid
(I) is ax {lr+ x)+ b y {m r+ fi)+ cz {nr+ 7)= I
or (aax+ b$y+ cyz— l)+ r (aix+ bm y+ cnz)= 0.
This plane for all values of r will pass through the line
a xx + b fiy+ c jz— I=0, alx+ bm y+ cnz= 0. ...(3)
This is the polar of the line (2) w.rX the conicoid (I).
Let I, ft, ·» be the direction ratios of the polar line (3) so that
we have
axh+ bflp+ cj*= 0, aIX vbm p+ cn*= 0.
b c {α β — m y) ca \1 γ -n a ) ~ a b ( m x —Ιβ) ---(5 )
Since the lines (2) and (3) are given to be perpendicular, we
have lk+ m p+ ny= 0
or Ibc (ιφ—my)-J-mai (fy - na)+nab (met—//?)—0.
Dividing bf abcimn throughout, we get
Central Comeoids 421
p _ _ i _ . y __ _°L. JL A =0
am an*bn bl ' d cm
or
-(5 )
The locus of the line (2) is obtained by eliminating I, m ,»
between (2) and (5) and hence is given Iqr
^ (K b 4 (K )^ (y 4 )-*
Ea. 5. I f P (Xu y i, Zi) and Q ix tx y » z j a n any two points
then fin d the equations o f the polar o f PQ with respect to the coni-
coid OJCi-J-Ajs-T-Cz2= I.
Sol. The equation of the given conkoid is
<uc*+bj*4 Czi = I. .:.(1)
Now the equations of the line joining P (xu J 1, r j and
¢ ( ¾ , I n Z1) *re
JC-
- —-Jr1 = -J——
- J 1 = ----
z—z, . .
L=r (say).
Jtf-Jfi y t —yi z t - * i
The co-ordinates of any point on this line PQ are
(*i+r (Xt - X i), y t+ r (J1- J 1)vZl-Hr (Zt - Z 1)).
The polar plane of this point w.r.t. the conkoid (I) is
a x (JC1-Hr (Xt - X1IJ-Hhj (J1-Hr (J1- J 1)Hcz (z,+r (Z1-Z 1)J=I
or (axx1+ byyi + czz1— \)+ r {ax (Xt X1) + by (J1- J 1)
-Hcz (Z1-Z 1)J=O.
This plane for values of r will pass through the line given by
th e p la n e s
O XX14 £>JJi+ C ZZ1 = I .(2 )
and a x ( X t - X 1) ^ b y (J 1- J 1)-H c z (Z 1- Z 1)= -O
i,e . «MTJTf+b y y t + C zz2= a x x t + b y y ^ c z z 1 -
IJ t„ a x x t + b y y t + C zzt = I. [u s in g ( 2 ) ] . -(3 )
The equations (2) and (3) are the required equations o. the
polar of the line PQ.
Ex. 6. I f P (xlr J 1, Z1) and Q (Jc1, J 1, Z t ) be any two points on
a conkoid axt+ by*+ cz*= I, show that the polar o f the line PQ is
the line o f intersection o f the tangent planes at P and Q. (AM. 1978)
SoL Proceeding as in Ex. 5 above the polar of the line PQ
is the line of intersection of the planes given by the equations (2)
and (3). But (2) and (3) are the equations of the tangent planes
to the conicoid (I) at the points P (X1, J 1, Z1) and Q (Xt, J 1, zt).
Hence the polar of the line PQ is the line of intersection of the
tangent planes at P and Q.
422 Analytical Geometry 3-D
O
r
0
δ.
al bm cn S
Vm V fi c'y I Vu V fi Vj I
Vl Vm c?n 0 Vl Vm Vn I
or (a -«') a (*- -V ) β (c—c') y =0, expanding
determinant
td bm cn along the fourth
column
a’l Vm c'n
or ( a - V ) « (b c '- b ’c) m n + (b -V ) β (<ca’- c ’a) nl
+(c—c') y (ab'—a’b) Im=O.
Dividing by Imn throughout, we get
(a—a’) (be'—Vc) (a//)+(b—V) (ca'—Va) {β/m)
+(c--c') (ab'—a'b) (y/n)=0. —(6)
The locus of the straight line (3) is obtained by eliminating
Umt n between (3) and (6), and therefore, it is given by
ifl- a ') (b c '-b 'c ) [ ~ ^ { b ~ b ' )
+ (C -O ( e b '- e 'b ) ^ ^ ) = 0 .
or ( « β V, ( * y \ i—I= Oυ
\ x - m H * —*
y - β ) z~ y /
(e y - β χ ) M Z -y x
or -O
vI
(y -fi)^ (x -* ) fr -r )
< *-« )
or ( y - β ) (ez—γ χ ) + ( ζ — y) (* y - β χ ) = 0 .
§ M. Locns of chords bisected at a given poiat. (PnnJab 1976)
Let the equation of the conicoid be
axt+ bjp+ c& = l. --(I)
Let f z , β , y) be the middle point of a chord of (I) so that its
equations may be given by
(x—a)JI= (y-fi)lm = (z-y)]n= r(s*y), ...(2)
where /, m , n are the actual d.c.’s o f the line.
Now the co-ordinates of any point on (2) at a distance r
from (α, β , y) are (ir - f t, m r+ β , n r+ γ ). Let this point lie on (I)
so that the distances of the points of intersection of the chord (2)
with the conicoid (I) from the point (η, β , γ ) are given by the
quadratic in r*
a (lr + * f + b (m r + fif+ c (n r+ y )* = I
or rr (e/*+ftmH-e»*)+2r (ala+bmfi+crry)
+(aa*+bfit + cy1-1 )= 0 . ...(3)
But («, β , γ) being the middle point of the chord (2), th e
distances of the points of intersection from (*, β, y) should be
4J4 Analytical Geometry 3-D
equal in magnitude but opposite in sign i.e. the sura of the two
-Wtltict of r should be zero and so we should have
. , __ —2 (altt+bmfi+crry) n
r*+ r *------- al’+bm'+cn*-------0
or al*+bmfi+cny—Q. ...(4)
Therefore, the locus of the chord (2) for varying values of
/, m, n is obtained by eliminating /, m, n between (2) and (4) and
is thus given by
act (λ - a)+h? ( y -fi)+ c y ( z ~ y )—0
Or aax+bfiy+ cyz= ao? 4 bfi*+ cy V
of' aajc+bfiy+cyz —\ —aa*+δβ*+ cy*—I
or T= S1 .(5)
where T and Si have their usual meanings.
Property. The section o f the conicoid by the plane (5) is a
conic with its centre at (α, β ,γ ).
We know that the section of a conicoid by a plane is a conic.
Here the given plane (5) is the locus of the chords of the coni
coid with their middle point as (α, β, y). Therefore, all the
chords of the conic section of the conicoid by the plane (5) and
passing through the point (α, β, y) are bisected at the point
(«, β, y) and hence by definition of centre the centre of the conic
is («, β >y)·
SOLVED EXAMPLES (C)
Ex. I. Find the equation to the plane which cuts 3jc*-|-2y*
—15z*=4 in a conic whose centre is at the point ( —2, 3, —I).
Sol. The equation of the conicoid is
3x* y* 15z*
S= • + 2 - ' -1 = 0
and the centre of the conic is ( - 2 , 3, —I).
We have
3 (-2 )* (3)* 1 5 (-1 )* 9 1_
*1— 4 — I-— 3--------1 = 3+ Λ 2’
and Τ = \ χ { - 2)+*τ ( 3 ) - i | l ( - l ) - - l » - | . x + ^ + ^ - i .
Λ The required equation of the plane is T = Sx
3 3 15
or or 2x - I y - 5 z+ 2 = 0 .
^ H Ans.
GZ· 2# Find the centre of the ■conic
**/9+yV16+z*/4-l, 2 x + 2 y -z = 3 .
Central Conicoids 425
Λ (®**+ft^*+cy*)*=p*(o*«*+i*^*+cV)-
Hence the locus of (a, 0, y) is the surface
(aj?+by*+cz*r=p9 (u*x*+*V +A *)-
Ex. 4. Shnr that the locus o f the middle pom is o f the chords
o f the canieoid as?+by*-rC 2?= I which passthrough a fix e d porri
(X09y 9 Z0) is ax ( x - x 0)+ by ( y - y ') + c z (Z -Z 0)=O.
(LaCknaw 1979, 82 ; Kaapm 79 ; IlKIwM 81)
SoL The equations of the given conicoid is
ax*+byi + cz* = \. —(I)
Let (a, 0 9 7) be the middle point of a chord of the corac oid
(I). Then this chord lies in the plane tT = S 19 Le.9
a o x+ b p y + c y z —I=aa*+bfP+cy*— I
or aax-\-bpy+cyz=aa*-\-bfi*+cy*. —( 2)
Now if the chords of the conicoid (I) having (ο, β 9 τ) as then-
middle point pass through the fixed point (X09J f9 z^, then the
point (X09 / , Z0) should lie on the plane (2).' Therefore, we have
a*x0+ b $ y'+ c/ ζ '= ao2+ b p * + cy
or asL ( * - x 0)+ b p (fi—y ')+ c y (y—zT)=0.
Hence the equation of the required locus of the middle point
(α, β 9γ ) of the chords of the conicoid (I) which pass through the
fixed point ( x \ / , z ) is
a x ( x —x 0)+ by (y —y ')+ c z (Z -Z 0)=Om
Ex. 7. Prove that the section o f the ellipsoid
x7 fl*+y*/h*+z*/c*=l
whose centre is at the point (in, £6, £c) passes through the extrem i
ties o f the axes . (Bahflhhaad M I)
Sol. The equation of the given ellipsoid is
s ^ /a ^ + y /ip + z - ic 2- 1 = o
-- The centre of the section is (Ja, \b 9 £c) and hence the equa
tion of the plane giving this section is 4T=S/
X jfl V j* Z jc ,
*· · a* + b* + c* l~ «» + -& r + ~*— 1
or xja+ yfb-t- z;c= I. _„(lj
The extremities of the axes of the given ellipsoid are (ο, 0, 0>,
(0, b, 0) and (0, 0, c) and the plane (I) clearly passes throagh these
three points. Hence the section whose centre is at the pnim
428 Amilytical Geometry 3-D
(2) on (I), therefore the sun of the roots {*>- of the above
equation should be zero and for it ire have
-Η*7 =θ or W ·)= - { c j i t f t . —PI
Now according to the given question if the chord 0 ) toadies
the sphere x*+jr+ 2*-r*t then we fare
the length of the perpendicular Arm the centre (CI9O9O) of the
sphere to Ac line (2 ) = ik radius r of the sphere
U. I °/ f+ | ° e* u °_ ' [ .H
or (my—flw}»+A F -H A ^= T e ( * * + ^
i v α*-Η**-ί-··=ΐ]
o im j jtm P-H "? Tn*| *?=** (**+**)
or (-r*+**) -?***=«
or ( -»*+**> J5
or t—
<-'·+**> {,+»£Μ **ίτ 'Γ- ®—*■·«
or (-* * + * ¾ w r - H V i - K i r + n v = · ·
The Iopb of (■ , /I, 7) is
(-1 --+jr*) (**r+A *)+?£y*+r?*J*=0
or -r V / A V + ^ j y + A V + r y + r z · f2 f c F ¥ = 0
or b jr i t o P + t f + c z · — f c ^ + c z * ί β ^ Η Η Ι ^ + β 5 - o r * ) = € L
py.
§11. N a B d to a C W M i.
(A) 7o /arf rfce qaainiB of lt f D rw f t» a UJtlral camira iJ Hf
the point (a, β , /). (Agra UQ; t i l i W · ! » , IHfcai 7·)
Let the cqnatioa of the central ooweoid be
e * + ^ * + e z * = l. —(11
The equation of the taagrm obne at ( c f , 7) of the cooicokl
(I) is
β χ χ + A flf T e y r = I - - .( 2 )
P = V I a V + b*fi*+ c2y2j ΟΓ A· * )
In view of (4) the actual direction cosines of the normal (3)
are a%p9 b$p9 cyp and hence the equations (3) of the normal to the
conicoid (I) at (a, 3, γ) in terms of actual direction cosines are
given by
*—q y - β ^ ζ - γ
cap αβρ ~~ayp .- (5)
(B) To find the equations of the normal to the ellipsoid
x2/a2+ y2lb2+ z2/c**=l at the point (α, β9 γ).
(Kanpur 1980; Locknow 81; Kornshetra 73)
Proceeding exactly as in § 11 (A), the equatipns of the normal
to the given ellipsoid at (α, β 9γ) are
χ — λ y — β ζ — γ
B H -W +B )-*
w j» (? + P + ? )"* ·
or ( α Φ + Ρ /Ρ + c V ) ( I/o*+ 1/6*+ 1/«*)=**
[V l/p*=a,<x, +6*/i*+cV>
.'. the point P(x, β, y) also lies on the surface
( a V + f c y + Λ » ) ( I/a* + 1/6*+ I /c*)= &*. -(3)
But the point P(x, β, y) lies on the given central conicoid (I)
and, therefore, the locus of P is the curve of intersection of the
conicoid (I) and the surface (3).
Ex. 2. Find the distance o f the points o f Intersection o f the
normal at P(x, β, y) to a central conicoid with the co-ordinate planes.
Sol. It is the same as the first part of Ex. I above.
Particular case when the given conicoid is an ellipsoid. Let the
central conicoid be, the ellipsoid
x*Ia*+y*Ibt ^ z tIct = I. --(I)
The equations of the normal to (I) at P(a, β, y) are
X *^ y - β _ z - y
apia» βρ/b* yplc* { y) ...(2)
where l/p*=«*/fl1+/3,/64+y*/c4 and where r denotes the actual
distance of any point on the normal (2) from P(«, β, y). If the
normal (2) meets the plane x = 0 in Gi then putting r=PGt in (2),
we get
,re ,— ·/*
Similarly if the normal (2) meets the co-ordinate planes y = 0
and z= 0 in G9 and G9 respectively then
PG9= —b*lp and PG9= -C tIp.
434 Analytical Geometry 3-D
= P P P [V and putting
the values of PGi etc. as
found in Ex. 2 above]
or r>= —(a8-f-b*+c*)/(3p).
Putting this value of r in (4), we get
a ( A 8- K b i -I-C i ) 12a * -b * -c* \
- 3a8----- = a ( 3a — ) ’
«_ 3ax,
so that
a ~ 2 a *-b *-c * ’
y __
Similarly β,
3b y, 3cz,
b 2 b 8 — c 8 — a8’ c ° 2 c 8 — A1—b2*
Putting these values in (3), we get
9a8JC,8 , 9b8y,8 . 9c87,8
( 2 A * - b , , - c 1,) 1 , + ( 2 b * - c 1,- A , ) 8 "t ’ ( 2 c 8 - A 8 - b i ) i
A* . **p* , A e _1_
(2 ^ -4 ^ -0 ^ ^ (2 * * - e*-«*j* + (2 e ·-··-* ·)* 9 Unmet
Ex. 4. Wimitke Iemgtkef tkemarmot ekard ikramgk Pef tke
ellipsoid χ*/«*+ρ*/»*+2*/«*= I amdpme Aatff it is efmol to 4PG*
where Gt is Ike potmt wkere tke mmmal ckord Jhramgk P meets tke
piame z=0, themP lies amtke came
x* (2c*-e*>K +J* (2c*-**>/ft*+a*A^=0.
SxL The equation o f the effipsoid is
J*/·*+**/*5+ **/**=!- --4*)
Let the co-ordinates of · point P oa (I) be (c, β. y \ so that
•*/«•+/»■/*»+,*/«*=1. <2)
The equations o f the n o n a l to the cflipsoid (I) at P (ο, ft. r)
are
where
p* a* ** c* (4)
The co-ordinates o f any point g (say) oa the Bbreal (3) are
« + (p u r/A ftHpflr/i^L r+tpyrft·) where r*=PQ-
VPQ n the nonaal chord of the ellipsoid (I) Ibea Q will lie
on (I) and so we get
or
** (?+£ +?)+M ?+£ +$M ii+S+? - 1H
or r ·' ■ * * « -< « )
__ —2
or
r e P* I**l+ )+ iF IIP i+ tfn ·)}
**PQ, the length of the noresal chord. ...(5)
Now if PQ=APG, (as given) then PQ= —4c?Ip
(See Ex. 2 above for the value of PGH
or —2 4e*
or
436 A n a fy tfe a i G eo m etry 3-D
Proved.
Seeooi part. The equations of the line Q R passing through
O (0. 0, 0) and drawn parallel to the normal at P are
x _ J _ z
= k (say).
CWe*) Iρβΐ**) Ipylct)
Taking 4= O R=G 3P - -C tIp. the co-ordinates of R are
C=-PGi cos Θ
or PGi cos θ+P'G’t cos ¢ '= 0 . Proveo.
Ex. 8. I f the normals at P and Q, points on the ellipsoid
XtIat -^ytIbt +z*/c*=l> intersect then prove that PQ is at right angles
to Its polar with respect to the ellipsoid.
Sol. The equation of the given ellipsoid is
x 2/a* -^y2Ibt +ZiIct = I. ...(I)
Let the co-ordinates of the points P and Q on (I) be
(«i. fiu yi) and (a*, fit, y») respectively.
The equations of the normals to the ellipsoid (I) at the points
P and Q are respectively given by
yi
«i Iat β,/b2 Yilct -(2 )
x - * i _ y - ^ i_ z - Y i
atf a2 β%Ib2 YiIc2 -0 )
If the normals (2) and (3) intersect, then we have
« ι-« * fii-fi* Y t-Y t
a t Ia* f i l l b* Y ilc t
Xi I a t fit/b \ Y tlc t
orb% Ct ...(8)
Let Iu mu /I1 be the d.c.’s of the polar line of PQ t-e. of the
line of intersection of the plane (6) and (8). Then we have
h — mi — "t
βι y»—ft»n *ιβ»—
b*c* c*a* a*b*
Therefore, if the line PQ [given by (5)] and its polar [whose
d.c/s are given by (9)] are to be perpendicular then applying the
condition for perpendicularity, we have
+ ( « —>■·) ( 11¾ . ! ^ 1) - 0
which is true by virtue of (4). Hence the result.
§ 12. Number of normals.
To prove that six normals can be drawn to an ellipsoid from a
given point (xj, y u Zi).
(Allahabad 1975, 78; Lucknow 71, 74, 77; Madras 76;
Meerut 86; Rohilkband 78; Punjab 72)
Let the equation of the ellipsoid be
x*lo*-\-y*/b*+z*lc*“ l. ...(I)
The equations of the normal to the ellipsoid (I) at P (α, β, y)
are
x —a _ y —β _ 2—y
λ (say).
«/α* β jb* γ /c* ( 2)
If the normal (2) passes through the given point (xu y u zt),
then this point will satisfy (2) aud hence we have
Xx~a _ y i— λ.
ala* β/b* γ /c*
γλ
Λ - 0 + fr. y+ Ct
or . b*yi* . Λ ,*
(a*+A)* + (b*+A)* + tc»+A)* “ ...(4)
[Putting the values from (3)]
This equation is of sixth degree in A and hence gives six
values of A. Puttiug these six values of A one by one in (3), we
get six values of α, β, y i e. we get six points on the ellipsoid (I)
the normals at which pass through a given point (Xi,yi, Zi).
Hencefrom a given point six normals can be drawn to an ellipsoid.
Corollary. Proceeding exactly as above, we can prove that
from a given point (Xl t ^1, Z1) six normals can be drawn to any
central conicoid ex*+by*+«·*=»I.
§ 13. Cubic curve through the feet of the normals.
To prove that the six feet of normals (drawn from a given point
to an ellipsoid) are the intersection o f a cubic curve with the given
ellipsoid.
Let the ,equation of the ellipsoid be
x*/<J, +y*/^i +z*/c*ea 1· ···(!)
Let ( X j , y„ Z1) be the given point. Now if the normal at
P { a , /3, y) to the ellipsoid (I) passes through (Xj, ylf Z1) then
proceeding as in § 12, we get
_ _ P Xi_ o P yi __
C1Z1
e*+A * bb*'V+ kA'· yr ~c*+ A"
_ c*-f A ...(2)
Now consider the curve whose parametric equations are
3, r j α*χ ι V J Pyi - , Pzl
fl*+A ’ y h*-|-A’ c*+A ...(3)
where Ais a parameter. We have proved in§12 that A has six
yhlues [given by equation (4) of § 12] and hence corresponding to
ieach of these six values of A, we shall get a point on the curve (3).
Again consider a plane A x+ B y+Cz+Dc=O. ...(4)
The points of intersection of the curve (3) with the plane (4)
are given by
.:.(5)
This equation being a cubic in Λ gives three values of λ and
shows that the curve (3) meets the plane (4) in three points and
hence the curve (3) is a cubic curve.
Hence we conclude that the six feet of the normals that are
442 Analytical Geometry 3-D
P lU b t - C t) + ^ - 1 (Ct -O t) + ^ ( a * - ? )
= ( « * + * ) ((>*— c * )+ (/> 2+ A ) ( c * _ o 2) + ( c * + A ) ( f l * - 0 ‘ )
«=0. ...(5)
E lim in a tin g I, m , n b e tw e e n (2 ) a n d (5 ), th e e q u a tio n o f th e
c o n e o n w h ic h th e n o rm a l (2 ) lie s is g iv e n b y ,
p x t (b 2+ 2 c 2) Pyl (Ct -Q t) DZ1 (as —b%)
x —x, y —y , ' +
+ Z -Z 1 ~
Λ, x< Ib2- C t) , y, ( C 2 - Q i ) , Z1 (a2—b2) Λ
·*-*. + y-y i + 2- Zi ...(6)
T h is is th e re q u ire d e q u a tio n o f th e c o n e o o w h ic h lie a ll th e
s ix n o rm a ls d ra w n fro m (X1, yt, Z1) to th e e llip s o id ( I ) .
Omiral Omicoids 443
Corollary. To prove that the curve through the six feet of the
normals drawn from a point {xu y u zi) 1° tin ellipsoid lies on the
cone through the six concurrent normals.
The parametric equations of the cubic curve through the feet
of six normals are given by [See § 13]
A r1 v- » y ±
β*+λ· y ~ p + \ ' ...(7)
where λ is a parameter.
If this curve is to lie on the cone (6) then the values of x %y, z
from (7) should satisfy the equation (6). Putting the values of
x 9y9 z from (7) in (6), we get
Σ (α2+λ) (fc2- c 2)= 0
* P Y
Now we clearly have
(--* * ) {c-- a * ) + c*) (a2-A J)
=A ( b * - c sf + X (C t - O i ) + A (ο*-**)=,0
444 Analytical Geometry 3-D
or a c * ) X 1 *= b (c*—o*) y x = c ( a * — 6 * ) Z 1.
(b * —
.·. The locus of P(xt, y t. zt) is the line given by
a (ft2—c*) x —b (c2—a*) y = c (a2—ft*) z.
Ex. 4. Prove that the lines drawn from the origin parallel to
the normals to ax*-\-by*-^czt = I at its points o f intersection with the
plane lx+ m y+ nz= p generate the cone
, lx* y* z*\ Ilx my nz \*
Η « +£ + γ Μ γ + 7 + Τ Γ ( 1977)
Sol. The equation of the given conicoid is
ax*+6y*+czf= l . • •(I)
The equation of the given plane is
lx+ m y+ nz= p. .( 2)
Let (α9β, γ) be a point of intersection of the conicoid (I) with
the plane (2), so that we have
α<χ*+ δβ2+ογ**=1 and la+mfi+ny=>p. —(3)
Now the equations of the normal to the conicoid (I) at
/ O Λ X-OL y - β ζ —γ
_ _ _ _ _ _ „ .,4 )
JC -α y —β ζ —γ
α/fl* “” /3/6a “ We* -« (2 )
Let the equations of the given line be
(*—Xi)lt= (y—yi)lm ~ (z—z1)ln ...(3)
If the normal (2) intersects the given line (3), then the lines
(2) and (3) are coplanar so that we have
· (4>
This is the equation of a conicoid. But the point (α, β, γ)
lies on the ellipsoid (I) as well. H encetbe locus of (α, β, y) is
the curve of intersection of the ellipsoid (I) and the conicoid (4).
Ex. 6. Two planes are drawn through the six feet o f the
normals drawn to the ellipsoid χ*Ια*+γ*Ιδ*+ζ*Ιο*·=1 from a given
point, each plane containing three. . Prove that if At and At be the
poles o f these planes with respect to the ellipsoid then
A1Att - O A 1* -O A tt (e*+h*+c*).
Sol. The equation of the ellipsoid is
xVa’ + y 1/*2+ ZiVci = I - -(1)
Let V (x,, y u zt) be the given point. Proceeding as in § 12,
the values of A giving the feet of six normals drawn from
P (*i, y u zt) to the ellipsoid (I) are given by
OtX1* CtZ1*
- 1= 0 .
(α * + Α )* 'Γ (6*+ ^ + λ )·
(ί»β+ Α )2 ------ ...(2)
Let the co-ordinates of the poles A1 and A3 of the two planes
be (xa, yt, Zi) and (xa, y3, z3) respectively, so that the equations of
the polar planes of A1 and A% are respectively ^jven by
XXt * * i , m i zz* m o
a* + b2 + c* ^ ...( 3 )
The feet of the six normals, namely
/ a*x, F y1 CtZt \
\at + h , b * + \ t c*+X J
where A has six values, lie on the two planes given by (3) and so
we have
448 Analytical Geometry 3-V
X t y% Ζ»
:. v »- Xi yi zi Xi y\ Zi
x t y t zt χ% y* Zi
x t y t zt Xi y* zz
I
= Xi X t X t X\ X% Xz
y i P t y t yi y% yz
Zi Zt Zt Zi Zz Z9
X1+ x 2+ x t ax y i+ y t + yt h/3 z i+ z t + z 8. cy
a* “V* b* “ p*· ' c* “V
Squaring and adding, we get
(*!+*»+*»)* O', , (Zi +Z js+ z,)* , a*a*+h*/3*+cV
a* ' + b* + c* + p*
Ex11 S jl*
or
a· + b* ·* Μ “ ( τ )+“ ( 7 )+ ( μ τγ )]
α*«*+ 6*j8*+ e*y*
Pi
or (1 + 1 + 1 +0+ 0+0)T ?«=a,a*+6,^ + c V
[using (4) and (3) of § 17]
or 3 (a*+/3»+y*)»=(a*a»+h*/9*+<?V), from (3).
The locus of («, β, γ) is
3 (x*+yt + zt)t saa*x, + bty*+cizt. Proved.
Ex. 10. I f λ, μ, v are the angles between a set o f equal conju
gate diameters of the ellipsoid x*la*+y*/b*+z*jc*=\, then show
that
, , . . , 3E (b * -c a)*
cos* A+«w* μ +cos* ν- 2 (0 , + 6»+ ^ ϊ ·
Sol. The equation of the given ellipsoid is
x*la*+y*/b*+2*/c*<=^l. -..(I)
Let OPt OQt OR be a set of equal conjugate semi-diameters
of the ellipsoid (I) so that
OP=- Ogc= OR= r (say).
We know that OP*+OQ*+OR*^a*+b*+c*.
Λ 3r*=a*+b*+c*. .v.(2)
If the co-ordinates of Pt Qt R are (xlt y u z,), (xt, y 2t zt ),
(xt, y t, Z3) respectively, then the d.r.’s of OQ and OR are
x tt y2t zt and x8, y3. z8 respectively. If λ be the angle between
OQ and OR, then
„„„ i _ x*x*+y*y*+ztza XiX9+ yty»+Z2Z2
V W + y S + z ^ V W + y S + z t ' ) ------------ Γ*
Λ cos· λ—(x2*t +jW s+*s*t)e/ '4· ...(3)
From Lagrange’s identity, we have
(xJ+ yS+ z*2) (X9*+yzt +Zs2) - ( x 2x3+ y2ys+ZiZ9)t c=Z {yizt - y t zt)%.
Putting from (3) on the L.H.S. and from the relations (6) of
§ 17 on the R.H.S. of this identity, we have
2 4 cos*. A, = P—ciXl' c'a'yi2 a*bW
.r2—r4 — - 4-, +-
a2
462 Analytical Geometry 3-D
_ P t m* . «· I /* , to2 , η* 3 i/»+m*+n·)
β·+ 4» + c*” r« ΟΓ α1 + 6* + c» " e*+6»+c» ...(6)
[Patting the value of T1Trom (S) and using the fact that
/*4·ιη1·4-«, =Β1]·
The relation (6) shows that the semi-diameter OP generates
the cone f.e., the locos of OP is the cone
Jt1 jp za 3 (X1-Kp1-I-Zs)
(P+ IP+ Ct a*+tP+c*
or (Xt ItP) (2( P - P - C t) M y aIb*) (2V - C t -(P )
+(ZtIct) (2c1—a1—b*)=0.
Exercises
1. Show that the plane x + 2 p + 3 z= 2 touches the conicoid
Jt1- 2jP+3z*=2 and find tbe point Of contact. (Meerut 1983)
Ana. ( 1 ,- 1 , 1 ) .
2. Find the equations of the tangent planes to the ellipsoid
7 ^ + 5 ^ + 3z*=60 which pass through the line
5y—3 z = 0 = 7 x + I Op—30. (Kanpur 1974).
Ana. 7χ+ 5ρ+ 3 ζ= 3 0 , 14x+10p+9z=50.
3. A tangent plane to tbe conicoid ax*+Ky2-f-cz, = l meets the
coordinate axes in P, Q and R. Prove that the centroid of
tbe triangle PQR lies on the surface
-L J - _!__g
axt+ ayt+ az*~
4. Show that the locus of the point of intersection of three
mutually perpendicular tangent planes to a central conicoid
is a sphere concentric with the conoicid.
(Meerut 1984 S)
[Hint. Refer § 7.]
5. Show that the locus of the centres of all sections of the
conicoid ex1+bp1+CZ1- I by planes which pass through a
fixed point (x', y', z') is
o x ( x - x ') + b y (y - y ') + c z (z -z ')= 0 .
6. Show that the centres of the sections of tbe ellipsoid
χ*1(Ρ+γ*ΙΙΡ+z*l(P==l that are (i) parallel to a given line
lie on a fixed plane and (ii) that pass through a given line lie
on a conic.
464 Analytteal Geometry 3-D
7. Prove that the six normals drawn from any point to a central
conicoid are the generators of a quadric cone.
(Meerat 1984,89 S)
[Hint. Proceed as in § 14.]
8. P r o v e that the feet of the six normals drawn to the central
conicoidax*+by*+ c z* = I from any point (*ι, Λ, *i) l*e 0,1
the curve of intersection of the given ellipsoid and the cone
a (b—c) Xi b (c—a) yt c (a—b)
x + y + z
(RoUlkhnnl 1977)